Buy BOOKS at Discounted Price

Volume And Surface Area Of Solids

Class 10th Mathematics RS Aggarwal Solution
Exercise 19a
  1. Two cubes each of volume 27 cm^3 are joined end to end to form a solid. Find…
  2. The Volume of a hemisphere is 2425 1/2 cm^3 . Find its curved surface area.…
  3. If the total surface area of a solid hemisphere is 462 cm^2 , find its volume.…
  4. A 5-m-wide cloth is used to make a conical tent of base diameter 14 m and…
  5. If the volumes of two cones are in the ratio of 1 : 4 and their diameters are…
  6. The slant height of a conical mountain is 2.5km and the area of its base is…
  7. The Sum of the radius of the base and the height of a solid cylinder is 37…
  8. The surface area of a sphere is 2464 cm^2 . If its radius be doubled, what will…
  9. A military tent of height 8.25m is in the form of a right circular cylinder of…
  10. A tent is in the shape of a right circular cylinder up to a height of 3 m and…
  11. A circus tent is cylindrical to a height of 3 m and conical above it. If its…
  12. A rocket is in the form of a circular cylinder closed at the lower and a cone…
  13. A solid is in the shape of a cone surmounted on a hemisphere, the radius of…
  14. A toy is in the form of a cone mounted on a hemisphere of same radius 7 cm. If…
  15. A toy is in the shape of a cone mounted on a hemisphere of same base radius.…
  16. A cylindrical container of radius 6 cm and height 15 cm is filled with…
  17. A vessel is in the form of a hemispherical bowl surmounted by a hallow…
  18. A toy is in the form of a cylinder with hemisphere ends. If the whole length…
  19. A medicine capsule is in the shape of a cylinder with two hemisphere stuck to…
  20. A wooden article was made by scooting out a hemisphere from each end of a…
  21. A solid is in the form of a right circular cone mounted on a hemisphere. The…
  22. From a solid cylinder whose height is 8 cm and radius 6 cm, a conical cavity…
  23. From a solid cylinder of height 2.8 cm and diameter 4.2 cm, a conical cavity…
  24. From a solid cylinder of height 14 cm and base diameter 7 cm, two equal…
  25. A metallic cylinder has radius 3 cm and height 5 cm. To reuse its weight, a…
  26. A spherical glass vessel has a cylindrical neck 7 cm long and 4 cm in…
  27. The given figure represent a solid consisting of a cylinder surmounted by a…
  28. From a cubical piece of wood of side 21 cm, a hemisphere is carved out in such…
  29. A cubical block of side 10 cm is surmounted by a hemisphere. What is the…
  30. A toy is in the shape of a right circular cylinder with a hemisphere on one…
  31. The inner diameter of a glass is 7 cm and it has a raised portion in the…
  32. A wooden toy is in the shape of a cone mounted on a cylinder, as shown in the…
Exercise 19b
  1. The dimensions of a metallic cuboid are 100 cm × 80 cm × 64 cm. It is melted…
  2. A cone of height 20 cm and radius of base 5 cm is made up of modelling clay. A…
  3. Metallic spheres of radii 6 cm, 8 cm and 10 cm respectively are melted to form…
  4. A solid metal cone with radius of base 12 cm and height 24 cm is melted to form…
  5. The radii of internal and external surface of a hollow spherical shell are 3 cm…
  6. The internal and external diameters of a hollow hemispherical shell are 6 cm…
  7. A copper rode of diameter 2 cm and length 10 cm is drown into a wire of uniform…
  8. A hemispherical bowl of internal diameter 30 cm contains some liquid. This…
  9. A solid metallic sphere of diameter 21 cm is melted and recast into a number of…
  10. A spherical cannon ball 28 cm in diameter is melted and recast into a right…
  11. A spherical ball of radius 3 cm is melted and recast into three spherical…
  12. A spherical shell of lead whose external and internal diameter are…
  13. A hemisphere of lead of radius 9 cm is into a right circular cone of height…
  14. A spherical ball of diameter 21 cm is melted and recast into cubes, each of…
  15. How many lead balls, each of radius 1 cm, can be made from a sphere of radius…
  16. A solid sphere of radius 3 cm is melted and them cast into small spherical…
  17. The diameter of a sphere is 42 cm. It is melted and drawn into a cylindrical…
  18. The diameter of a copper sphere is 18 cm. It is melted and drown into a long…
  19. A hemispherical bowl of internal radius 9 cm is full of water. Its contents…
  20. A hemispherical tank, full of water, is emptied by a pipe at the rate of 25/7…
  21. The rain water from a roof of 44 m × 20 m drains into a cylindrical tank…
  22. The rain water from a 22 m × 20 m roof drain into a cylindrical vessel of…
  23. A solid right circular cone of height 60 cm and radius 30 cm is dropped in a…
  24. Water is flowing through a cylindrical pipe of internal diameters 2 cm, into a…
  25. Water is flowing at the rate of 6 km/hr through a pipe of diameter 14 cm into…
  26. Water in a canal, 6 m wide and 1.5 m deep, is flowing at a speed of 4 km/hr.…
  27. A farmer connects a pipe of internal diameter 25 cm from a canal into a…
  28. Water running in a cylindrical pipe of inner diameter 7 cm, is collected in a…
  29. 150 spherical marbles, each of diameter 14 cm, are dropped in a cylindrical…
  30. Marbles of diameters 1.4 cm are dropped into a cylindrical beaker of diameter…
  31. In a village, a well with 10 m inside diameter, is dug 14 m deep. Earth taken…
  32. In a corner of a rectangular field with dimensions 35 m × 22 m, a well with…
  33. A copper wire of diameter 6 mm is evenly wrapped on a cylinder of length 18 cm…
  34. A right triangle whose sides are 15 cm and 20 cm (Other than hypotenuse) , is…
Exercise 19c
  1. A drinking glass is in the shape of a frustum of a cone of height 14 cm. The…
  2. The radii of the circular ends of a solid frustum of a cone are 18 cm and 12 cm…
  3. A metallic bucket, open at the top, of height 24 cm is in the form of the…
  4. A container, open the top, is in the form of a frustum of a cone of height 24…
  5. A container, open at the top and made up of metal sheet, is in the form of a…
  6. The radii of the circular ends of a solid frustum of a cone are 33 cm and 27…
  7. A bucket is in the form of a frustum of a cone. Its depts. Is 15 cm and the…
  8. A bucket made up of a metal sheet is in the form of frustum of a cone of height…
  9. A bucket made up of a metal sheet is in the form of frustum of a cone. Its…
  10. A container in the shape of a frustum of a cone having diameters of its two…
  11. A bucket is in the form of a frustum of a cone and its can hold 28.49 litres…
  12. The radii of the circular ends of a bucket of height 15 cm are 14 cm and r cm…
  13. The radii of the circular ends of a solid frustum of a cone are 33 cm and 27…
  14. A tent is made in the form of a frustum of a cone surmounted by another cone.…
  15. A tent consists of a frustum of a cone, surmounted by a cone. If the diameter…
  16. The perimeters of the two circular ends of a frustum of a cone are 48 cm and…
  17. A solid cone of base radius 10 cm is cut into two parts through the midpoint…
  18. The height of a right circular cone is 20 cm. A small cone is cut off at the…
  19. A solid metallic right circular cone 20 cm high and whose vertical angle is…
  20. A fez, the cap used by the Turks, is shaped like the frustum of a cone. If its…
  21. An oil funnel made of tin sheet consist of a 10 cm long cylindrical portion…
Exercise 19d
  1. A river 1.5 m deep and 36 m wide is flowing at the rate of 3.5 km/hr. Find the…
  2. The volume of a cube is 729 cm^3 . Find its surface area.
  3. How many cubes of 10 cm edge can be put in a cubical box of 1 m edge?…
  4. Three cubes of iron whose edges are 6 cm, 8 cm and 10 cm respectively are…
  5. Five identical cubes, each of edge 5 cm, are placed adjacent to each other.…
  6. The volumes of two cubes are in the ratio 8:27. Find the ratio of their surface…
  7. The volume of a right circular cylinder with its height equal to the radius is…
  8. The ratio between the radius of the base and the height of a cylinder is 2:3.…
  9. The radii of two cylinder are in the ratio of 2:3 and their height are in the…
  10. 66 cubic cm of silver is drawn into a wire 1 mm in diameter. Calculate the…
  11. If the area of the base of a right circular cone is 3850 cm^2 and its height…
  12. A cylinder with base radius 8 cm and height 2 cm is melted to form a cone of…
  13. a right cylindrical vessel is full of water. How many right cones having the…
  14. The volume of a sphere is 4851 cm^3 . Find its curved surface area.…
  15. The curved surface area of a sphere is 5544 cm^3 . Find its volume.…
  16. The surface areas of two sphere are in the ratio of 4:25. Find the ratio of…
  17. A solid metallic sphere of radius 8 cm is melted and into spherical balls each…
  18. How many lead shots each 3 mm in diameter can be made from a cuboid of…
  19. A metallic cone of radius 12 cm and height 24 cm is melted and made into…
  20. A hemisphere of lead of radius 6 cm is cast into a right circular cone of…
  21. A copper sphere of diameter 18 cm is drawn into wire of diameter 4 mm. Find…
  22. The radii of the circular ends of a frustum of height 6 cm are 14 cm and 6 cm…
  23. Find the ratio of the volume of a cube to that of a sphere which will fit…
  24. Find the ratio of the volumes of a cylinder, a cone and a sphere, if each has…
  25. Two cubes each of volume 125 cm^3 are joined end to form a solid. Find the…
  26. Three metallic cubes whose edges are 3 cm, 4 cm and 5 cm, are melted and…
  27. A solid metallic sphere of diameter 8 cm is melted and drawn into a…
  28. A 5-m-wide cloth is used to make a conical tent of base diameter 14 m and…
  29. A wooden toy was made by scooping out a hemisphere of same radius from each…
  30. Three cubes of a metal whose edge are in the ratio 3:4:5 are melted and…
  31. A hollow sphere of external and internal diameter 8 cm and 4 cm respectively…
  32. A bucket of height 24 cm is in the form of frustum of a cone whose circular…
  33. The interior of a building is in the form of a right circular of diameter 4.2…
  34. A metallic solid right circular cone is of height 84 cm and the radius of its…
  35. A toy is in the form of a cone of radius 3.5 cm mounted on a hemisphere of…
  36. If the radii of the circular ends of a bucket 28 cm high, are 28 cm and 7 cm,…
  37. A bucket is in the form of a frustum of a cone with a capacity of 12308.8 cm^3…
  38. A milk container is made of metal sheet in the shape of frustum of a cone…
  39. A solid metallic sphere of diameter 28 cm is melted and recast into a number…
  40. A cylindrical vessel with internal diameter 10 cm and height 10.5 cm is full…
Multiple Choice Questions (mcq)
  1. A cylindrical pencil sharpened at one edge is the combination ofA. a cylinder and a…
  2. A shuttlecock used for playing badminton is the combination of A. Cylinder and a…
  3. A funnel is the combination of delta A. a cylinder and a cone B. a cylinder and a…
  4. A Sarahi is a combination of delta A. a sphere and a cylinder B. a hemisphere and a…
  5. The shape of a glass (tumbler) is usually in the form of A. a cylinder B. frustum of a…
  6. The shape of a Gilli in the Gilli-Danda game is a combination of A. a cone and a…
  7. A plumbline (sahul) is the combination of A. A hemisphere and a cone B. A cylinder and…
  8. A cone is cut by a plane parallel to its base and the upper part is removed. The part…
  9. During conversion of a solid from one shape to another, the volume of the new shape…
  10. In a right circular cone, the cross section made by a plane parallel to the base is…
  11. A solid piece of iron in the form of a cuboid of dimensions (49 cm × 33 cm × 24 cm) is…
  12. The radius (in cm) of the largest right circular cone that can be cut out from a cube…
  13. A metallic solid sphere of radius 9 cm is melted to form a solid cylinder of radius 9…
  14. A rectangular sheet of paper 40 cm × 22 cm, is rolled to form a hollow cylinder of…
  15. The number of solid sphere, each of diameter 6 cm, that can be made by melting a solid…
  16. The surface areas of two sphere are in the ratio 16:9. The ratio of their volume isA.…
  17. If the surface area of a sphere is 616 cm^2 , its diameter (in cm) isA. 7 B. 14 C. 28…
  18. If the radius of a sphere becomes 3 times then its volume will becomeA. 3 times B. 6…
  19. If the height of a bucket in the shape of frustum of a cone is 16 cm and the diameter…
  20. A sphere of diameter 18 cm is dropped into a cylinder vessel of diameter 36 cm, partly…
  21. A solid right circular cone is cut into two parts at the middle of its height by a…
  22. The radii of the circular ends of a bucket of height 40 cm are 24 cm and 15 cm. The…
  23. A solid is hemispherical at the bottom and conical (of same radius) above it. If the…
  24. If the radius of the base of a right circular cylinder is halved, keeping the height…
  25. A cubical ice-cream brick of edge 22 cm is to be distributed among some children by…
  26. A mason constructs a wall of dimension (270 cm × 300cm × 350cm) with bricks, each of…
  27. Twelve solid sphere of the same size are made by melting a solid metallic cylinder of…
  28. The diameter of two circular ends of a bucket are 44 cm and 24 cm, and the height of…
  29. The slant height of a bucket is 45 cm and the radii of its top and bottom are 28 cm…
  30. The volumes of two Spheres are in the ratio 64: 27. The ratio of their surface areas…
  31. A hollow cube of internal edge 22 cm is filled with spherical marbles of diameter 0.5…
  32. A metallic spherical shell of internal and external diameter 4 cm and 8 cm…
  33. A medicine capsule is in the shape of a cylinder of diameter 0.5 cm with two…
  34. The length of the longest pole that can be kept in a room (12 m × 9 m × 8 m) isA. 29 m…
  35. The length of the diagonal of a cube is 6 √ 3 cm. Its total surface area isA. 144 cm^2…
  36. The volume of a cube is 2744 cm^3 . Its surface area isA. 196 cm^2 B. 1176 cm^2 C. 784…
  37. The total surface area of a cube is 864 cm^2 . Its volume isA. 3456 cm^3 B. 432 cm^3…
  38. How many bricks each measuring (25 cm × 11.25 cm × 6 cm) will be required to construct…
  39. The area of the base of a rectangular tank is 6500 cm^2 and the volume of water…
  40. The volume of a wall, 5 times as high as it is broad and 8 times as long as it is…
  41. If the areas of three adjacent faces of a cuboid are x, y, z respectively then the…
  42. The sum of length, breadth and height of a cuboid is 19 cm and its diagonal is 5 √ 5…
  43. If each edge of a cube is increased by 50%, the percentage increase in the surface…
  44. How many bags of grain can be stored in a cuboidal granary (8 m × 6 m × 3 m), if each…
  45. A cube of side 6 cm is cut into a number of cubes each of side 2 cm. The number of…
  46. In a shower, 5 cm of rain falls. The volume of the water that falls on 2 hectares of…
  47. Two cubes have their volumes in the ratio 1:27. The ratio of their surface areas isA.…
  48. The diameter of the base of a cylinder is 4 cm and its height is 14 cm. The volume of…
  49. The diameter of a cylinder is 28 cm and its height is 20 cm. The total surface area of…
  50. The height of a cylinder is 14 cm and its curved surface area is 264 cm^2 . The volume…
  51. The curved surface area of a cylinder is 1760 cm^2 and its base radius is 14 cm. The…
  52. The ratio of the total surface area to the lateral surface area of a cylinder with…
  53. The curved surface area of a cylindrical pillar is 264 m^2 and its volume is 924 m^3 .…
  54. The ratio between the radius of the base and the height of the cylinder is 2: 3. If…
  55. The radii of two cylinders are in the ratio 2:3 and their height in the ratio 5 : 3.…
  56. Two circular cylinder of equal volume have their height in the ratio 1: 2. The ratio…
  57. The radius of the base of a cone is 5 cm and its height is 12 cm. Its curved surface…
  58. The diameter of the base of a cone is 42 cm and its volume is 12936 cm^3 . Its height…
  59. The area of the base of a right circular cone is 154 cm^2 and its height is 14cm. Its…
  60. On increasing each of the radius of the base and the height of a cone by 20% its…
  61. The radii of the base of a cylinder and a cone are in the ratio 3:4. If they have…
  62. A metallic Cylinder of radius 8cm and height 2cm is melted and converted into a right…
  63. The height of the conical tent is 14m and its floor area is 346.5m^2 . How much…
  64. The diameter of a sphere is 14cm. Its Volume isA. 1428 cm^3 B. 1439 cm^3 C. 1437 1/3…
  65. The ratio between the volumes of two spheres is 8:27. What is the ratio between the…
  66. A hollow metallic Sphere with external diameter8cm and internal diameter 4cm is melted…
  67. A metallic cone having base radius 2.1 cm and height 8.4 cm is melted and moulded into…
  68. The volume of a hemisphere is 19404 cm^3 . The total surface area of the hemisphere…
  69. The surface area of a sphere is 154 cm^2 . The volume of the sphere isA. 179 2/3 cm^3…
  70. The total surface area of a hemisphere of radius 7 cm isA. (588π) cm^2 B. (392π) cm^2…
  71. The circular ends of a bucket are of radii 35 cm and 14 cm and the height of the…
  72. If the radii of the end of a bucket are 5 cm and 15 cm and it is 24 cm high then its…
  73. A circus tent is cylindrical to a height of 4 m and conical above it. If its diameter…
  74. Column I Column II (a) A solid metallic sphere of radius 8 cm is melted and the…
  75. Column I Column II (a) The radii of the circular ends of a bucket in the form of…
  76. Assertion (A): If the radii of the circular ends of a bucket 24 cm high are 15 cm and…
  77. Assertion (A): A hemisphere of radius 7 cm is to be painted outside on the surface.…
  78. Assertion (A): The number of coins 1.75 cm in diameter and 2 mm thick from a melted…
  79. Assertion (A): If the volumes of two sphere are in the ratio 27:8 then their surface…
  80. Assertion (A): The curved surface area of a cone of base radius 3 cm and height 4 cm…
Formative Assessment (unit Test)
  1. Find the number of solid sphere, each of diameter 6 cm, that could be moulded to Form a…
  2. Two right circular cylinder of equal volume have their height in the ratio 1:2. What is…
  3. A circus tent is cylindrical to a height of 4 m and conical above it. If its diameter…
  4. The radii of the top and bottom of a bucket of slant height 45 cm are 28 cm and 7 cm…
  5. A solid metal cone with radius of base 12 cm and height 24 cm is melted to form solid…
  6. A hemisphere bowl of internal diameter 30 cm is full of a liquid. This liquid is filled…
  7. A solid metallic sphere of diameter 21 cm is melted and recast into small cones, each…
  8. The diameter of a sphere is 42 cm. It is melted and drawn into a cylindrical wire of…
  9. A drinking glass is in the shape of frustum of a cone of height 21 cm with 6 cm and 4…
  10. Two cubes, each of volume 64 cm^3 , are joined end to end. Find the total surface area…
  11. The radius of the base and the height of a solid right circular cylinder are in the…
  12. A toy is in the form of a cone mounted on a hemisphere on a hemisphere of common base…
  13. A hemispherical bowl of internal radius 9 cm is full of water. This water is to be…
  14. The surface areas of a sphere and a cube are equal. Find the ratio of their volumes.…
  15. The slant height of the frustum of a cone is 4 cm and the perimeters (i.e. ,…
  16. A solid is composed of a cylinder with hemisphere ends. If the whole length of the…
  17. From a solid cylinder whose height is 15 cm and diameter 16 cm, a conical cavity of…
  18. A solid rectangular block of dimension 4.4 m, 2.6 m, 1 m is cast into a hollow…
  19. An open metal bucket is in the shape of a frustum of a cone, mounted on a hollow…
  20. A farmer connect a pipe of internal diameter 20 cm from a canal into a cylindrical…

Exercise 19a
Question 1.

Two cubes each of volume 27 cm3 are joined end to end to form a solid. Find the surface area of the resulting cuboid.


Answer:

Let the length of the side of cube be ‘a’ cm.


Volume of each cube = 27 cm3


Volume of cube = a3


∴ a3 = 27 cm3


⇒ a = (27 cm3)1/3


⇒ a = 3 cm


Length of a side of cube = 3 cm


Since, two cubes are joined and a cuboid is formed so,



Length of cuboid = l = 2a = 2 × 3 cm = 6 cm


Breadth of cuboid = b = a = 3 cm


Height of cuboid = h = a = 3 cm


Surface area of cuboid = 2 × (l × b + b × h + l × h)


∴ Surface area of resulting cuboid = 2 × (6 × 3 + 3 × 3 + 6 × 3) cm2


= 2 × (18 + 9 + 18) cm2


= 2 × 45 cm2


= 90 cm2


So, surface area of resulting cuboid is 90 cm2



Question 2.

The Volume of a hemisphere is . Find its curved surface area.


Answer:

Let the radius of hemisphere be r cm


Volume of hemisphere is given by πr3


Given, volume of hemisphere = 24251/2 cm3


π r3 = 24251/2


⇒ r3 = 4851 × 1/2 × 3/2 × 7/22


⇒ r3 = 1157.625 cm3


⇒ r = (1157.625)1/3 cm


⇒ r = 10.5 cm


Curved Surface Area of hemisphere = 2πr2


Curved Surface Area of hemisphere = 2 × 22/7 × (10.5)2 cm2


= 693 cm2


∴ Curved surface area of hemisphere = 693 cm2



Question 3.

If the total surface area of a solid hemisphere is 462 cm2, find its volume.


Answer:

Let the radius of solid sphere be r cm


Total surface area of solid hemisphere = 3πr2


Given, total surface area of solid hemisphere = 462 cm2


∴ 3πr2 = 462 cm2


⇒ 3 × 22/7 × r2 = 462 cm2


⇒ r2 = 462 × 1/3 × 7/22 cm2 = 49 cm2


⇒ r = 7 cm


Volume of solid hemisphere = π r3


= 2/3 × 22/7 × 73 cm3


= 718.67 cm3


∴ Volume of solid hemisphere is 718.67 cm3



Question 4.

A 5-m-wide cloth is used to make a conical tent of base diameter 14 m and height 24 m. Find the cost of cloth used at the rate of Rs 25 per metre.


Answer:

Width of cloth used = 5 m


Diameter of conical tent to be made = 14 m


Let the radius of the conical tent be r m


Radius of conical tent = r = diameter ÷ 2 = 14/2 m = 7 m


Height of conical tent = h = 24 m


Let the slant height of conical tent be l


So,


m2


⇒ l = 25 m


Area of cloth required to make a conical tent = Curved Surface area of conical tent


= πrl


= 22/7 × 7 × 25 m2


= 550 m2


Length of cloth used = Area of cloth required ÷ width of cloth


= 550/5 m


= 110 m


∴ Length of cloth used = 110 m


Cost of cloth used = Rs 25 per meter


Total Cost of cloth required to make a conical tent = 110 × Rs 25


= Rs 2750


∴ Total cost of cloth required to make a conical tent = Rs 2750



Question 5.

If the volumes of two cones are in the ratio of 1 : 4 and their diameters are in the ratio of 4 : 5, find the ratio of their heights.


Answer:

Let V1 be the volume of first cone and V2 be the volume of second cone.


Then, V1:V2 = 1:4


Let d1 be the diameter of first cone and d2 be the diameter of second cone.


Then d1:d2 = 4:5


Let h1 be the height of first cone and h2 be the height of second cone.


We know that volume of cone is given by V = 1/3 × π(d2/4)h










∴ h1:h2 = 25:64


∴ Ratio of height of two cones is 25:64.



Question 6.

The slant height of a conical mountain is 2.5km and the area of its base is 1.54 km2. Find the height of mountain.


Answer:

Let the radius of base be ‘r’ km and slant height be ‘l’ km


Slant height of conical mountain = 2.5 km


Area of its base = 1.54 km2


Area of base is given by πr2


∴ πr2 = 1.54 km2


⇒ 22/7 × r2 = 1.54 km2


⇒ r2 = 1.54 × 7/22 km2 = .49 km2


⇒ r = 0.7 km


Let ‘h’ be the height of the mountain


We know,


l2 = r2 + h2


Substituting the values of l and r in the above equation


2.52 = 0.72 + h2


h2 = 2.52 – 0.72 = 6.25 – 0.49 km2


h2 = 5.76 km2


h = 2.4 km


∴ Height of the mountain = 2.4 km



Question 7.

The Sum of the radius of the base and the height of a solid cylinder is 37 metres. If the total surface area of the cylinder be 1628sq metres, find its volume.


Answer:

Let the Radius of the solid cylinder be ‘r’ m and its height be ‘h’ m.


Given,


Sum of radius and height of solid cylinder = 37 m


r + h = 37 m


r = 37 – h


Total surface area of solid cylinder = 1628 m2


Total surface area of solid cylinder is given by 2πr (h + r)


∴ 2πr (h + r) =1628 m2


Substituting the value of r + h in the above equation


⇒ 2πr × 37 = 1628 m2


⇒ r = 1628 × 7/22 × 1/2 × 1/37 m


⇒ r = 7 m


Since, r + h = 37 m


h = 37 – r m


h = 37 – 7 m = 30 m


Volume of solid cylinder = πr2h


= 22/7 × 72 × 30 m2


= 4620 m2



Question 8.

The surface area of a sphere is 2464 cm2. If its radius be doubled, what will be the surface area of the new sphere?


Answer:

Let the radius of sphere be ‘r’ cm


Surface area of sphere = 2464 cm2


Surface area of sphere is given by 4πr2


∴ 4πr2 = 2464 cm2


⇒ 4 × 22/7 × r2 = 2464 cm2


⇒ r2 = 2464 × 1/4 × 7/22 cm2 = 196 cm2


⇒ r = 14 cm


Radius of new sphere is double the radius of given sphere.


Let the radius of new sphere be r’ cm


∴ r’ = 2r


r’ = 2 × 14 cm = 28 cm


Surface area of new sphere = 4πr’2


= 4 × 22/7 × 282 cm2


= 9856 cm2


∴ Surface area of new sphere is 9856 cm2.



Question 9.

A military tent of height 8.25m is in the form of a right circular cylinder of base diameter 30 m and height 5.5 m surmounted by a right circular cone of a same base radius. Find the length of canvas used in making the tent, if the breadth of the canvas is 1.5 m.


Answer:

The military tent is made as a combination of right circular cylinder and right circular cone on top.


Total Height of tent = h = 8.25 m


Base diameter of tent = 30 m


Base radius of tent = r = 30/2 m = 15 m


Height of right circular cylinder = 5.5 m


Curved surface area of right circular cylindrical part of tent = 2πrh


Height of conical part = total height of tent – height of cylindrical part


hcone = 8.25 – 5.5 m = 2.75 m


Base radius of cone = 15 m


Let l be the slant height of cone


Then, l2 = hcone2 + r2 = 2.752 + 152 m2


l2 = 7.5625 + 225 m2 = 232.5625


l = 15.25


Curved surface area of conical part of the tent = πrl


Total surface area of the tent = Curved surface area of cylindrical part + curved surface area of conical part


Total surface area of tent = 2πrh + πrl


= πr (2h + l)


= 22/7 × 15 × (2 × 5.5 + 15.25) m2


= 22/7 × 15 × (11 + 15.25) m2


= 22/7 × 15 × 26.25 m2


= 1237.5 m2


Breadth of canvas used = 1.5 m


Length of canvas used = Total surface area of tent ÷ breadth of canvas used


Length of canvas used = 1237.5\1.5 m = 825 m


∴ Length of canvas used is 825 m.



Question 10.

A tent is in the shape of a right circular cylinder up to a height of 3 m and conical above it. The total height of the tent is 13.5 m and the radius of its base is 14 m. Find the cost of cloth required to make the tent at the rate of Rs 80 per square metre. [Take π = 22/7.]


Answer:

The tent is made as a combination of right circular cylinder and right circular cone on top.


Height of cylindrical part of the tent = h = 3 m


Radius of its base = r = 14 m


Curved surface area of cylindrical part of tent = 2πrh


= 2 × 22/7 × 14 × 3 m2


= 264 m2


Total height of the tent = 13.5 m


Height of conical part of the tent = total height of tent – height of cylindrical part.


Height of conical part of the tent = 13.5 – 3 m =10.5 m


Let the slant height of the conical part be l


l2 = hcone2 + r2


l2 = 10.52 + 142 = 110.25 + 196 m2 = 306.25 m2


l = 17.5 m


Curved surface area of conical part of tent = πrl


= 22/7 × 14 × 17.5 m2


= 770 m2


Total surface area of tent = Curved surface area of cylindrical part of tent + Curved surface area of conical part of tent


Total Surface area of tent = 264 m2 + 770 m2 = 1034 m2


Cloth required = Total Surface area of tent = 1034 m2


Cost of cloth = Rs 80/m2


Total cost of cloth required = Total surface area of tent × Cost of cloth


= 1034 × Rs 80


= Rs 82720


Cost of cloth required to make the tent is Rs 82720



Question 11.

A circus tent is cylindrical to a height of 3 m and conical above it. If its base radius is 52.5 m and the slant height of the conical portion is 53 m, find the area of canvas needed to make the tent. [Take π = 22/7.]


Answer:

The Circus tent is made as a combination of cylinder and cone on top.


Height of cylindrical part of tent = h = 3 m


Base radius of tent = r = 52.5 m


Area of canvas required for cylindrical part of tent = 2πrh


= 2 × 22/7 × 52.5 × 3 m2


= 990 m2


Slant height of cone = l = 53 m


Area of canvas required for conical part of the tent = πrl


= 22/7 × 52.5 × 53 m2


= 8745 m2


Area of canvas required to make the tent = Area of canvas required for cylindrical part of tent + Area of canvas required for conical part of tent


Area of canvas required to make the tent = 990 + 8745 m2 = 9735 m2



Question 12.

A rocket is in the form of a circular cylinder closed at the lower and a cone of the same radius is attached to the top. The radius of the cylinder is 2.5 m, its height is 21 m and the slant height of the cone is 8 m. Calculate the total surface area of the rocket.


Answer:

The rocket is in the form of cylinder closed at the bottom and cone on top.


Height of cylindrical part rocket = h = 21 m


Base radius of rocket = r = 2.5 m


Surface Area of cylindrical part of rocket = 2πrh + πr2


= 2 × 22/7 × 2.5 × 21 + 22/7 × 2.5 × 2.5 m2


= 330 + 19.64 m2 = 349.64 m2


Slant height of cone = l = 8 m


Surface Area of conical part of the rocket = πrl


= 22/7 × 2.5 × 8 m2


= 62.86 m2


Total surface area of the rocket = Surface Area of cylindrical part of rocket + Surface Area of conical part of rocket


Total surface area of the rocket = 349.64 + 62.86 m2 = 412.5 m2



Question 13.

A solid is in the shape of a cone surmounted on a hemisphere, the radius of each of them being 3.5 cm and the total height of the solid is 9.5 cm. Find the volume of the solid.


Answer:

The solid is in the form of a cone surmounted on a hemisphere.


Total height of solid = h = 9.5 m


Radius of Solid = r = 3.5 m


Volume of hemispherical part solid = 2/3 × πr3


= 2/3 × 22/7 × 3.53 m3


= 89.83 m3


Height of conical part of solid = hcone = Total height of solid – Radius of solid


Height of conical part of solid = hcone = 9.5 – 3.5 = 6 m


Volume of conical part of solid = 1/3 × πr2hcone


= 1/3 × 22/7 × 3.52 × 6 m3


= 77 m3


Volume of solid = Volume of hemispherical part solid + Volume of conical part solid


Volume of solid = 89.83 + 77 m3 = 166.83 m3



Question 14.

A toy is in the form of a cone mounted on a hemisphere of same radius 7 cm. If the total height of the toy is 31 cm, find its total surface area.


Answer:

The toy is in the form of a cone mounted on a hemisphere.


Total height of toy = h = 31 cm


Radius of toy = r = 7 cm


Surface area of hemispherical part toy = 2πr2


= 2 × 22/7 × 72 cm2


= 308 cm2


Height of conical part of toy = hcone = Total height of toy – Radius of toy


Height of conical part of toy = hcone = 31 – 7 = 24 cm


Let the slant height of the conical part be l


l2 = hcone2 + r2


l2 = 242 + 72 = 576 + 49 cm2 = 625 cm2


l = 25 cm


Surface area of conical part of toy = πrl


= 22/7 × 7 × 25 cm2


= 550 cm2


Total surface area of toy = Surface area of hemispherical part of toy + Surface area of conical part of toy


Total Surface area of toy = 308 cm2 + 550 cm2 = 858 cm2



Question 15.

A toy is in the shape of a cone mounted on a hemisphere of same base radius. If the volume of the toy is 231 cm3 and its diameter is 7 cm, find the height of the toy.


Answer:

A toy is in the shape of a cone mounted on a hemisphere of same base radius.


Volume of Toy = 231 cm3


Base Diameter of toy = 7 cm


Base radius of toy = 7/2 cm = 3.5 cm


Volume of hemisphere = 2/3 πr3


= 2/3 × 22/7 × 3.53 cm3



Volume of cone = Volume of hemisphere – Volume of toy



Volume of cone is given by 1/3 πr2h


Where h is the height of cone



Height of cone = 11 cm


Height of toy = Height of cone + Height of hemisphere


= 11 cm + 3.5 cm = 14.5 cm


[Height of hemisphere = Radius of hemisphere]


∴ Height of toy is 14.5 cm.


Question 16.

A cylindrical container of radius 6 cm and height 15 cm is filled with ice-cream. The whole ice-cream has to be distributed to 10 children in equal cones with hemispherical tops. If the height of the conical portion is 4 times the radius of its base, find the radius of the ice-cream cone.


Answer:

Radius of cylindrical container = r = 6 cm


Height of cylindrical container = h = 15 cm


Volume of cylindrical container = πr2h


= 22/7 × 6 × 6 × 15 cm3


= 1697.14 cm3


Whole ice-cream has to be distributed to 10 children in equal cones with hemispherical tops.


Let the radius of hemisphere and base of cone be r’


Height of cone = h = 4 times the radius of its base


h’ = 4r’


Volume of Hemisphere = 2/3 π(r’)3


Volume of cone = 1/3 π(r’)2h’ = 1/3 π(r’)2 × 4r’


= 2/3 π(r’)3


Volume of ice-cream = Volume of Hemisphere + Volume of cone


= 2/3 π(r’)3 + 4/3 π(r’)3 = 6/3 π(r’)3


Number of ice-creams = 10


∴ total volume of ice-cream = 10 × Volume of ice-cream


= 10 × 6/3 π(r’)3 = 60/3 π(r’)3


Also, total volume of ice-cream = Volume of cylindrical container


⇒ 60/3 π(r’)3 = 1697.14 cm3


⇒ 60/3 × 22/7 × (r’)3 = 1697.14 cm3


⇒ (r’)3 = 1697.14 × 3/60 × 7/22 = 27 cm3


⇒ r = 3 cm


∴ Radius of ice-cream cone = 3 cm



Question 17.

A vessel is in the form of a hemispherical bowl surmounted by a hallow cylinder. The diameter of the hemisphere is 21 cm and the total height of the vessel is 14.5 cm. Find its capacity.


Answer:

Vessel is in the form of a hemispherical bowl surmounted by a hallow cylinder.


Diameter of hemisphere = 21 cm


Radius of hemisphere = 10.5 cm


Volume of hemisphere = 2/3 πr3 = 2/3 × 22/7 × 10.53 cm3


= 2425.5 cm3


Total height of vessel = 14.5 cm


Height of cylinder = h = Total height of vessel - Radius of hemisphere


= 14.5 – 10.5 cm = 4 cm


Volume of cylinder = πr2h = 22/7 × 10.5 × 10.5 × 4 cm3


= 1386 cm3


Volume of vessel = Volume of hemisphere + Volume of cylinder


= 2425.5 cm3 + 1386 cm3


= 3811.5 cm3


∴ Capacity of vessel = 3811.5 cm3



Question 18.

A toy is in the form of a cylinder with hemisphere ends. If the whole length of the toy is 90 cm and its diameter is 42 cm, find the cost of painting the toy at the rate of 70paise per sq cm.


Answer:

Toy is in the form of a cylinder with hemisphere ends


Total length of toy = 90 cm


Diameter of toy = 42 cm


Radius of toy = r = 21 cm


Length of cylinder = l = Total length of toy – 2 × Radius of toy


= 90 – 2 × 21 cm = 48 cm


For cost of painting we need to find out the curved surface area of toy


Curved surface area of cylinder = 2πrl


= 2 × 22/7 × 21 × 48 cm2


= 6336 cm2


Curved surface area of hemispherical ends = 2 × 2πr2


= 2 × 2 × 22/7 × 21 × 21 cm2


= 5544 cm2


Surface area of toy = Curved surface area of cylinder + Curved surface area of hemispherical ends


Surface area of toy = 6336 cm2 + 5544 cm2 = 11880 cm2


Cost of painting = Rs 0.70/cm2


Total Cost of painting = Surface area of toy × Cost of painting


= 11880 cm2 × Rs 0.70/cm2 = Rs 8316.00


Total cost of painting the toy = Rs 8316.00



Question 19.

A medicine capsule is in the shape of a cylinder with two hemisphere stuck to each of its ends. The length of the entire capsule is 14 mm and the diameter of the capsule is 5 mm. Find its surface area.


Answer:

A medicine capsule is in the shape of a cylinder with two hemisphere stuck to each of its ends.


Total length of entire capsule = 14 mm


Diameter of capsule = 5 mm


Radius of capsule = r = Diameter ÷ 2 = 5/2 mm = 2.5 mm


Length of cylindrical part of capsule = l = Total length of entire capsule – 2 × Radius of capsule


= 14 – 2 × 2.5 mm = 14 - 5 mm = 9 mm


Curved surface area of cylindrical part of capsule = 2πrl


= 2 × 3.14 × 9 × 2.5 mm2


= 141.3 mm2


Curved surface area of hemispherical ends = 2 × 2πr2


= 2 × 2 × 3.14 × 2.5 × 2.5 cm2


= 78.5 mm2


Surface area of capsule = Curved surface area of cylindrical part of capsule + Curved surface area of hemispherical ends


Surface area of capsule = 141.3 mm2 + 78.5 mm2 = 219.8 mm2



Question 20.

A wooden article was made by scooting out a hemisphere from each end of a cylinder, as shown in the figure. If the height of the cylinder is 20 cm and its base is of diameter 7 cm, find the total surface area of the article when it is ready.



Answer:

The wooden article was made by scooting out a hemisphere from each end of a cylinder


Let the radius of cylinder be r cm and height be h cm.


Height of cylinder = h = 20 cm


Base diameter of cylinder = 7 cm


Base radius of cylinder = r = diameter ÷ 2 = 7/2 cm = 3.5 cm


Lateral Surface area of cylinder = 2πrh


= 2 × 22/7 × 3.5 × 20 cm2


= 2 × 22/7 × 3.5 × 20 cm2


= 440 cm2


Since, the wooden article was made by scooting out a hemisphere from each end of a cylinder


∴ Two hemispheres are taken out in total


Radius of cylinder = radius of hemisphere


∴ Radius of hemisphere = 3.5 cm


Lateral Surface area of hemisphere = 2πr2


= 2 × 22/7 × 3.5 × 3.5 cm2


= 77 cm2


Total Surface area of two hemispheres = 2 × 77 cm2 = 154 cm2


Total surface area of the article when it is ready = Lateral Surface area of cylinder + Lateral Surface area of hemisphere


Total surface area of the article when it is ready = 440 cm2 + 154 cm2


= 594 cm2



Question 21.

A solid is in the form of a right circular cone mounted on a hemisphere. The radius of the hemisphere is 2.1 cm and the height of the cone is 4 cm. The solid is placed in a cylindrical tub full of water in such a way that the whole solid is submerged in water. If the radius of the cylinder is 5 cm and its height is 9.8 cm, find the volume of the water left in the tub.


Answer:

A solid is in the form of a right circular cone mounted on a hemisphere.


Let r be the radius of hemisphere and cone


Let h be the height of the cone


Radius of hemisphere = r = 2.1 cm


Volume of hemisphere = 2/3 πr3


= 2/3 × 22/7 × 2.1 × 2.1 × 2.1 cm3


= 19.404 cm3


Height of cone = h = 4 cm


Radius of cone = r = 2.1 cm


Volume of cone = 1/3 πr2h


= 1/3 × 22/7 × 2.1 × 2.1 × 4 cm3


= 18.48 cm3


Volume of solid = Volume of hemisphere + Volume of cone


= 19.404 cm3 + 18.48 cm3 = 37.884 cm3


The solid is placed in a cylindrical tub full of water in such a way that the whole solid is submerged in water, so, to find the volume of water left in the tub we need to subtract volume of solid from cylindrical tub.


Radius of cylinder = r’ = 5 cm


Height of cylinder = h’ = 9.8 cm


Volume of cylindrical tub = πr’2h’ = 22/7 × 5 × 5 × 9.8 cm3


= 770 cm3


Volume of water left in the tub = Volume of cylindrical tub – Volume of solid


Volume of water left in the tub = 770 cm3 – 37.884 cm3 = 732.116 cm3


∴ Volume of water left in the tub is 732.116 cm3



Question 22.

From a solid cylinder whose height is 8 cm and radius 6 cm, a conical cavity of height 8 cm and of base radius 6 cm is hollowed out. Find the volume of the remaining solid. Also, find the total surface area of the remaining solid. [Take π = 3.14.]


Answer:

Height of solid cylinder = h = 8 cm


Radius of solid cylinder = r = 6 cm


Volume of solid cylinder = πr2h


= 3.14 × 6 × 6 × 8 cm3


= 904.32 cm3


Curved Surface area of solid cylinder = 2πrh


Height of conical cavity = h = 8 cm


Radius conical cavity = r = 6 cm


Volume of conical cavity = 1/3 πr2h


= 1/3 × 3.14 × 6 × 6 × 8 cm3


= 301.44 cm3


Let l be the slant height of conical cavity


l2 = r2 + h2


⇒ l2 = (62 + 82) cm2


⇒ l2 = (36 + 64) cm2


⇒ l2 = 100 cm2


⇒ l = 10 cm


Curved Surface area of conical cavity = πrl


Since, conical cavity is hollowed out from solid cylinder, so, volume and total surface area of remaining solid will be found out by subtracting volume and total surface area of conical cavity from volume and total surface area of solid cylinder.


Volume of remaining solid = Volume of solid cylinder – Volume of conical cavity


Volume of remaining solid = 904.32 cm3 – 301.44 cm3


= 602.88 cm3


Total surface area of remaining solid = Curved Surface area of solid cylinder + Curved Surface area of conical cavity + Area of circular base


Total surface area of remaining solid = 2πrh + πrl + πr2


= πr × (2h + l + r)


= 3.14 × 6 × (2 × 8 + 10 + 6) cm2


= 3.14 × 6 × 32 cm2


= 602.88 cm2



Question 23.

From a solid cylinder of height 2.8 cm and diameter 4.2 cm, a conical cavity of the same height and same diameter is hollowed out. Find the total surface area of the remaining solid.


Answer:

Height of solid cylinder = h = 2.8 cm


Diameter of solid cylinder = 4.2 cm


Radius of solid cylinder = r = Diameter ÷ 2 = 2.1 cm


Curved Surface area of solid cylinder = 2πrh


= 2 × 22/7 × 2.1 × 2.8 cm2


= 2 × 22/7 × 2.1 × 2.8 cm2


= 36.96 cm2


Height of conical cavity = h = 2.8 cm


Radius conical cavity = r = 2.1 cm


Let l be the slant height of conical cavity


l2 = r2 + h2


⇒ l2 = (2.82 + 2.12) cm2


⇒ l2 = (7.84 + 4.41) cm2


⇒ l2 = 12.25 cm2


⇒ l = 3.5 cm


Curved Surface area of conical cavity = πrl


= 22/7 × 2.1 × 3.5


= 23.1 cm2


Total surface area of remaining solid = Curved surface area of solid cylinder + Curved surface area of conical cavity + Area of circular base


Total surface area of remaining solid = (36.96 + 23.1 + 22/7 × 2.12) cm2


= (36.96 + 23.1 + 13.86) cm2


= 73.92 cm2



Question 24.

From a solid cylinder of height 14 cm and base diameter 7 cm, two equal conical holes each of radius 2.1 cm and height 4 cm are cut off. Find the volume of the remaining solid.


Answer:

Height of solid cylinder = h = 14 cm


Diameter of solid cylinder = 7 cm


Radius of solid cylinder = r = Diameter ÷ 2 = 7/2 cm = 3.5 cm


Volume of solid cylinder = πr2h


= 22/7 × 3.5 × 3.5 × 14 cm3


= 539 cm3


Height of conical cavity = h’ = 4 cm


Radius conical cavity = r’ = 2.1 cm


Volume of conical cavity = 1/3 πr’2h’


= 1/3 × 22/7 × 2.1 × 2.1 × 4 cm3


= 18.48 cm3


Since, there are two conical cavities


∴ Volume of two conical cavities = 2 × 18.48 cm3 = 36.96 cm3


Volume of remaining solid = Volume of solid cylinder – Volume of two conical cavity


Volume of remaining solid = 539 cm3 – 36.96 cm3


= 502.04 cm3



Question 25.

A metallic cylinder has radius 3 cm and height 5 cm. To redue its weight, a conical hole is drilled in the cylinder. The conical hole has a radius of 3/2 cm and its depth is 8/9 cm. Calculate the ratio of the volume of metal left in the cylinder to the volume of metal taken out in conical shape.


Answer:

Height of metallic cylinder = h = 5 cm

Radius of metallic cylinder = r = 3 cm

Volume of solid cylinder = πr2h

= π × 3 × 3 × 5 cm3

= 45π cm3


Height of conical hole = h’ = 8/9 cm


Radius conical hole = r’ = 3/2 cm


Volume of conical hole = 1/3 πr’2h’


= 1/3 × π × 3/2 × 3/2 × 8/9 cm3


= 2/3 π cm3


Volume of metal left in cylinder = Volume of metallic cylinder – Volume of conical hole


Volume of metal left in cylinder = 45π - 2/3 π = 133π/3


Ratio of the volume of metal left in the cylinder to the volume of metal taken out in conical shape = Volume of metal left in cylinder/ Volume of conical hole


Volume of metal left in cylinder : Volume of conical hole = 133π/3 : 2/3 π


Volume of metal left in cylinder: Volume of conical hole = 133: 2


Ratio of the volume of metal left in the cylinder to the volume of metal taken out in conical shape is 339:4


Question 26.

A spherical glass vessel has a cylindrical neck 7 cm long and 4 cm in diameter. The diameter of the spherical part is 21cm. Find the quantity of water it can hold. [Use π = 22/7]


Answer:

Length of cylindrical neck = l = 7 cm


Diameter of cylindrical neck = 4 cm


Radius of cylindrical neck = r = Diameter ÷ 2 = 4/2 cm = 2 cm


Volume of cylindrical neck = πr2l


= 22/7 × 2 × 2 × 7 cm3


= 88 cm3


Diameter of spherical part = 21 cm


Radius of spherical part = r’ = Diameter ÷ 2 = 21/2 cm = 10.5 cm


Volume of spherical part = 4/3 πr’3


= 4/3 × 22/7 × 10.5 × 10.5 × 10.5 cm3


= 4851 cm3


Quantity of water spherical glass vessel with cylindrical neck can hold = Volume of spherical part + Volume of cylindrical neck


Quantity of water spherical glass vessel with cylindrical neck can hold = 4851 cm3 + 88 cm3 = 4939 cm3


Quantity of water spherical glass vessel with cylindrical neck can hold is 4939 cm3.



Question 27.

The given figure represent a solid consisting of a cylinder surmounted by a cone at one end a hemisphere at the other. Find the volume of the solid.



Answer:

The solid consisting of a cylinder surmounted by a cone at one end a hemisphere at the other.


Length of cylinder = l = 6.5 cm


Diameter of cylinder = 7 cm


Radius of cylinder = r = Diameter ÷ 2 = 7/2 cm = 3.5 cm


Volume of cylinder = πr2l


= 22/7 × 3.5 × 3.5 × 6.5 cm3


= 250.25 cm3


Length of cone = l’ = 12.8 cm – 6.5 cm = 6.3 cm


Diameter of cone = 7 cm


Radius of cone = r = Diameter ÷ 2 = 7/2 cm = 3.5 cm


Volume of cone = 1/3 πr2l’


= 1/3 × 22/7 × 3.5 × 3.5 × 6.3 cm3


= 80.85 cm3


Diameter of hemisphere = 7 cm


Radius of hemisphere = r = Diameter ÷ 2 = 7/2 cm = 3.5 cm


Volume of hemisphere = 2/3 πr3


= 2/3 × 22/7 × 3.5 × 3.5 × 3.5 cm3


= 89.83 cm3


Volume of the solid = Volume of cylinder + Volume of cone + Volume of hemisphere


Volume of solid = 250.25 cm3 + 80.85 cm3 + 89.83 cm3


= 420.93 cm3



Question 28.

From a cubical piece of wood of side 21 cm, a hemisphere is carved out in such a way that the diameter of the hemisphere is equal to the side of the cubical piece. Find the surface area and volume of the remaining piece.


Answer:

Length of cubical piece of wood = a = 21 cm


Volume of cubical piece of wood = a3


= 21 × 21 × 21 cm3


= 9261 cm3


Surface area of cubical piece of wood = 6a2


= 6 × 21 × 21 cm2


= 2646 cm2


Since, a hemisphere is carved out in such a way that the diameter of the hemisphere is equal to the side of the cubical piece.


So, diameter of hemisphere = length of side of the cubical piece


Diameter of hemisphere = 21 cm


Radius of hemisphere = r = Diameter ÷ 2 = 21/2 cm = 10.5 cm


Volume of hemisphere = 2/3 πr3


= 2/3 × 22/7 × 10.5 × 10.5 × 10.5 cm3


= 2425.5 cm3


Surface area of hemisphere = 2πr2


= 2 × 22/7 × 10.5 × 10.5 cm2


= 693 cm2


A hemisphere is carved out from cubical piece of wood


Volume of remaining solid = Volume of cubical piece of wood – Volume of hemisphere


Volume of remaining solid = 9261cm3 – 2425.5 cm3 = 6835.5 cm3


Surface area remaining piece of solid = surface area of cubical piece of wood – Area of circular base of hemisphere + Curved Surface area of hemisphere


Surface area remaining piece of solid = 6a2 – πr2 + 2πr2


= (2646 – 22/7 × 10.52 + 693) cm2


= 2992.5 cm2



Question 29.

A cubical block of side 10 cm is surmounted by a hemisphere. What is the largest diameter that the hemisphere can have? Find the cost of painting the total surface area of the solid so formed, at the rate of Rs 5 per 100 sq cm. [Use π = 3.14.]


Answer:

Length of side of cubical block = a = 10 cm


Since, a cubical block is surmounted by a hemisphere, so, the largest diameter of hemisphere = 10 cm


Since, hemisphere will be touching the sides of cubical block.


Radius of hemisphere = r = Diameter ÷ 2 = 10/2 cm = 5 cm


Surface area of solid = Surface area of cube – Area of circular part of hemisphere + Curved surface area of hemisphere


Total Surface area of solid = 6a2 – πr2 + 2πr2 = 6a2 + πr2


= 6 × 10 × 10 cm2 + 3.14 × 5 × 5 cm2


= 678.5 cm2


Rate of painting = Rs 5/100 cm2


Cost of painting the total surface area of the solid so formed = Total Surface area of solid × Rate of painting


Cost of painting the total surface area of the solid = Rs 5/100 × 678.5


= Rs 33.925



Question 30.

A toy is in the shape of a right circular cylinder with a hemisphere on one end a cone on the other. The radius and height of the cylindrical part are 5 cm and 13 cm respectively. The radii of the hemispherical and the conical parts are the same as that of the cylindrical part. Find the surface area of the toy, if the total height of the toy is 30 cm.


Answer:

The toy is in the shape of a right circular cylinder surmounted by a cone at one end a hemisphere at the other.


Total height of toy = 30 cm


Height of cylinder = h = 13 cm


Radius of cylinder = r = 5 cm


Curved surface area of cylinder = 2πrh


= 2 × 22/7 × 5 × 13 cm2


Height of cone = h’ = Total height of toy – Height of cylinder – Radius of hemisphere


Height of cone = h’ = 30 cm – 13 cm – 5 cm = 12 cm


Radius of cone = r = Radius of cylinder


Radius of cone = r = 5 cm


Let the slant height of cone be l


l2 = h’2 + r2


⇒ l2 = 122 + 52 cm2 = 144 + 25 cm2 = 169 cm2


⇒ l = 13 cm


Curved surface area of cone = πrl


= 22/7 × 5 × 13 cm2


Radius of hemisphere = r = Radius of cylinder


Radius of hemisphere = r = 5 cm


Curved surface area of hemisphere = 2πr2


= 2 × 22/7 × 5 × 5 cm2


Surface area of the toy = Surface area of cylinder + Surface area of cone + Surface area of hemisphere


Surface area of toy = 2πrh + πrl + 2πr2


= πr (2h + l + 2r)


= 22/7 × 5 × (2 × 13 + 13 + 2 × 5) cm2


= 22/7 × 5 × 49 cm2


= 770 cm2


Surface area of toy is 770 cm2



Question 31.

The inner diameter of a glass is 7 cm and it has a raised portion in the bottom in the shape of a hemisphere, as shown in the figure. If the height of the glass is 16 cm, find the apparent capacity and the actual capacity of the glass.



Answer:

Inner diameter of a glass = 7 cm


Inner radius of glass = r = 7/2 cm = 3.5 cm


Height of glass = h = 16 cm


Apparent capacity of glass = πr2h


= 22/7 × 3.5 × 3.5 × 16 cm3


= 616 cm3


Volume of the hemisphere in the bottom = 2/3 πr3


= 2/3 × 22/7 × 3.53 cm3


= 89.83 cm3


Actual capacity of the class = Apparent capacity of glass – Volume of the hemisphere


Actual capacity of the class = 616 cm3 – 89.83 cm3 = 526.17 cm3



Question 32.

A wooden toy is in the shape of a cone mounted on a cylinder, as shown in the figure. The total height of the toy is 26 cm, while the height of the conical part is 6 cm. The diameter of the base of the conical part is 5 cm and that of the cylindrical part in 4 cm. The conical part and the cylindrical part are respectively painted red and white. Find the area to be painted by each of them colours. [Take π = 22/7.]



Answer:

The wooden toy is in the shape of a cone mounted on a cylinder


Total height of the toy = 26 cm


Height of conical part = H = 6 cm


Height of cylindrical part = Total height of the toy – Height of conical part


h = 26 cm – 6 cm = 20 cm


Diameter of conical part = 5 cm


Radius of conical part = R = Diameter/2 = 5/2 cm = 2.5 cm


Let L be the slant height of the cone


L2 = H2 + R2


⇒ L2 = 62 + 2.52 cm2 = 36 + 6.25 cm2 = 42.25 cm2


⇒ L = 6.5 cm


Diameter of cylindrical part = 4 cm


Radius of cylindrical part = r = Diameter/2 = 4/2 cm = 2 cm


Area to be painted Red = Curved Surface area of cone + Base area of cone – base area of cylinder


Area to be painted Red = πRL + πR2 – πr2 = π (RL + R2 – r2)


= 22/7 × (2.5 × 6.5 + 2.5 × 2.5 – 2 × 2) cm2


= 22/7 × (16.25 + 6.25 – 4) cm2


= 22/7 × 18.5 cm2


= 58.143 cm2


Area to be painted White = Curved Surface area of cylinder + Base area of cylinder


Area to be painted White = 2πrh + πr2 = πr (2h + r)


= 22/7 × 2 × (2 × 20 + 2) cm2


= 22/7 × 2 × (40 + 2) cm2


= 22/7 × 2 × 42 cm2 = 264 cm2


∴ Area to be painted red is 58.143 cm2 and area to be painted white is 264 cm2.




Exercise 19b
Question 1.

The dimensions of a metallic cuboid are 100 cm × 80 cm × 64 cm. It is melted and recast into a cube. Find the surface area of the cube.


Answer:

Given,


The dimensions of a metallic cuboid = 100 cm × 80 cm × 64 cm


Let’s find out the volume of the cuboid first;


Volume of the cuboid = l × b × h


⇒ 100 × 80 × 64 = 512000cm3


As cuboid is recast into a cube;


So,


Volume of cube = volume of cuboid


⇒ l3 = 512000 ⇒ l =


⇒ l = 80 cm


Now,


As the length of the side of the cube = 80 cm


The surface area of the cube = 6(l)2


= 6 x 80 x 80


= 38400 cm2


So, the surface area of the cube is 38400 cm2



Question 2.

A cone of height 20 cm and radius of base 5 cm is made up of modelling clay. A child reshapes it in the form of a sphere. Find the diameter of the sphere.


Answer:

We have,


The radius of the cone (r) = 5cm and


The height of the cone (h) = 20cm


Let the radius of the sphere be R;


As,


Volume of sphere = Volume of cone


πR3 = πr2h


⇒ 4R3 = 5 × 5 × 20


⇒ R3 = = 125 cm


⇒ R = 5 cm


Diameter of the sphere = 2R = 2 x 5 = 10 cm


So, the diameter of the sphere is 10 cm



Question 3.

Metallic spheres of radii 6 cm, 8 cm and 10 cm respectively are melted to form a single solid sphere. Find the radius of the resulting sphere.


Answer:

Given,


The radius (r1) of 1st sphere = 6 cm


Radius of 2nd sphere (r2) = 8 cm and


Radius f third sphere (r3) = 10 cm


Let the radius of the resulting sphere be R;


So now we have,


Volume of resulting sphere = Volume of three metallic spheres


πR3 =


πR3 =


⇒ R3 = 216 + 512 + 1000


⇒ R3 = 1728


⇒R =


R = 12 cm


So, the radius of the resulting sphere is 12cm.



Question 4.

A solid metal cone with radius of base 12 cm and height 24 cm is melted to form solid spherical balls of diameter 6 cm each. Find the number of balls thus formed.


Answer:

Let the number of balls formed = n


Since the metal cone is melted to form spherical balls;


So we have;


Volume of metal cone = Total volume of n spherical balls


Volume of cone = n(Volume of 1 spherical ball)







n = 32


So, 32 spherical balls can be formed.



Question 5.

The radii of internal and external surface of a hollow spherical shell are 3 cm and 5 cm respectively. It is melted and recast into a solid cylinder of diameter 14 cm. Find the height of the cylinder.


Answer:

We have,


The internal base radius of spherical shell, r1 = 3 cm,


The external base radius of spherical shell, r2 = 5 cm and


The base radius of solid cylinder, r = 7 cm


Let the height of the cylinder be h


As,


The hollow spherical shell is melted into a solid cylinder;


So,


Volume of solid cylinder = Volume of spherical shell


⇒ πr2h =


⇒ πr2h =


⇒ r2h =


⇒ 49 × h = (125−27)


⇒ h = × 9849


Therefore,


h =


So, the height of the cylinder is .



Question 6.

The internal and external diameters of a hollow hemispherical shell are 6 cm and 10 cm respectively. It is melted and recast into a solid cone of base diameter 14 cm. Find the height of the cone so formed.


Answer:

Given,


Internal diameter of the hemisphere = 6 cm


External diameter of the hemisphere = 10 cm


Diameter of cone = 14 cm


So we have,


Internal radius(r) of the hemisphere = 3 cm


External radius(R) of the hemisphere = 5 cm


Radius of cone = 7 cm


Now,


Volume of the hollow hemisphere = Volume of the cone
Volume of the hollow hemisphere =


Volume of the cone =


So,





= 2 × 98 = 49 × h


= h


So,


h = = 4 cm


Thus, the height of the cone formed is 4 cm.



Question 7.

A copper rode of diameter 2 cm and length 10 cm is drown into a wire of uniform thickness and length 10 m. Find the thickness of the wire.


Answer:

Given,


Diameter of the copper Rod = 2cm


Length of the Rod = 10 cm


Length of the wire = 10 m = 1000cm


So here we have,


Radius of the copper rod = 1 cm


Let suppose the radius of the wire = r


Volume of the rod = volume of the wire


πr2h = πr2h


1 × 1 × 10 = 1000 × r


10 = 1000r


r = = 0.1 cm


The diameter of the wire = 2r = 2 × 0.1 = 0.2 cm


So, the thickness of the wire is 0.2 cm or 2mm.



Question 8.

A hemispherical bowl of internal diameter 30 cm contains some liquid. This liquid is to be filled into cylindrical- shaped bottles each of diameter 5 cm and height 6 cm. Find the number of bottles necessary to empty the bowl.


Answer:

Let the required bottles = n


Internal diameter of the hemispherical sphere = 30 cm


Internal Radius of the hemispherical sphere = 15 cm


Diameter of the cylindrical bottle = 5 cm


Radius of the cylindrical bottle = 2.5 cm


Now,


Volume of the hemispherical sphere = πr3


= π × 15 × 15 × 15


= π10 × 15 × 15


= 2,250π cm3


And,


Volume of 1 cylindrical bottle = πr2h = π × 2.5 × 2.5 × 6 = 37.5π cm3


Amount of water in n bottles = Amount of water in bowl


n × 37.5π = 2,250π


n = = 60 bottles


So, 60 numbers of bottles are required to empty the bowl.



Question 9.

A solid metallic sphere of diameter 21 cm is melted and recast into a number of smaller cones, each of diameter 3.5 cm and height 3 cm. Find the number of cones so formed.


Answer:

Given,


Diameter of the sphere = 21 cm


Diameter of the cone = 3.5 cm = cm


Height of the cone = 3 cm


So here we have,


Radius of the sphere = 10.5 cm


Radius of the cone = 1.75 cm


Volume of the sphere = πr3


=


Volume of the cone =


No of cones =





= 504


So,


504 numbers of cones are formed.



Question 10.

A spherical cannon ball 28 cm in diameter is melted and recast into a right circular conical mould, base of which is 35 cm in diameter. Find the height of the cone.


Answer:

Given,


Diameter of cannon ball = 28cm


So the Radius of cannon ball = 14 cm


Volume of cannon ball = πr3 = π × 143


Radius of the cone = cm


Let the height of cone be h cm.


Volume of cone = π × h cm3


Here we have,


= π × 143 = π × h


= h = π × 14 × 14 × 14 ×


= h = 35.84 cm


Hence, the height of the cone = 35.84 cm.



Question 11.

A spherical ball of radius 3 cm is melted and recast into three spherical balls. The radii of two of these balls are 1.5 cm and 2 cm. Find the radius of the third ball.


Answer:

Given,


Radius of the spherical ball = 3 cm


After recasting the ball into three spherical balls;


Radius of the first ball = r1 = 1.5 cm


Radius of the first ball = r2 = 2 cm


Let the radius of the third ball be r3 cm.


Then,


Volume of third ball = Volume of spherical ball – volume of 2 small balls


Volume of the third ball = π × 33π π × 23


36π – – 32π/3cm3 = 125π/6cm3


4/3πr3 = 125π/6


r3 = 125π × 3/6 × 4 × π = 125/8


r = = 2.5 cm


Hence, the radius of the third ball is 2.5cm.



Question 12.

A spherical shell of lead whose external and internal diameter are respectively 24 cm and 18 cm, is melted and recast into a right circular cylinder 37 cm high. Find the diameter of the base of the cylinder.


Answer:

External diameter of shell = 24cm and


Internal diameter of shell = 18cm


So,


External radius of shell = 12 cm and


Internal radius = 9 cm


Volume of lead in the shell = π[123–93]cm3


Let the radius of the cylinder be r cm


Height of the cylinder = 37 cm


Volume of the cylinder = πr2h = πr2(37)


π[123–93] = πr2 × 37


π × 999 = πr2 × 37


r2 = × π × 999 × π = 36cm2


r = = 6cm


Hence, diameter of the base of the cylinder = 12 cm



Question 13.

A hemisphere of lead of radius 9 cm is into a right circular cone of height 72cm. Find the radius of the base of the cone.


Answer:

Volume of hemisphere of radius 9 cm


= × π × 9 × 9 × 9cm3


Volume of circular cone (height = 72 cm)


× π × r2 × 72cm


Volume of cone = volume of hemisphere


× π × r2 × 72 = π × 9 × 9 × 9


r2 = π × 9 × 9 × 9 × π = 20.25


r = = 4.5cm


Hence, radius of the base of the cone = 4.5 cm



Question 14.

A spherical ball of diameter 21 cm is melted and recast into cubes, each of side 1 cm. Find the number of cubes so formed


Answer:

Diameter of sphere = 21 cm


Hence, radius of the sphere =


Volume of sphere = π × r3 =


Volume of cube = a3 = 13


Let the number of cubes formed be n


Volume of sphere = n(volume of cube)


= n × 1


4 × 22 × = n × 1


11 × 21 × 21 = n × 1


4851 = n


So,


Hence, the number of cubes is 4851



Question 15.

How many lead balls, each of radius 1 cm, can be made from a sphere of radius 8 cm?


Answer:

Given,


Radius of the sphere = 8 cm


Radius of the ball we made = 1 cm


So,


Volume of sphere (when r = 1 cm) = πr3


= × 1 × 1 × 1 × π cm3


Volume of sphere (when r = 8 cm) = πr3


= × 8 × 8 × 8 × π cm3


Let the number of balls be n;


Then we have;


n × × 1 × 1 × 1 × π = × 8 × 8 × 8 × π


n = = 512


Hence, the number of lead balls can be made is 512.



Question 16.

A solid sphere of radius 3 cm is melted and them cast into small spherical balls, each of diameter 0.6 cm. Find the number of small balls so obtained.


Answer:

Given,


Radius of sphere = 3 cm


Volume of sphere = πr3 = × π × 3 × 3 × 3cm3 = 36π cm3


Radius of small sphere = cm = 0.3 cm


Volume of small sphere = × π × 0.3 × 0.3 × 0.3cm3


= cm3


Let the number of small balls be n;


n × = × π × 3 × 3 × 3


n = 1000


Hence, the number of small balls = 1000



Question 17.

The diameter of a sphere is 42 cm. It is melted and drawn into a cylindrical wire of diameter 2.8 cm. Find the length of wire.


Answer:

Given,


Diameter of sphere = 42 cm


Radius of sphere = 21 cm


Volume of sphere = πr3 = × π × 21 × 21 × 21 cm3


Diameter of cylindrical wire = 2.8 cm


Radius of cylindrical wire = 1.4 cm


Volume of cylindrical wire = πr2h = π × 1.4 × 1.4 × h cm3 = 1.96πh cm3


Volume of cylindrical wire = volume of sphere


1.96πh = × π × 21 × 21 × 21


h = cm


h = 6300


h m = 63 m


Hence, length of the wire = 63 m



Question 18.

The diameter of a copper sphere is 18 cm. It is melted and drown into a long wire of uniform cross section. If the length of the wire is 108 m, find its diameter.


Answer:

Given,


Diameter of sphere = 18 cm


Length of wire = 108 m = 10800 cm


Radius of copper sphere = m = 36 m


Volume of sphere = πr3 = × π × 9 × 9 × 9cm3 = 972π cm3


Let the radius of wire be r cm


= πr2l = πr2 × 10800


But the volume of wire = volume of sphere


⇒ πr2 × 10800 = 972π


r2 = = 0.09 cm2


r = cm = 0.3


Hence, the diameter = 2r = 0.6 cm



Question 19.

A hemispherical bowl of internal radius 9 cm is full of water. Its contents are emptied into a cylindrical vessel of internal radius 6 cm. Find the height of water in the cylindrical vessel.


Answer:

Given,


Internal radius of hemispherical bowl (r1) = 9 cm


Internal radius of cyclical vessel (r2) = 6 cm


Let the height of water in the cylindrical vessel be h.


So,


Volume of hemispherical bowl = volume of cylindrical vessel






h = 13.5 cm


Hence, height of water in the cylindrical vessel is 13.5 cm



Question 20.

A hemispherical tank, full of water, is emptied by a pipe at the rate of 25/7 litres per second. How much time will it take to empty half the tank if the diameter of the base of the tank is 3 m?


Answer:

Given,


Diameter of the hemispherical tank = 3 m


Radius of hemispherical tank = m


Volume of tank = m3


= … (1m3 = 1000L)


Half the tank = L


Now,


In 1 sec = L of water is emptied


Required time


=



= 990 sec


So,



= 16.5 min

To empty half the tank 16.5 min are required.


Question 21.

The rain water from a roof of 44 m × 20 m drains into a cylindrical tank having diameter of base 4 m and height 3.5 m. If the tank is just full, find the rainfall in cm.


Answer:

Length of the roof = 44 m


Breadth of the roof = 20 m


Diameter of the cylindrical tank = 4 m


Radius of the cylindrical tank = 2 m


Height of the cylindrical tank = 3.5 m


Volume of roof = l x b x h
= 44X20 X h
volume of cylinder = x2x2x


Height of roof = height of rainfall
And
Volume of water on roof = Volume of Water in cylindrical tank
44X 20 X h = × 2 x 2 x
44 X 20 X h = 22 × 2 × 2 ×


880 × h = 44
h =
h = .05 m
h = 5 cm



Question 22.

The rain water from a 22 m × 20 m roof drain into a cylindrical vessel of diameter 2 m and height 3.5 m. If the rain water collected from the roof fills 4/5th of the cylindrical vessel then find the rainfall in centimetre.


Answer:

Given,


Length of the roof = 22 m


Breadth of the roof = 20 m


Diameter of the cylindrical vessel = 2 m


Radius of the cylindrical vessel = 1 m


Height of the cylindrical vessel = 3.5 m


Volume of the rainfall = area of the roof × depth of the rainfall


πr2h = area of the roof × d


× 3.14 × 1 × 3.5 = 22 × 20 × d


× 11 = 440d


8.8 = 440d


0.02 = d


d = 0.02 m


So,


The rainfall is 2 cm.



Question 23.

A solid right circular cone of height 60 cm and radius 30 cm is dropped in a right circular cylinder full of water, of height 180 cm and radius 60 cm. Find the volume of water left in the cylinder, in cubic metres.


Answer:

Height of a cone = 60 cm,


Radius of a cone = 30 cm
volume of cone =
Height of cylinder = 180 cm,


Radius of cylinder = 60 cm
Volume of cylinder = πr2h
Volume of the remaining water = volume of cylinder - volume of cone


= π × 60 × 60 × 180 – × π × 30 × 30 × 60


= 648000π – 1800π (Taking 1800 as common)


= 18000(36π-π)


= 18000 x 35 x = 1.98 m3



Question 24.

Water is flowing through a cylindrical pipe of internal diameters 2 cm, into a cylindrical tank of base radius 40 cm, at the rate of 0.4 m per second. Determine the rise in level of water in tank in half an hour.


Answer:

Given,


Diameter of the cylindrical pipe = 2 cm
Radius of the cylindrical pipe = 1 m
Height of the cylindrical pipe = 0.4m/s = 40 cm/s

Water flown in 1 sec = 40 cm
Water flown in 30 minutes (30 × 60 seconds)
= 40 × 60 × 30 m = 72000 cm

Radius of cylindrical tank = 40 cm
Let Height be h,
As we can see the volume of water which passes through the cylindrical pipe is equal to the volume of water present in the cylindrical tank after half an hour.
So,

Volume of water which passes through the cylindrical = volume of water present in the cylindrical tank after half an hour
πr2h = πr2h
(1)2 × 72000 = (40)2 × h
72000 = 1600 × h
h = 45 cm


Hence,

The rise in level of water in tank in half an hour will be 45 cm.


Question 25.

Water is flowing at the rate of 6 km/hr through a pipe of diameter 14 cm into a rectangular tank which is 60 m long and 22 m wide. Determine the time in which the level of water in the tank will rise by 7 cm.


Answer:

Given,


Speed of the water flowing through the pipe, H = 6 km/hr


= cm/s


Diameter of the pipe = 14 cm


Radius (R) of pipe = = 7 cm


Length (l) of the rectangular tank = 60 m = 6000 cm,


Breadth (b) of the rectangular tank = 22 m = 2200 cm and


Height (h) or Rise in the level of water in the tank = 7 cm


Now,


Volume of the water in rectangular tank = l × b × h = 6000 x 2200 x 7 = 92400000cm3


Also,


Volume of the water flowing through the pipe in 1 s = πR2H


= cm3


So,


The time taken to rise the water =


cm3


= 92400 x = 3600s = 1 hr


So,


In 1 hour time the level of water in the tank will rise by 7cm.



Question 26.

Water in a canal, 6 m wide and 1.5 m deep, is flowing at a speed of 4 km/hr. How much area will it irrigate in 10 minutes if 8 cm of standing water is needed for irrigation?


Answer:

Given,


Width of the canal = 6 m


Depth of the canal, = 1.5 m


Length of the cuboid = 666.67 m


Water is flowing at a speed of 4 km/hr = 4000m/hr


Thus,


Area irrigated in 10 minutes = hour = x 4000 = 666.67 m


Hence,


Volume of the water flowing in hour = Volume of the canal


⇒ Volume of the water flowing in hour = 666.67 x 6 x 1.5 = 6000.03 = 60000 m3


Let a m2 is the area irrigated in hour,


Then,


⇒ a × = 6000


⇒ a = m2 = 75000 m2



Question 27.

A farmer connects a pipe of internal diameter 25 cm from a canal into a cylindrical tank in his field, which is 12 m in diameter and 2.5 m deep. If water flows through the pipe at the rate of 3.6km/hr, in how much time will the tank be filled? Also, find the cost of water if the canal department charges at the rate of Rs 0.07 per m3.


Answer:

Given,


Internal diameter of the pipe = 25 cm


Radius (r) of the pipe =


Diameter of the cylindrical tank = 12 m


Radius (R) of the tank = 6 m


Height (h) of the tank = 2.5 m


In 1 hour water comes out of the pipe = 3.6 km = 3600 m


Let the total hrs = n


Volume of the water coming out of the pipe in n hrs = volume of the cylindrical tank



= 6 × 6 × 2.5


3600 n = 36 × 2.5 × 8 × 8


100 n = 8 × 8 × 2.5


1000 n = 8 × 8 × 25


n = = 1.6 hrs


n = 1 hrs 36 minutes


Now calculate the cost;


Cost of the water = Volume of the cylindrical tank × 0.07


= = 19.80 rs


So,


Total time required to fill the tank is 1 hr 36 minutes and the cost is 19.80 rs.



Question 28.

Water running in a cylindrical pipe of inner diameter 7 cm, is collected in a container at the rate of 192.5 litres per minute. Find the rate of flow of water in the pipe in km/hr.


Answer:

Given,


Diameter of the cylindrical pipe = 7 cm


Radius of the pipe = 3.5 cm


Volumetric flow rate = 192.5 l/min


Flow rate =


Area = pi × r2 = 3.14 × 3.5 × 3.5 = 38.5 cm2


Since 1 l = 1dm3 = 1000 cm3


Volumetric flow rate = 192.5 × 1000 cm3/min


So,


Flow rate = = 5000 cm/min


Flow rate in km/h = 5000 cm/min = 5000 × 0.00001 km/( h) = 3 km/h



Question 29.

150 spherical marbles, each of diameter 14 cm, are dropped in a cylindrical vessel of diameter 7 cm containing some water, which are completely immersed in water. Find the rise in the level of water in the vessel.


Answer:

Given,


Diameter of marble = 1.4 cm


Radius of marble = 0.7 cm


Number of marbles = 150


Diameter of cylinder = 7cm


Radius of cylinder = 3.5cm


150 × Volume of spherical marbles = volume of cylindrical vessel
⇒ 150 x × π × 0.7 × 0.7 × 0.7 = π × 3.5 × 3.5 × h


⇒ 50 x 4 x (0. 7)3 = (3.5)2 x h


⇒ 200 x 0.343 = 12.25 x h


= 5.6 = h


So,


h = 5.6 cm


The rise in the level of water in the vessel (h) = 5.6 cm



Question 30.

Marbles of diameters 1.4 cm are dropped into a cylindrical beaker of diameter 7 cm, containing some water. Find the number of marbles that should be dropped into beaker so that the water level rises by 5.6 cm.


Answer:

Given,


Diameter of marble = 1.4 cm


Diameter of cylinder = 7 cm


Radius of cylinder = 3.5 cm


Cylinder height = 5.6 cm


Radius of marble = 0.7 cm


Volume of 1 marble = πr3


= = 1.4373


So,


Let the number of marbles be n,


Increase in the Volume of cylinder due to n marbles = πr2h


= 3.14 × 3.5 × 3.5 × 5.6 = 215.6 cm3


Hence the total numbers of marble required = = 150 marbles



Question 31.

In a village, a well with 10 m inside diameter, is dug 14 m deep. Earth taken out of it is spread all around to a width of 5 m to form an embankment. Find the height of the embankment. What value of the villagers is reflected here?


Answer:

Given,


Diameter of the well = 10 m


Radius of the well = 5 m
Height of the well = 14 m
Width of the embankment = 5 m
Therefore radius of the embankment = 5 + 5 = 10 m
Let h be the height of the embankment,


Hence,


The volume of the embankment = volume of the well


π(R – r)2h = πr2h
(102 – 52) x h = 52 x 14
(100 – 25) x h = 25 x 14
h =
Therefore,


The height of the embankment, h = 4.67 m



Question 32.

In a corner of a rectangular field with dimensions 35 m × 22 m, a well with 14m inside diameters is dug 8 m deep. The earth dug out is spread evenly over the remaining part of the field. Find the rise in the level of the field.


Answer:

Given,


Diameter of the well = 14 m


Radius of the well = 7 m


Height of the well = 8 m


Now,


Volume of the earth dug out of the well = πr2h


= × 7 × 7 × 8 = 1,232 m3


Area on which earth dug out is spread = l × b - r2h


= 35 × 22 - × 7 × 7


= 770 - 154 = 616 m


Level of the earth raised = = 2 m


So, the rise in the level of the field is 2 m.



Question 33.

A copper wire of diameter 6 mm is evenly wrapped on a cylinder of length 18 cm and diameter 49 cm to cover its whole surface. Find the length and the volume of the wire. If the density of copper be 8.8 g per cu-cm, find the weight of the wire.


Answer:

Given,


Diameter of the copper wire = 6 mm = 0.6 cm


Radius of the copper wire = 0.3 cm


Length of the cylinder = 18 cm


Diameter of the cylinder = 49 cm


Radius of the cylinder = cm


Density of the copper = 8.8g


Number of the rotations on the cylinder = = 30 cm


Base circumference of the cylinder = 2πr = = 154 cm


So,


Length of the wire = 154 × 30 = 4620 cm


Volume of the wire = πr2h


= × 0.3 × 0.3 × 4620 = 1306.8


Weight of the wire = volume × density


= 1306.8 × 8.8 = 11,499.84 gm



Question 34.

A right triangle whose sides are 15 cm and 20 cm (Other than hypotenuse) , is made to revolve about its hypotenuse. Find the volume and surface area of the double cone so formed. (Choose value of π as found appropriate)


Answer:

Let name the triangle, ABC


Given,


Sides of the triangle are 15 and 20 cm,


BD ⟘ AC.


In the given case BD is the radius of the double cone generated by triangle by revolving.


Now by Pythagoras theorem,


AC2 = AB2 + BC2


AC2 = (15)2 + (20)2


AC2 = 225 + 400


AC2 = 625 = (25)2


AC = 25 cm


Let AD be m cm;


∴ CD = (25 – m) cm


By using the Pythagoras theorem in ∆ABD and ∆CBD;


∆ABD,


AD2 + BD2 = AB2


m2 + BD2 = 225


BD2 = 225 – m2 ……….(i)


∆CBD,


BD2 + CD2 = BC2


BD2 + (25 – m)2 = (20)2


BD2 = (20)2 – (25 – m)2 …..(ii)


By putting both the equations (i) and (ii) together,


225 – m2 = (20)2 – (625 – m)2


225 – m2 = 400 – (625 + m2 – 50m) ……….by (a2 – b2)


225 – m2 = - 225 – m2 + 50m


225 – m2 + 225 + m2 = 50m


450 = 50m


So,


m = = 9 cm


BD2 = (15)2 – (9)2


BD2 = 225 – 81 = 144 cm


BD = 12 cm


Radius of the generated double cone = 12 cm


Now,


Volume of the cone generated = Volume of the upper cone + Volume of the lower cone



(AD + CD)


(12)2× (25)


= 1200π cm3 = 3,771.42 cm3


Surface area of the double cone formed;


= L.S.A of upper cone + L.S.A of the lower cone


= π (BD) × (AB) + π (BD) × (BC)


= π × 12cm × 15 cm + π × 12 cm × 20 cm


= 420π cm2 = 1320 cm2


So, the volume is 1200π cm3 and surface area is 420π cm2, of the double cone so formed.



Exercise 19c
Question 1.

A drinking glass is in the shape of a frustum of a cone of height 14 cm. The diameter of its two circular ends are 16 cm and 12 cm. Find the capacity of the glass.


Answer:

Given: Height of glass = h = 14 cm


Diameter of lower circular end of glass = 12 cm


Diameter of upper circular end of glass = 16 cm


∴ Radius of lower circular end = r = 12/2 = 6 cm


∴ Radius of lower circular end = R = 16/2 = 8 cm




= 44 × 49.33 cm3


2170.52 cm3


∴ Capacity of glass = 2170.52 cm3



Question 2.

The radii of the circular ends of a solid frustum of a cone are 18 cm and 12 cm and its height is 8 cm. Find its total surface area. [Use π = 3.14.]


Answer:

Given: Radius of lower circular end = r = 12 cm


Radius of upper circular end = R = 18 cm


Height of frustum = h = 8 cm


Formula: Total surface area of frustum = πr2 + πR2 + π(R + r)l cm2


Where l = slant height


For slant height we have



∴ l = 10 cm


∴ total surface area of frustum = π × 122 + π × 182 + π × (18 + 12) × 10 cm2


= 3.14 × (144 + 324 + 300) cm2


= 3.14 × 768 cm2


= 2411.52 cm2


∴ total surface area of frustum = 2411.52 cm2



Question 3.

A metallic bucket, open at the top, of height 24 cm is in the form of the frustum of a cone, the radii of whose lower and upper circular ends are 7 cm and 14 cm respectively. Find

(i) the volume of water which can completely fill the bucket;

(ii) the area of the metal sheet used to make the bucket.


Answer:

Given: Height of bucket = h = 24 cm


Radius of lower circular end = r = 7 cm


Radius of upper circular end = R = 14 cm



Total surface area of frustum = πr2 + πR2 + π(R + r)l cm2


Where l = slant height




∴ l = 25 cm


(i) volume of water which will completely fill the bucket = volume of frustum



= 8 × 22 × 49


= 8722 cm3


∴ volume of water which will completely fill the bucket = 8722 cm3


(ii) area of metal sheet used


Since the top is open we need to subtract the area of top/upper circle from total surface area of frustum because we don’t require a metal plate for top.


Radius of top/upper circle = R


Area of upper circle = πR2


∴ area of metal sheet used = (total surface area of frustum)-πR2


∴ Area of metal sheet used = πr2 + πR2 + π(R + r)l-πR2cm2


= πr2 + π(R + r)l cm2



= 22 × 82 cm2


= 1804 cm2


∴ Area of metal sheet used to make bucket = 1804 cm2



Question 4.

A container, open the top, is in the form of a frustum of a cone of height 24 cm with radii of its lower and upper circular ends as 8 cm and 20 cm respectively. Find the cost of milk which can completely fill the container at the rate of Rs 24 per litre.


Answer:

Given: height of frustum container = h = 24 cm


Radius of lower circular end = r = 8 cm


Radius of upper circular end = R = 20 cm


Cost of 1 litre milk = 24 Rs



Volume of milk completely fill the container = volume of frustum of cone



= 8 × 3.14 × (400 + 64 + 160) cm3


= 8 × 3.14 × 624 cm3


= 15674.88 cm3


Now, 1 litre is 1000 cm3


∴ 15674.88 cm3 = 15674.88/1000 = 15.67488 litres


∴ Cost of milk which can completely fill the container = 15.67488 × cost of 1 litre milk


= 15.67488 × 24 Rs


= 376.19712 Rs


∴ Cost of milk which can completely fill the container = 376.19712 Rs



Question 5.

A container, open at the top and made up of metal sheet, is in the form of a frustum of a cone of height 16 cm with diameters of its lower and upper ends as 16 cm and 14 cm respectively. Find the cost of metal sheet used to make the container, if it costs Rs 10 per 100 cm2.


Answer:

Given: height of container frustum = h = 16 cm


Diameter of lower circular end = 16 cm


Diameter of upper circular end = 14 cm


∴ Radius of lower circular end = r = 16/2 = 8 cm


∴ Radius of upper circular end = R = 14/2 = 7 cm


Cost of 100 cm2 metal sheet = 10 Rs


∴ Cost of 1 cm2 metal sheet = 10/100 = 0.1 Rs


Formula: Total surface area of frustum = πr2 + πR2 + π(R + r)l cm2


Where l = slant height




∴ l = 16.0312 cm


Since the top is open we need to subtract the area of top/upper circle from total surface area of frustum because we don’t require a metal plate for top.


Radius of top/upper circle = R


Area of upper circle = πR2


∴ area of metal sheet used = (total surface area of frustum)-πR2


= πr2 + πR2 + π(R + r)l- πR2 cm2


= πr2 + π(R + r)l cm2


= π × (82 + (7 + 8)16.0312) cm2


= 3.14 × 304.468 cm2


= 956.029 cm2


∴ 956.029 cm2 metal sheet is required to make the container.


∴ Cost of 956.029 cm2 metal sheet = 956.029 × cost of 1 cm2 metal sheet


= 956.029 × 0.1 Rs


= 95.6029 Rs


∴ Cost of metal sheet required to make container = 95.6029 Rs



Question 6.

The radii of the circular ends of a solid frustum of a cone are 33 cm and 27 cm, and its slant height is 10 cm. Find its capacity and total surface area. [Take π = 22/7.]


Answer:

Given: Radius of lower circular end = r = 27 cm


Radius of upper circular end = R = 33 cm


Slant height = l = 10cm



Total surface area of frustum = πr2 + πR2 + π(R + r)l cm2


Here h height of frustum is not given and we need h to find the volume of frustum therefore we must first calculate the value of h as follows



using formula for slant height and with the help of given data we get



Squaring both sides


∴ 100 = 36 + h2


∴ h2 = 64


∴ h = 8


As length cannot be negative


∴ h = 8 cm




= 22 × 8 × 129


= 22704 cm3


∴ capacity = volume of frustum = 22704 cm3


Total surface area of frustum = πr2 + πR2 + π(R + r)l cm2



= (22/7) × (1089 + 729 + 600)


= (22/7) × 2418


= 7599.428 cm2


∴ total surface area = 7599.428 cm2



Question 7.

A bucket is in the form of a frustum of a cone. Its depts. Is 15 cm and the diameters of the top and the bottom are 56 cm and 42 cm respectively. Find how many litres of water the bucket can hold. [Take π = 22/7.]


Answer:

Given: Depth of the bucket = height of frustum = h = 15 cm


Diameter of top of bucket = 56 cm


Diameter of bottom of bucket = 42 cm


∴ Radius of top = R = 56/2 = 28 cm


∴ Radius of bottom = r = 42/2 = 21 cm



Volume of water bucket can hold = volume of bucket which is in form of frustum



= (22/7) × 5 × (784 + 441 + 588) cm3


= (22/7) × 5 × 1813 cm3


= 22 × 5 × 259 cm3


= 28490 cm3


Now 1 litre = 1000 cm3


∴ 28490 cm3 = 28490/1000 litres


= 28.49 litres


∴ bucket can hold 28.49 litres of water



Question 8.

A bucket made up of a metal sheet is in the form of frustum of a cone of height 16 cm with radii of its lower and upper ends as 8 cm and 20 cm respectively. Find the cost of the bucket if the cost of metal sheet used is Rs 15 per 100 cm2. [Use π = 3.14.]


Answer:

Given: height of container frustum = h = 16 cm


Radius of lower circular end = r = 8 cm


Radius of upper circular end = R = 20 cm


Cost of 100 cm2 metal sheet = 15 Rs


∴ Cost of 1 cm2 metal sheet = 15/100 = 0.15 Rs


Formula: Total surface area of frustum = πr2 + πR2 + π(R + r)l cm2


Where l = slant height




∴ l = 20 cm


Since it is given that a bucket is to be made hence the top is open we need to subtract the area of top/upper circle from total surface area of frustum because we don’t require a metal plate for top.


Radius of top/upper circle = R


Area of upper circle = πR2


∴ area of metal sheet used = (total surface area of frustum)-πR2


= πr2 + πR2 + π(R + r)l- πR2 cm2


= πr2 + π(R + r)l cm2


= π × (82 + (20 + 8)20) cm2


= 3.14 × 624 cm2


= 1959.36 cm2


∴ 1959.36 cm2 metal sheet is required to make the container.


∴ Cost of 1959.36 cm2 metal sheet = 1959.36 × cost of 1 cm2 metal sheet


= 1959.36 × 0.15 Rs


= 293.904 Rs


∴ Cost of metal sheet required to make container = 293.904 Rs



Question 9.

A bucket made up of a metal sheet is in the form of frustum of a cone. Its depth is 24 cm and the diameters of the top and bottom are 30 cm and 10 cm respectively. Find the cost of milk which can completely fill the bucket at the rate of Rs 20 per litre and the cost of metal sheet used if it costs Rs 10 per 100 cm2. [Use π = 3.14.]


Answer:

Given: depth of bucket = height of bucket/frustum = h = 24 cm


Diameter of lower circular end = 10 cm


Diameter of upper circular end = 30 cm


∴ Radius of lower circular end = r = 10/2 = 5 cm


∴ Radius of lower circular end = R = 30/2 = 15 cm


Cost of 100 cm2 metal sheet = 10 Rs


∴ Cost of 1 cm2 metal sheet = 10/100 = 0.1 Rs


Cost of 1 litre milk = 20 Rs



Total surface area of frustum = πr2 + πR2 + π(R + r)l cm2


Where l = slant height




∴ l = 26 cm


Since the top is open we need to subtract the area of top/upper circle from total surface area of frustum because we don’t require a metal plate for top.


Radius of top/upper circle = R


Area of upper circle = πR2


∴ area of metal sheet used = (total surface area of frustum)-πR2


= πr2 + πR2 + π(R + r)l- πR2 cm2


= πr2 + π(R + r)l cm2


= π × (52 + (15 + 5)26) cm2


= 3.14 × 545 cm2


= 1711.3 cm2


∴ 1711.3 cm2 metal sheet is required to make the container.


∴ Cost of 1711.3 cm2 metal sheet = 1711.3 × cost of 1 cm2 metal sheet


= 1711.3 × 0.1 Rs


= 171.13 Rs


∴ Cost of metal sheet required to make container = 171.13 Rs


Now,


Volume of milk which can completely fill the bucket = volume of frustum



= (1/3) × 3.14 × 26 × 325 cm3


= 26533/3 cm3


= 8844.33 cm3


Now 1 litre = 1000 cm3


∴ 8844.33 cm3 = 8844.33/1000 litres


= 8.84433 litres


∴ Volume of milk which can completely fill the bucket = 8.84433 litres


∴ Cost of milk which can completely fill the bucket = volume of milk which can completely


fill the bucket × cost of 1 litre milk Rs


= 8.84433 × 20 Rs


= 176.8866 Rs


Cost of milk which can completely fill the bucket = 176.8866 Rs



Question 10.

A container in the shape of a frustum of a cone having diameters of its two circular faces as 35 cm and 30 cm and vertical height 14 cm, is completely filled with oil. If each cm3 of oil has mass 1.2g, then find the cost of oil in the container if it costs Rs 40 per kg.


Answer:

Given: height of container/frustum = h = 14 cm


Diameter of top of container = 35 cm


Diameter of bottom of container = 30 cm


∴ Radius of top = R = 35/2 = 17.5 cm


∴ Radius of bottom = r = 30/2 = 15 cm


1 cm3 of oil = 1.2g of oil


Cost of 1 kg oil = 40 Rs



Volume of oil in container = volume of container which is in form of frustum



= (22/3) × 2 × (306.25 + 225 + 262.5) cm3


= (22/3) × 2 × 793.75 cm3


= 34925/3 cm3


= 11641.667 cm3


∴ Volume of oil in container = 11641.667 cm3


∴ 11641.667 cm3 of oil = 11641.667 × 1.2 g


= 13970.0004 g


We know 1000 g = 1 kg


∴ 13970.0004 g = 13970.0004/1000 kg


= 13.970 kg


∴ Cost of 13.970 kg oil = 13.970 × cost of 1 kg oil Rs


= 13.970 × 40 Rs


= 558.8 Rs


∴ Cost of oil in container = 558.8 Rs



Question 11.

A bucket is in the form of a frustum of a cone and its can hold 28.49 litres of water. If the radii of its circular ends are 28 cm and 21 cm, find the height of the bucket.


Answer:

Given: volume of bucket = 28.49 litres


1 litre = 1000 cm3


∴ 28.49 litres = 28.49 × 1000 cm3


∴ Volume of bucket = 28490 cm3


Radius of upper circular end = R = 28 cm


Radius of lower circular end = r = 21 cm


Let ‘h’ be the height of the bucket



Volume of bucket = volume of frustum of cone



∴ 28490 × 21 = h × 22 × (784 + 441 + 588)


∴ h = 598290/39886 cm


∴ h = 15 cm


∴ Height of bucket = h = 15 cm



Question 12.

The radii of the circular ends of a bucket of height 15 cm are 14 cm and r cm (r < 14). If the volume of the bucket is 5390 cm3, find the value of r.


Answer:

Given: volume of bucket = 5390 cm3


Radius of upper circular end = R = 14 cm


Radius of lower circular end = r cm & r is less than 14


Height of bucket = h = 15



Volume of bucket = volume of frustum of cone



∴ 5390 × 7 = 22 × 5 × (196 + r2 + 14r)


∴ 37730/110 = 196 + r2 + 14r


∴ 343 = 196 + r2 + 14r


∴ r2 + 14r-147 = 0


∴ r2 + 21r-7r-147 = 0


∴ r(r + 21)-7(r + 21) = 0


∴ (r-7)(r + 21) = 0


∴ r = 7 or r = -21


Since we require r<14 ∴ r = 7 cm



Question 13.

The radii of the circular ends of a solid frustum of a cone are 33 cm and 27 cm and its slant height is 10 cm. Find its total surface area. [Use π = 3.14.]


Answer:

Given: Radius of lower circular end = r = 27 cm


Radius of upper circular end = R = 33 cm


Slant height = l = 10cm


Formula: Total surface area of frustum = πr2 + πR2 + π(R + r)l cm2


= 3.14 × (332 + 272 + (33 + 27) × 10) cm2


= 3.14 × (1089 + 729 + 600) cm2


= 3.14 × 2418 cm2


= 7592.52 cm2


∴ total surface area = 7592.52 cm2



Question 14.

A tent is made in the form of a frustum of a cone surmounted by another cone. The diameter of the base and the top of the frustum are 20 m and 6 m respectively, and the height is 24 m. If the height of the tent is 28 m and the radius of the conical part is equal to the radius of the top of the frustum, find the quantity of canvas required. [Take π = 22/7.]


Answer:


Given: Diameter of base of frustum = 20 m


Diameter of top of frustum = 6 m


∴ Radius of base = R = 20/2 = 10 m


∴ Radius of top = r = 6/2 = 3 m


Height of frustum = hf = 24 m


Height of tent = ht = 28 m


∴ height of cone = hc = ht -hf = 28-24 = 4 m


Formula: Total surface area of frustum = πr2 + πR2 + π(R + r)lf m2


Total surface area of cone = πrlc


Where lf = slant height of frustum & lc = slant height of cone




∴ lf = 25 m


For slant height of cone consider right angled ΔABC from figure


AB = hc = 4 m ; BC = r = 3 m ; AC = lc


By pythagoras theorm


AB2 + BC2 = AC2


∴ 42 + 32 = lc2


∴ lc = 5


Since length cannot be negative lc = 5 m


Since for tent we don’t require the upper circle of frustum and the lower circle of frustum hence we should subtract their area as we don’t require canvas for that.


Surface area of upper circle = πr2


Surface area of lower circle = πR2


∴ Surface area of frustum for which canvas is required = πr2 + πR2 + π(R + r)lf - πr2-πR2cm2


= π(R + r)lfm2


= (22/7) × (10 + 3) × 25 m2


= (22/7) × 325 m2


= 1021.4285 m2


Surface area of cone = πrlc m2


= (22/7) × 3 × 5 m2


= (22/7) × 15 m2


= 47.1428 m2


∴ Quantity of canvas required = surface area of frustum + surface area of cone


= 1021.4285 + 47.1428 m2


= 1068.5713 m2


∴ Quantity of canvas required = 1068.5713 m2



Question 15.

A tent consists of a frustum of a cone, surmounted by a cone. If the diameter of the upper and lower circular ends of the frustum be 14 m and 26 m respectively, the height of the frustum be 8 m and the height of the surmounted conical portion be 12 m, find the area of the canvas required to make the tent. (Assume that the radii of the upper circular end of the frustum and the base of the surmounted conical portion are equal.)


Answer:


Given: Diameter of base of frustum = 26 m


Diameter of top of frustum = 14 m


∴ Radius of base = R = 26/2 = 13 m


∴ Radius of top = r = 14/2 = 7 m


Height of frustum = hf = 8 m


∴ height of cone = hc = 12 m


Formula: Total surface area of frustum = πr2 + πR2 + π(R + r)lf m2


Total surface area of cone = πrlc


Where lf = slant height of frustum & lc = slant height of cone




∴ lf = 10 m


Consider right angled ΔABC from figure


AB = hc = 12 m ; BC = r = 7 m ; AC = lc


By pythagoras theorm


AB2 + BC2 = AC2


∴ 122 + 72 = lc2


∴ lc = 13.892


Since length cannot be negative lc = 13.892 m


Since for tent we don’t require the upper circle of frustum and the lower circle of frustum hence we should subtract their area as we don’t require canvas for that.


Surface area of upper circle = πr2


Surface area of lower circle = πR2


∴ Surface area of frustum for which canvas is required = πr2 + πR2 + π(R + r)lf - πr2-πR2cm2


= π(R + r)lfm2


= 3.14 × (13 + 7) × 10 m2


= 3.14 × 200 m2


= 628 m2


Surface area of cone = πrlc m2


= 3.14 × 7 × 13.892 m2


= 3.14 × 97.244 m2


= 305.346 m2


∴ Quantity of canvas required = surface area of frustum + surface area of cone


= 628 + 305.346 m2


= 933.346 m2


∴ Quantity of canvas required = 933.346 m2



Question 16.

The perimeters of the two circular ends of a frustum of a cone are 48 cm and 36 cm. If the height of the frustum is 11 cm. find its volume and curved surface area.


Answer:

Given: perimeter of upper circle = 36 cm


Perimeter of lower circle = 48 cm


Height of frustum = h = 11 cm


Let r: radius of upper circle & R: radius of lower circle



Total surface area of frustum = πr2 + πR2 + π(R + r)l cm2


Where l = slant height



Now perimeter of circle = circumference of circle = 2π × radius


∴ Perimeter of upper circle = 2πr


∴ 36 = 2 × 3.14 × r


∴ r = 36/6.28 cm


∴ r = 5.732 cm


∴ Perimeter of lower circle = 2πR


∴ 48 = 2 × 3.14 × R


∴ R = 48/6.28 cm


∴ R = 7.643 cm



∴ l = 11.164 cm


∴ Volume of frustum = (1/3) × 3.14 × 11 × (7.6432 + 5.7322 + 7.643 × 5.732) cm3


= (1/3) × 34.54 × (58.415 + 32.855 + 43.809) cm3


= 11.513 × 135.079 cm3


= 1555.164 cm3


∴ Volume of frustum = 1555.164 cm3


Now we have asked curved surface area so we should subtract the top and bottom surface areas which are flat circles.


Surface area of top = πr2


Surface area of bottom = πR2


∴ Curved surface area = total surface area - πr2 - πR2 cm2


= πr2 + πR2 + π(R + r)l - πr2 - πR2 cm2


= π(R + r)l cm2


= 3.14 × (7.643 + 5.732) × 11.164 cm2


= 3.14 × 13.375 × 11.164 cm2


= 468.86 cm2


∴ curved surface area = 468.86 cm2



Question 17.

A solid cone of base radius 10 cm is cut into two parts through the midpoint of its height, by a plane parallel to its base. Find the ratio of the volumes of the two parts of the cone.


Answer:


Let ‘H’ be the height of cone ‘R’ be the radius of base of cone.


R = 10 cm


When the cone is cut at midpoint of H by a plane parallel to its base we get a cone of height H/2 and a frustum also of height H/2


Let the radius of the base of the cone which we got after cutting and the radius of upper circle of frustum be ‘r’ as shown in figure


From figure consider ΔABC and ΔADE


∠ABC = ∠ADE = 90˚


∠BAC = ∠DAE …(common angle)


as two angles are equal by AA criteria we can say that


ΔABC∼ΔADE



∴ r = 5 cm


Let Vc be volume of the cone and Vf be the volume of frustum


Volume of cone = (1/3)π(radius)2(height) cm3


∴ Vc = (1/3) × π × r2 × (H/2) cm3


= (1/3) × π × 52 × (H/2) cm3


= (1/3) × π × 25 × (H/2) cm3


Volume of frustum = (1/3)πh(R2 + r2 + Rr) cm3


∴ Vf = (1/3) × π × (H/2)(102 + 52 + 10 × 5) cm3


= (1/3) × π × (H/2) × 175 cm3



∴ The ratio of volumes of two parts after cutting = Vc:Vf = 1:7



Question 18.

The height of a right circular cone is 20 cm. A small cone is cut off at the top by a plane parallel to the base. If its volume be 1/8 of the volume of the given cone, at what height above the base is the section made?


Answer:


Let the cutting plane be passing through points B and C as shown


Height of cone = AD = H = 20 cm


Height of small cone which we get after cutting = AB = hc


Let ‘r’ be the radius of small cone ∴ we have BC = r


‘R’ be radius of original cone which is to be cut ∴ we have DE = R


From figure consider ΔABC and ΔADE


∠ABC = ∠ADE = 90˚


∠BAC = ∠DAE …(common angle)


as two angles are equal by AA criteria we can say that


ΔABC∼ΔADE



Let V1 be the volume of cone to be cut


Let V2 be the volume of small cone which we get after cutting


Volume of cone = (1/3)π(radius)2(height) cm3


∴ V1 = (1/3) × π × R2 × hc


∴ V2 = (1/3) × π × r2 × 20


Given is that the volume of small cone is (1/8) times the original cone


∴ V2 = (1/8) V1


∴ (1/3) × π × r2 × 20 = (1/8) × (1/3) × π × R2 × hc



Using equation (i) we get



∴ hc3 = 203/8 cm


∴ hc = 20/2 cm


∴ hc = 10 cm


But we have to find the height from base i.e. we have to find BD from figure


∴ 20 = BD + hc


∴ 20 = BD + 10


∴ BD = 10 cm


∴ 10 cm above base the section is made.



Question 19.

A solid metallic right circular cone 20 cm high and whose vertical angle is 60˚, is cut into two parts at the middle of its height by a plane parallel to its base. If the frustum so obtained be drawn into a wire of diameter 1/12cm, find the length of the wire.


Answer:


Let ‘R’ be the radius of the base of the cone which is also the base of frustum i.e. lower circular end as shown in the figure


DE = R


Let ‘r’ be the radius of the upper circular end of frustum which we get after cutting the cone


BC = r


The height of the cone is 20 cm and we had cut the cone at midpoint therefore height of the frustum so obtained is 10 cm


Vertical angle as shown in the figure is 60˚


Now a wire of diameter 1/12 (i.e. radius 1/24) is made out of the frustum let ‘l’ be the length of the wire


As we are using the full frustum to make wire therefore volumes of both the frustum and the wire must be equal.


∴ Volume of frustum = volume of wire made …(i)


Consider ΔABC


∠BAC = 30˚ ; AB = 10 cm ; BC = r



∴ r = 10/√3 cm


Consider ΔADE


∠DAE = 30˚ ; AD = 20 cm ; DE = R



∴ R = 20/√3 cm


Now using equation (i)





∴ 7000/9 = l/576


∴ 777.778 = l/576


∴ l = 448000 cm


Length of wire = 448000 cm


Question 20.

A fez, the cap used by the Turks, is shaped like the frustum of a cone. If its radius on the open side is 10 cm, radius at the upper base is 4 cm and its slant height is 15 cm, find the area of material used for making it. [Use π = 22/7.]



Answer:

Given: Radius of lower circular end = R = 10 cm


Radius of upper circular end = r = 4 cm


Slant height = l = 15cm


Formula: Total surface area of frustum = πr2 + πR2 + π(R + r)l cm2


As the lower circular end is open we need to subtract the area of lower circular end from total surface area since we don’t require material for lower circular end it should be open so that it is wearable.


∴ Total surface area = πr2 + πR2 + π(R + r)l- πR2 cm2


= (22/7) × (42 + (10 + 4) × 15) cm2


= (22/7) × (16 + 210) cm2


= 22 × 226/7 cm2


= 710.285 cm2


∴ total surface area = 710.285 cm2


∴ Area of material used = 710.285 cm2



Question 21.

An oil funnel made of tin sheet consist of a 10 cm long cylindrical portion attached to a frustum of a cone. If the total height is 22 cm, diameter of the cylindrical portion is 8 cm and the diameter of the top of the funnel is 18 cm, find the area of the tin sheet required to make the funnel.



Answer:


Divide the funnel into two parts frustum and cylinder as shown in the figure


Parameters of frustum:


Diameter of upper circular end = 18 cm


∴ Radius of upper circular end = r = 18/2 = 9 cm


The radius of cylinder is equal to the radius of lower circular end of frustum


∴ radius of lower circular end = R = 4 cm


Height of frustum = total height – height of cylinder


= 22 – 10


= 12 cm


∴ height of frustum = h = 12 cm


Total surface area of frustum = πr2 + πR2 + π(R + r)l cm2


Where l = slant height




∴ l = 13 cm


Since for the frustum part of the funnel we don’t require the upper circular end and the lower circular end hence we need to subtract those areas from total surface area.


Area of upper circular end = πr2


Area of lower circular end = πR2


total surface area = πr2 + πR2 + π(R + r)l- πr2- πR2


= π(R + r)l


= 3.14 × (9 + 4) × 13


= 530.66 cm2


∴ total surface area of frustum for which tin is required = 530.66 cm2


Parameters of cylinder:


Height of cylinder = 10 cm


Radius of cylinder = 4 cm


∴ Area of tin require to make cylinder = 2π × (radius) × (height)


= 2 × 3.14 × 4 × 10


= 251.2 cm2


∴ Area of tin required to make the funnel = area of frustum for which tin is required + area


of tin require to make cylinder


∴ area of tin required to make funnel = 530.66 + 251.2


= 781.86 cm2


∴ Area of tin sheet require to make the funnel = 781.86 cm2




Exercise 19d
Question 1.

A river 1.5 m deep and 36 m wide is flowing at the rate of 3.5 km/hr. Find the amount of water (in cubic metres) that runs into the sea per minute.


Answer:

Given,


Depth of the river = 1.5 m


Width of the river = 36 m


Flow rate of river = 3.5 km/hr


Now first change the rate of flow of the water in meter/min


As we know,


1 km = 1000 m


1 hour = 60 minutes


So,





River travel in a minute = 350/6 m


Now,


The amount of water that runs into sea per minute;


350/6 × 1.5 × 36 = 350 × 1.5 × 6 = 3150


So,


The amount of water that runs into the sea per minute is 3150 m3



Question 2.

The volume of a cube is 729 cm3. Find its surface area.


Answer:

Given,


Volume of the cube = 729 cm3


Let the edge of the cube = a cm


So,


Volume (v) of the cube = a3
a3 = 729
a3 = (9cm)3
a = 9 cm


Lateral surface area of cube = 4a2
= 4 × 92
= 4 × 81
= 324cm2
Total surface area of the cube = 6a2
= 6 × 92
= 6 × 81
= 486 cm2



Question 3.

How many cubes of 10 cm edge can be put in a cubical box of 1 m edge?


Answer:

Given,


Edge of the Cubical Box = 1 m


Volume of the Cubical Box = a3


Edge of the cubes = 10 cm


Volume of the cube = a3


Number of Cubes = Volume of box/volume of the cube
= 100 × 100 × 100/10 × 10 × 10
= 1000000/1000
= 1000 cubes



Question 4.

Three cubes of iron whose edges are 6 cm, 8 cm and 10 cm respectively are melted and formed into a single cube. Find the edge of the new cube formed.


Answer:

Given,


Edge of the first cube = 6 cm


Volume of the first cube = a3 = (6)3 cm


Edge of the second cube = 8 cm


Volume of the second cube = a3 = (8)3 cm


Edge of the third cube = 10 cm


Volume of the third cube = a3 = (10)3 cm


So,


Volume of the formed cube = Volume of the First + Second + Third Cube


Volume of the formed cube = v1 + v2 + v3
= 63 + 83 + 103
= 216 + 512 + 1000
= 1728
Now volume of new cube = a3= 1728
Edge of new cube = a = ∛1728
a = 12
Therefore surface area of new cube = 6a2
= 6 × 122
= 6 × 12 × 12
= 864 cm2



Question 5.

Five identical cubes, each of edge 5 cm, are placed adjacent to each other. Find the volume of the resulting cuboid.


Answer:

Given,


Edge of the given Cube = 5 cm


Now,


Length (l) of the resulting cuboid = Edge × Number of cubes


= 5 x 5 = 25 cm


Breadth (b) of the resulting cuboid = 5 cm


Height (h) of the resulting cuboid = 5 cm


So,


Volume of the resulting cuboid = l × b × h


= 25 x 5 x 5


= 625 cm3


Hence,


The volume of the resulting cube is 625 cm3



Question 6.

The volumes of two cubes are in the ratio 8:27. Find the ratio of their surface areas.


Answer:

Given,


Ratio of two cube = 8:27


Let the edges of the cubes to x and y


As,





…………..(i)


Now,


The ratio of the surface areas of the cubes


⇒ 6x2/6y2


= (x/y)2


= (2/3)2 …………………….. [Using (i)]


= 4/9


= 4 : 9


So,


The ratio of the surface areas of the given cubes is 4 : 9.



Question 7.

The volume of a right circular cylinder with its height equal to the radius is . Find the height of the cylinder.


Answer:

Given,


Radius of the right circular cylinder = 176/7 cm3


Height of the right circular cylinder = Radius of the right circular cylinder


So,


⇒ h = r


As,


Volume of the right circular Cylinder = 176/7 cm3


⇒ πr2h = 176/7


⇒ 22/7 × h2 × h=176/7


⇒ h3 =176 × 7/7 × 22


⇒ h3 = 8


⇒ h = 3√8


Therefore,


h = 2 cm


So,


The height of the right circular cylinder is 2 cm



Question 8.

The ratio between the radius of the base and the height of a cylinder is 2:3. If the volume of the cylinder is 12936 cm3. Find the radius of the base of the cylinder.


Answer:

Given,


Ratio of the base and the height of a cylinder is 2:3


So,


Let the radius of the base = r


And


The height of the cylinder = h


r : h = 2 : 3


That is,


r/h = 2/3


So,


h = 3r/2 – – – – – – – – – (i)


As,


Volume of the cylinder = 12936 cm3


⇒ πr2h=12936


⇒ 22/7 × r2 × 3r/2=12936 [Using (i)]


⇒ 33/7 × r3=12936


⇒ r3 = 12936 × 7/33


⇒ r3 = 2744


⇒ r = 3√2744


Therefore,


r = 14 cm


So,


The radius of the base of the cylinder is 14 cm.



Question 9.

The radii of two cylinder are in the ratio of 2:3 and their height are in the ratio of 5:3. Find the ratio of their volumes.


Answer:

Let the radus of the first cylinder = r1


And the radus of the second cylinder = r2;


Let the height of first cylinder = h1


And the height of second cylinder = h2


Given,


r1 : r2 = 2 : 3


r1/r2 = 23 ……………….. (i)


And


h1 : h2 = 5 : 3


h1/h2 = 5/3 ……… (ii)


Now,


The ratio of the volumes of the cylinders =




……… [Using (i) and (ii)]


= 20/27


= 20 : 27


So,


The ratio of the volumes of the given cylinders is 20 : 27



Question 10.

66 cubic cm of silver is drawn into a wire 1 mm in diameter. Calculate the length of the wire in metres.


Answer:

Given,


Volume of the wire = 66 cm3


Diameter of the wire = 1 mm


Radius (r) of wire = 12 = 0.5 mm = 0.05 cm


Let the length of the wire be l


As,


Volume of the wire = 66 cm3


⇒ πr2l = 66


⇒ 227 × 0.05 × 0.05 × l = 66


⇒ l = 66 × 722 × 0.05 × 0.05


Therefore,


l = 8400 cm = 84 m


So,


The length of the wire is 84 m.



Question 11.

If the area of the base of a right circular cone is 3850 cm2 and its height is 84 cm, find the slant height of the cone.


Answer:

Given,


Area of the base of the right circular cone = 3850 cm2


Height of the right circular cone = 84 cm


Let the radius of the cone = r


And the slant height of the cone = l


As,


Area of the base of the cone = 3850 cm2


⇒ πr2 = 3850


⇒ 227 × r2 = 3850


⇒ r2 = 3850 × 722


⇒ r2 = 1225


⇒ r = √1225


Therefore,


r = 35 cm


Now,


length = √h2 + r2


= √(84)2 + (35)2


= √7056 + 1225


= √8281


= 91 cm


So,


The slant height of the given cone is 91 cm.



Question 12.

A cylinder with base radius 8 cm and height 2 cm is melted to form a cone of height 6 cm. Calculate the radius of the base of the cone.


Answer:

Given,


Base radius (r) of the cylinder = 8 cm


Height (h) of the cylinder = 2 cm and


Height (H) of the cone = 6 cm


Let the base radius of the cone = R


Now,


As the cylinder is melted to form the cone,


So,


Volume of the cone = Volume of the cylinder


⇒ 1/3πR2H = πr2h


⇒ R2 = 3r2hH


⇒ R2=3 × 8 × 8 × 26


⇒ R2 = 64


⇒ R = √64


Therefore,


R = 8 cm


So,


The radius of the base of the cone is 8 cm.



Question 13.

a right cylindrical vessel is full of water. How many right cones having the same radius and height as those of the right cylinder will be needed to store that water?


Answer:

Let suppose the radius of the cone = r


And height of the cone = h,


Then,


Radius of the cylindrical vessel = r and


Height of the cylindrical vessel = h


Now,


The required number of cones = Volume of the cylindrical vessel/Volume of a cone


= πr2h/(1/3πr2h)


= 3


So,


The number of the cones that is required to store the water is 3.



Question 14.

The volume of a sphere is 4851 cm3. Find its curved surface area.


Answer:

The volume of the sphere = 4851 cm3


Let the radius of the sphere = r


As,


Volume of the sphere = 4851 cm3


⇒ 4/3πr3 = 4851




⇒ r3 = 92618


⇒ r = 3√92618


⇒ r = 212cm


Now,


The Curved surface area of the sphere = 4πr2


= 4 × 22/7 × 21/2 × 21/2


= 1386 cm2


So,


The curved surface area of the sphere is 1386 cm2



Question 15.

The curved surface area of a sphere is 5544 cm3. Find its volume.


Answer:

Given,


Curved surface area of the sphere = 5544 cm3


Let the radius of the sphere = r


As we know,


Curved surface area of the sphere = 4πr2


So,


⇒ 4πr2 = 5544




⇒ r2 = 441


⇒ r = √441


⇒ r = 21 cm


Now,


Volume of the sphere = 4/3πr3


= 4/3 × 22/7 × 21 × 21 × 21


= 38808 cm3


So,


The volume of the sphere is 38808 cm3.



Question 16.

The surface areas of two sphere are in the ratio of 4:25. Find the ratio of their volumes.


Answer:

Let suppose the radius of first spheres = r


And the radius of second spheres = R


As per the question,


Surface area of the first sphere /Surface area of the second sphere = 4/25


⇒ 4πr2/4πR2 = 4/25


⇒ (r/R)2 = 4/25


⇒ r/R = √4/25


⇒ r/R = 2/5 …………………(i)


Now,




= (r/R)3


= (2/5)3 …………………[Using (i)]


= 8/125


= 8 : 125


So,


The ratio of the volumes of the given spheres will be 8 : 125



Question 17.

A solid metallic sphere of radius 8 cm is melted and into spherical balls each of radius 2 cm. Find the number of spherical balls obtained.


Answer:

Given,


Radius (R) of the solid metallic sphere = 8 cm


Radius (r) of the spherical ball = 2 cm


Now,


The number of spherical balls obtained =



= (R/r)3


= (8/2)3


= 43


= 64


So,


The number of spherical balls obtained is 64.



Question 18.

How many lead shots each 3 mm in diameter can be made from a cuboid of dimensions 9 cm × 12 cm?


Answer:

Given,


Diameter of the lead shot = 3 mm


Radius (r) of a lead shot = 3/2 = 1.5 mm = 0.15 cm and


Dimensions of the cuboid = 9 cm x 11 cm x 12 cm


Now,


The number of the lead shots =




= 84000


So,


84000 number of lead shots can be made from the cuboid.



Question 19.

A metallic cone of radius 12 cm and height 24 cm is melted and made into spheres of radius 2cm each. How many sphere are formed?


Answer:

Cone:

Radius = 12cm


Height = 24 cm.


Sphere, radius = 2cm


Volume of cone is given as,



⇒ V = 3620 cm3


Volume of sphere ,



⇒ V = 33.52 cm3


∴ the number of squares formed will be: 3620/34 = 106



Question 20.

A hemisphere of lead of radius 6 cm is cast into a right circular cone of height 75 cm. Find the radius of the base of the cone.


Answer:

Given,


Radius (R) of the hemisphere = 6 cm


Height (h) of the cone = 75 cm


Let the radius of the base of the cone = r


Now,


Volume of the cone = Volume of the hemisphere





⇒ r2 = 5.76


⇒ r = √5.76


Therefore,


r = 2.4 cm


So,


The radius of the base of the cone is 2.4 cm.



Question 21.

A copper sphere of diameter 18 cm is drawn into wire of diameter 4 mm. Find the length of the wire.


Answer:

The basic concept required to solve any such question is that the volume of the two figures will be same, so here we will equate the volume of sphere to that of wire which is in shape of a cylinder and subsequently will find out the height of the cylinder.


Given diameter of copper sphere = D = 18 cm


∴ Radius of the sphere = R = d/2=18/2 = 9 cm


As we know the wire is cylindrical in shape so,


Let the height of the cylindrical wire be ‘h’ cm


Given diameter of cylindrical wire = d = 4 mm


⇒ Radius of the cylindrical wire = r = d/2= 4/2= 2 mm= 0.2 cm (∵ 1 mm = 0.1 cm)


Volume of a sphere=(where R=radius of sphere)→eqn1


(putting value of R in eqn 1)



⇒ Volume of sphere = 4π × 243= 972π cm3→eqn2


Volume of cylinder = πr2h


Where r = radius of base of cylinder and h = height of cylinder


⇒ Volume of cylindrical wire = π × (0.2)2 × h (putting value of r)


= 0.04π × h cm3→eqn3


Now on equating equation 2 and equation 3, we get,


Volume of sphere = Volume of cylindrical wire


⇒ 972π = 0.04πh


⇒ π(927) = π(0.04h) (taking π common on both sides)


⇒ 927 = 0.04h



∴ h = 24300 cm


The height of cylindrical wire is 24300 cm or 243 m.



Question 22.

The radii of the circular ends of a frustum of height 6 cm are 14 cm and 6 cm respectively. Find the slant height of the frustum.


Answer:


Given height of frustum = h =6 cm


Radius of top = r = 6 cm


Radius of bottom = R = 14 cm


Let the slant height of the frustum be ‘ℓ’ cm


We know in frustum


(Slant height)2 = (height)2 + (R – r)2→eqn1


⇒ ℓ2 = 62 + (14 – 6)2 (putting values of r, R and h in eqn1)


⇒ ℓ2 = 36 + 82


⇒ ℓ2 = 36 + 64


⇒ ℓ2 = 100


⇒ ℓ = √100


∴ ℓ = 10 cm


Slant height of the frustum is 10 cm.



Question 23.

Find the ratio of the volume of a cube to that of a sphere which will fit inside it.


Answer:

Let the radius of the sphere be ‘R’ units


And the cube which will fit inside it be of edge ‘a’ units


Explanation: The longest diagonal of the cube that will fit inside the sphere will be the diameter of the of the sphere.


∴ The longest diagonal of cube = the diameter of the sphere


Consider ΔBCD, ∠BDC = 90°


BD = CD = a units (as they are the edges of cube)



⇒ BC2 = a2 + a2 (putting value of BD and CD)


⇒BC2 = 2a2


⇒BC = √(2a2)


∴ BC = a√2 units →eqn1


Now consider ΔABC, ∠ABC = 90°


Here, AB = a units and BC = a√2 units



⇒AC2 = a2 + (a√2)2 (putting values of AB and BC)


⇒ AC2 = a2 + 2a2


⇒AC2 = 3a2


⇒AC = √(3a2)


∴ AC = a√3 units


∴ Diameter of sphere = D = a√3 units


And we know, D = 2 × R


⇒ R = D/2 (put value of D )



Also, Volume of a sphere→eqn2


Put value of R in eqn2




∴ Volume of sphere = πa2 cubic units → eqn3


Volume of cube = (edge)3


∴ Volume of cube = a3 cubic units →eqn4


Ratio of volume of cube to that of sphere


(putting values from eqn3 and eqn4)


⇒Ratio of volume of cube to that of sphere



Ratio of volume of cube to that of sphere is a:π



Question 24.

Find the ratio of the volumes of a cylinder, a cone and a sphere, if each has the same diameter and same height?


Answer:

Let the radius of cylinder, cone and sphere be ‘r’ cm


Let the diameter of cylinder, sphere and cone be ‘2r’ units


∴ Height of cylinder, cone, sphere = h = 2r units


Volume of cylinder = π(r2)h


⇒ Volume of cylinder = π × r2 × 2r (putting value of h)


∴ Volume of cylinder = 2π × r3→eqn1


Volume of sphere



Volume of cone(put the value of h)


⇒Volume of cone


∴ Volume of cone


Ratio of volume of cylinder to that of a cone to that of a sphere as:


⇒ Volume of cylinder : Volume of Cone : Volume of sphere



(dividing the above relation by (πr3)


(dividing the above ratio by 2)


⇒3 : 1 : 2 (multiplying the above ratio by 3)


Ratio of volume of cylinder to that of cone to that of sphere is 3 : 1 : 2



Question 25.

Two cubes each of volume 125 cm3 are joined end to form a solid. Find the surface area of the resulting cuboid.


Answer:

Given volume of each cube = 125 cm3


Let the edge of each cube be ‘a’ cm


So, Volume of cube = a3


⇒ a3 = 125


⇒ a = ∛(125)


∴ a = 5 cm →eqn1


Now when we join the two cubes then resulting cuboid will be of length twice that of the cube and breadth and height of the resulting cuboid will be same as that of the cube


⇒ length of cuboid = L = 2 × a


⇒ L = 2 × 5 (putting value of a from eqn1)


∴ L = 10 cm


Now, Breadth of cuboid = B = a


⇒ B = 5 cm


Similarly, height of the cuboid = H = 5 cm


Note: Where ever in a question Surface Area is mentioned, it means Total surface area.


Surface area = Total surface area = 2(LB + BH + HL)


⇒ S.A = 2((10 × 5) + (5 × 5) + (10 × 5))


⇒ S.A = 2(50 + 25 + 50)


⇒ S.A = 2 × 125


∴ S.A = 250 cm2


Surface Area of resulting cuboid is 250 cm2.



Question 26.

Three metallic cubes whose edges are 3 cm, 4 cm and 5 cm, are melted and recast into a single large cube. Find the edge of the new cube formed.


Answer:

Let the edges of cubes be a1, a2 and a3


So, a1 = 3 cm, a2 = 4 cm, and a3 = 5 cm


Explanation: Here the sum of volumes of all three cubes will be equal to the volume of the resulting larger cube as the resulting cube is formed by melting the three cubes.


Volume of cube with edge a1 = v1 = (a1)3


⇒v1 = (3)3


∴ v1 = 27 cm3→eqn1


Similarly


Volume of cube with edge a2 = v2 = (a2)3


⇒v2 = (4)3


∴ v2 = 64 cm3→eqn2


Volume of cube with edge a3 = v3 = (a3)3


⇒v3 = (5)3


∴ v3 = 125 cm3→eqn3


Now let the volume of resulting cube be ‘V’ cm3


So, V = v1 + v2 + v3


⇒ V = 27 + 64 + 125 (from eqn1, eqn2 and eqn3)


∴ V = 216 cm3→eqn4


Let the edge of resulting cube be ‘a’ cm


So, volume of the resulting cube = V = a3→eqn5


Equate equation 4 and 5,


⇒ a3 = 216


⇒ a = ∛(216)


∴ a = 6 cm


The edge of new cube formed is 6 cm.



Question 27.

A solid metallic sphere of diameter 8 cm is melted and drawn into a cylindrical wire of uniform width. If the length of the wire is 12 m, find its width.


Answer:

Let the diameter of sphere be ‘D’ and Radius of sphere be ‘R’


∴ D = 8 m


Also, we know


R = D/2


⇒ R = 8/2


∴ R = 4 m


Explanation: Here the volume of sphere will be equal to the volume of the resulting cylinder as the resulting cylinder is formed by melting the sphere.


Volume of the sphere, V1





Let the length/height of the cylinder be ‘H’ and let the radius of the cylinder be ‘r’ and volume of the cylinder be ‘V2


∴ H = 12 m


Volume of the cylinder = V2 = π(r2)H


⇒ V2 = π(r2) × 12 (putting value of H)


⇒ V2 = 12π × r2 m3→eqn2


Now equate equation 1 and 2,


⇒ V2 = V1








∴ r = 2.66 m


Width of cylinder = diameter of cylinder = 2 × radius


⇒ Width of cylinder = 2 × r


⇒ Width of cylinder = 2 × 2.66


∴ Width of cylinder = 5.32 m


Width of the resulting cylinder is 5.32 m



Question 28.

A 5-m-wide cloth is used to make a conical tent of base diameter 14 m and height 24 m. Find the cost of cloth used, at the rate of Rs 25 per metre.


Answer:

Let the length of the cloth used be ‘L’ cm


Area of cloth used = 5 × L →eqn1


Also, Given Diameter = d = 14 m and height = h = 24 m


∴ Radius = r = D/2


⇒ r = 14/2


∴ r = 7 m


Let the slant height of the cone be ℓ m


So, (Slant height)2 = (Height)2 + (Radius)2


Put the values in the above relation


⇒ ℓ2 = h2 + r2


⇒ ℓ2 = 242 + 72


⇒ ℓ2 = 576 + 49


⇒ ℓ2 = 625


⇒ ℓ = √(625)


∴ ℓ = 25 cm →eqn1


Also, we know Curved Surface Area of cone = πrℓ


Where r = radius of base, ℓ = slant height


C.S.A = π × 7 × 25



⇒ C.S.A = 22 × 25


⇒ C.S.A = 550 m2→eqn2


Now the Curved surface area of conical tent will be equal to the area of the cloth used to make the tent


⇒ C.S.A = Area of cloth


⇒ 550 = 5 × L (from eqn1 and eqn2)



∴ L = 110 m


So, cost of the cloth used = rate of cloth × Length of the cloth


⇒ Cost of cloth used = 25 × 110


⇒ Cost of cloth = Rs.2750


Cost of the cloth used is Rs. 2750



Question 29.

A wooden toy was made by scooping out a hemisphere of same radius from each end of a solid cylinder. If the height of the cylinder is 10 cm and its base is of radius 3.5 cm, find the volume of wood in the toy.


Answer:

Given height of cylinder = h = 10 cm


Radius of cylinder = r = 3.5 cm


Radius of hemisphere = R = 3.5 cm


Explanation: In this question the volume of wood in toy can be calculated by subtracting the volume of two hemisphere from the volume of cylinder.


So, volume of cylinder = πr2h


Where r = radius of cylinder and h = height of cylinder


⇒ Volume of cylinder = π × (3.5)2 × 10 (from given values)


⇒ Volume of cylinder = π × 12.25 × 10


∴ Volume of cylinder = 122.5π cm3→eqn1


Volume of a hemisphere(where R is radius of hemisphere)






∴ Volume of two hemisphere


Volume of wood in toy = eqn1 – eqn2


⇒Volume of wood in toy


(taking π common)






∴ Volume of wood in toy = 205.333 cm3


Volume of wood in toy is 205.333 cm3.


Question 30.

Three cubes of a metal whose edge are in the ratio 3:4:5 are melted and converted into a single cube whose diagonal is 12 √3 cm. Find the edges of the three cubes.


Answer:

Let the edges of metal cubes be a1, a2 and a3


And it is given that ratio of edges is 3:4:5


So, let a1 = 3x, a2 = 4x and a3 = 5x


Volume of cube with edge a1 = v1 = (a1)3


⇒v1 = (3x)3


∴ v1 = 27x3 cm3→eqn1


Similarly


Volume of cube with edge a2 = v2 = (a2)3


⇒v2 = (4x)3


∴ v2 = 64x3 cm3→eqn2


Volume of cube with edge a3 = v3 = (a3)3


⇒v3 = (5x)3


∴ v3 = 125x3 cm3→eqn3


Now let the volume of resulting cube be ‘V’ cm3


So, V = v1 + v2 + v3


⇒ V = 27x3 + 64x3 + 125x3 (from eqn1, eqn2 and eqn3)


∴ V = 216x3 cm3→eqn4


It is given that the diagonal of the resulting cube is 12√3 cm



Let the edge of resulting cube be ‘a’ cm


Consider ΔBCD, ∠BDC = 90°


BD = CD = a cm (as they are the edges of cube)



⇒ BC2 = a2 + a2 (putting value of BD and CD)


⇒BC2 = 2a2


⇒BC = √(2a2)


∴ BC = a√2 cm


Now consider ΔABC, ∠ABC = 90°


Here, AB = a cm and BC = a√2 cm and AC = 12√3 cm



⇒(12√3)2 = a2 + (a√2)2 (putting values of AB, AC and BC)


⇒ 144 × 3 = a2 + 2a2


⇒ 144 × 3 = 3a2


⇒ 144 = a2


⇒ a = √144


∴ a = 12 cm


So, volume of the resulting cube = V = a3


⇒ V = 123 (putting value of a)


∴ V = 1728 cm3 –eqn5


Equate equation 4 and 5


⇒ 216x3 = 1728


⇒ x3 = 1728/216


⇒ x3 = 8


⇒ x = ∛8


∴ x = 2


So a1 = 3x = 3 × 2


⇒ a1 = 6 cm


Similarly a2 = 4x = 4 × 2


∴ a2 = 8 cm


Similarly a3 = 5x = 5 × 2


∴ a3 = 10 cm


The edges of three cubes are 6 cm, 8 cm and 10 cm.



Question 31.

A hollow sphere of external and internal diameter 8 cm and 4 cm respectively is melted into a solid cone of base diameter 8 cm. Find the height of the cone.


Answer:

External diameter of hollow sphere = D = 8 cm


⇒ External radius of hollow sphere = R = D/2


⇒ R = 8/2


⇒ R = 4 cm


Internal diameter of hollow sphere = d = 4 cm


⇒ Internal radius of hollow sphere = r = d/2


⇒ r = 4/2


⇒ r = 2 cm


Let the height of the resulting cone be ‘h’ cm


Let the volume of External sphere be V1 and that of internal be V2.


Explanation: Here the volume of hollow sphere will be equal to the volume of the resulting cone as the resulting cone is formed by melting the sphere.


Volume of the External sphere=(put the value of R)









Volume of sphere = V = External volume – Internal volume





Given base radius of the resulting cone = r’ = 8 cm


Le the height be ‘h’ and volume of the resulting cone be V’



(putting value of r' )



Equate equation 3 and 4,


⇒ V = V’



⇒ 224 = 64h


⇒ h = 224/64


∴ h = 3.5 cm


The height of resulting cone is 3.5 cm.



Question 32.

A bucket of height 24 cm is in the form of frustum of a cone whose circular ends are of diameter 28 cm and 42 cm. Find the cost of milk at the rate of Rs 30 per litre, which the bucket can hold.


Answer:

Upper end radius of frustum = d/2= 28/2 = 14 cm


Lower end radius of frustum = D/2 = 42/2 =21 cm


Height of the frustum = 24 cm


And we know,


The amount of milk that bucket can hold = Volume of the bucket


And, Volume of bucket = Volume of Frustum


∴ Amount of milk that bucket can hold = Volume of frustum


⇒Volume of frustum


Where R = Radius of larger or lower end and r = Radius of smaller or upper end and h = height of frustum π = 22/7


⇒Volume of frustum




= 22 × 8 × 133


∴ Volume of frustum = 23408 cm3


Also we know that 1 litre = 1000 cm3


⇒ Volume of frustum in litre = 23408/1000= 23.408 litre


⇒ Amount of milk that the bucket hold = 23.408 litre


⇒ The cost of milk = Rate of milk × amount of milk bucket holds


⇒ The cost of milk = 30 × 23.408


∴ = Rs. 702.24


The cost of milk is Rs.702.24



Question 33.

The interior of a building is in the form of a right circular of diameter 4.2 m and height 4 m surmounted by a cone of same diameter. The height of the cone is 2.8 m. Find the outer surface area of the building.


Answer:

Explanation: Here in order to find th outer surface area of building we need to simply add the curved surface areas of cone and cylinder.


Given height of cylinder = h = 4 m


Height of cone = h’ = 2.8 m


Diameter of cylinder = diameter of cone = d = 4.2 m


⇒ Radius of cone = Radius of cylinder = d/2 =4.2m/2 = 2.1 m


Outer surface area of building = C.S.A of cylinder + C.S.A of cone


Now, C.S.A of cylinder = 2πrh →eqn1


Where r = radius of base of cylinder, h = height of cylinder


And C.S.A of cone = πrℓ →eqn2


Where r = radius of base of cone, ℓ = Slant height of cone


We know in a cone


(Slant height)2 = (height)2 + (radius)2 (put the given values)


(ℓ)2 = (2.8)2 + (2.1)2


⇒ (ℓ)2 = 7.84 + 4.41


⇒ (ℓ)2 = 12.25


⇒ ℓ = √(12.25)


∴ ℓ = 3.5 m


Now, C.S.A of cone = π × 2.1 × 3.5 (putting the values in eqn2)


⇒ C.S.A of cone = 7.35π m2→eqn3


C.S.A of cylinder = 2 × π × 2.1 × 4 (putting the values in eqn1)


⇒ C.S.A of cylinder = 2 × π × 4.41 × 4


∴ C.S.A of cylinder = 16.8π m2→eqn4


Outer surface area of building = eqn3 + eqn4


⇒ Outer surface area = 7.35π + 16.8π


= 24.15π


∴ Outer surface area =75.9 m2


The outer surface area of building is 75.9 m2.



Question 34.

A metallic solid right circular cone is of height 84 cm and the radius of its base is 21 cm. It is melted and recast into a solid sphere. Find the diameter of the sphere.


Answer:

Let the Radius of cone be ‘r’ and height of cone be ‘h’


∴ r = 21 cm and h = 84 cm


Explanation: Here the volume of cone will be equal to the volume of the resulting sphere as the resulting sphere is formed by melting the cone.


Volume of cone,



⇒ V1 = π × 441 × 28


∴ V1 = 12348π m3→eqn1


Let the Radius of resulting sphere be ‘R’ cm


Volume of the sphere=


Now equate equation 1 and 2,


⇒ V2 = V1





⇒ R3 = 3087 × 3


⇒ R3 = 9261


⇒ R =∛(9261)


∴ R = 21 cm


Diameter of Sphere = 2 × radius


⇒ Diameter of Sphere = 2 × R = 42 m


Diameter of the resulting Sphere is 42 m



Question 35.

A toy is in the form of a cone of radius 3.5 cm mounted on a hemisphere of same radius. The total height of the toy is 15.5 cm. Find the total surface area of the toy.


Answer:


Explanation: The total surface area of the toy can be calculated by taking the sum of the Curved surface area of cone and that of hemisphere of same radius.


Given total height of toy = H = 15.5 cm


Radius of hemisphere = Radius of cone = r = 3.5 cm


Now the height of cone = h = total height – radius of hemisphere


⇒ Height of cone = h = 15.5 – 3.5


∴ Height of cone = h = 12 cm


Let the slant height of the cone be ‘ℓ’ cm


Also we know that in a cone,


(Slant height)2 = (Height)2 + (Radius)2


⇒ ℓ2 = 122 + 3.52 (putting the values)


⇒ ℓ2 = 144 + 12.25


⇒ ℓ2 = 156.25


⇒ ℓ = √(156.25)


∴ ℓ = 12.5 cm


C.S.A of cone = πrℓ


⇒ C.S.A of cone = π × 3.5 × 12.5 (putting the given values)


∴ C.S.A of cone = 43.75π m2 →eqn1


C.S.A of hemisphere = 2πr2


⇒ C.S.A of hemisphere = 2 × π × 3.52


= 2 × π × 12.25


= 24.5π m2 →eqn2


Now total surface area of toy = eqn1 + eqn2


⇒ Total surface area of toy = 43.75π + 24.5π


= 68.25π


⇒The total surface area of toy (putting π= 22/7)


= 9.75 × 22


= 214.5 m2


The total surface area of the toy is 214.5 m2.


Question 36.

If the radii of the circular ends of a bucket 28 cm high, are 28 cm and 7 cm, find its capacity and total surface area.


Answer:

Explanation: Here the bucket is in the shape of a frustum. So capacity of bucket will be equal to the volume of the frustum and in order to calculate the total surface area of the bucket we will subtract the top end circular area from the total surface area of the frustum as the bucket is open on top.


Upper end radius of frustum/bucket = R = 28 cm


Lower end radius of frustum/bucket =r = 7 cm


Height of the frustum/bucket = 28 cm


And we know,


The capacity of bucket = Volume of the bucket


And, Volume of bucket = Volume of Frustum


∴ Capacity of bucket = Volume of frustum


⇒Volume of frustum


Where R = Radius of larger or upper end and r = Radius of smaller or lower end and h = height of frustum π = 22/7


⇒Volume of frustum




= 22 × 4 × 343


∴ Volume of frustum = 30184 cm3


∴ Capacity of bucket = 30184 cm3


T.S.A of bucket = T.S.A of frustum – Area of upper circle →eqn1


Let the slant height of the frustum be ‘ℓ’ cm


So, ℓ2 = h2 + (R – r)2


⇒ ℓ2 = 282 + (28 – 7)2


⇒ ℓ2 = 784 + (21)2


⇒ ℓ2 = 784 + 441


⇒ ℓ2 = 1225


⇒ ℓ = √(1225)


∴ ℓ = 35 cm


⇒ T.S.A of frustum = π(R + r)ℓ + πR2 + πr2


= π(28 + 7) × 35 + π(28)2 + π(7)2


= 35 × 35π + 784π + 49π


= 1225π + 784π + 49π →eqn2


Area of upper circle = πR2


= π(28)2


= 784π →eqn3


T.S.A of bucket = 1225π + 784π + 49π – 784π (from eqn2 and 3)


⇒ T.S.A of bucket = 1274π


⇒T.S.A of bucket=


⇒ T.S.A of bucket = 182 × 22


∴ T.S.A of bucket = 4004 cm2


The capacity and total surface area of the bucket is 30184 cm3 and 4004 cm2.



Question 37.

A bucket is in the form of a frustum of a cone with a capacity of 12308.8 cm3 of water. The radii of the top and bottom circular ends are 20 cm and 12 cm respectively. Find the height of the bucket. (Use π = 3.14.)


Answer:

Upper end radius of frustum/bucket = R = 20 cm


Lower end radius of frustum/bucket =r = 12 cm


Height of the frustum/bucket be ‘h’ cm


And we know,


The capacity of bucket = Volume of the bucket


And, Volume of bucket = Volume of Frustum


Volume of frustum/bucket = V = 12308.8 cm3


∴ Capacity of bucket = Volume of frustum


⇒Volume of frustum


Where R = Radius of larger or upper end and r = Radius of smaller or lower end and h = height of frustum π = 3.14



(putting the values)


⇒ 3.14 × h × (144 + 400 + 240) = 12308.8 × 3


⇒ 3.14 × h × 784 = 36926.4


⇒ 2461.76 × h = 36926.4


⇒ h = 36926.4/2461.76


∴ h = 15 cm


The height of the bucket is 15 cm.



Question 38.

A milk container is made of metal sheet in the shape of frustum of a cone whose volume is . The radii of its lower and upper circular ends are 8 cm and 20 cm respectively. Find the cost of metal sheet used in making the container at the rate of Rs 1.40 per cm2.


Answer:

Upper end radius of container = R = 20 cm


Lower end radius of container =r = 8 cm


Height of the container be ‘h’ cm


As container is in shape of frustum


Volume of container = Volume of Frustum





Where R = Radius of larger or upper end and r = Radius of smaller or lower end and h = height of frustum π = 22/7



(putting the values)




⇒ 22 × h × 208 = 73216


⇒ 4576 × h = 73216


⇒ h = 73216/4576


∴ h = 16 cm


Let the slant height of the frustum be ‘ℓ’ cm


So, ℓ2 = h2 + (R – r)2


⇒ ℓ2 = 162 + (20 – 8)2


⇒ ℓ2 = 256 + (12)2


⇒ ℓ2 = 256 + 144


⇒ ℓ2 = 400


⇒ ℓ = √(400)


∴ ℓ = 20 cm


Now the Area of sheet used in making the container can be calculated by simply subtracting the area of the upper circular end from the T.S.A of frustum


⇒ T.S.A of frustum = π(R + r)ℓ + πR2 + πr2


= π(20 + 8) × 20 + π(20)2 + π(8)2


= 28 × 20π + 400π + 64π


= 560π + 464π


= 1024π →eqn1


Area of upper circular end = πR2


= π(20)2


= 400π →eqn2


T.S.A of container = 1024π – 400π (from eqn2 and 1)


⇒ T.S.A of container = 624π



⇒ Cost of metal sheet used = T.S.A of container × Rate per cm2


⇒ Cost of mtal sheet used


= 13728 × 0.2


= Rs. 2745.6


The cost of the metal sheet used is Rs. 2745.6



Question 39.

A solid metallic sphere of diameter 28 cm is melted and recast into a number of smaller cones, each of diameter and height 3 cm. Find the number of cones so formed.


Answer:

Explanation: Here the volume of all the resulting cones will be exactly equal to the volume of the sphere from which they are formed. Son we would find the volume of sphere and then divide the volume of sphere with the volume of one cone to find the number of cones formed.


Diameter of the sphere = D = 28 cm


Radius of the sphere = 28/2


Radius of the sphere = R = 14 cm
Volume of the sphere(put the value of R)


⇒Volume of the sphere




Let the number of cones formed out of the sphere be ‘x’


Diameter of each cone


Given the height of each cone = h = 3 cm


Then, radius








Volume of ‘n’ number of cones = n × volume of one cone


Volumes of ‘m’ number of cones = volume of sphere





⇒ m = 224 × 3


∴ m = 672


The number of cones formed out of the sphere is 672



Question 40.

A cylindrical vessel with internal diameter 10 cm and height 10.5 cm is full of water. A solid cone of base diameter 7 cm and height 6 cm is completely immersed in water. Find the volume (i) displaced out of the cylinder (ii) left in the cylinder.


Answer:

Given internal diameter of cylinder = D = 10 cm


Internal radius of cylinder = R = D/2 = 10/2


Internal radius of cylinder = R = 5 cm


Height of cylinder = H = 10.5 cm


Diameter of solid cone = d = 7 cm


Radius of solid cone = r = d/2 = 7/2


Radius of solid cone = r = 3.5 cm


Height of cone = h = 6 cm


(i)Volume displaced out of cylinder


By Archimedes principle we can easily say that,


Volume displaced out of cylinder = Volume of the solid cone






⇒ V = 22 × 1.75 × 2


∴ V = 77 cm3


The volume displaced out of cylinder is 77 cm3.


(ii)Volume left in cylinder


Volume left in cylinder = Volume of cylinder – Volume displaced out


Volume of cylinder = π(R)2H


⇒ Volume of cylinder = π × (5)2 × 10.5 (putting the given values)


⇒ Volume of cylinder = π × 25 × 10.5


⇒ Volume of cylinder = π × 262.5



⇒ Volume of cylinder = 22 × 37.5


∴ Volume of cylinder = 825 cm3→eqn1


Volume left in cylinder = 825 – 77 (from eqn1 and (i))


∴ Volume left in cylinder = 748 cm3


The volume left in the cylinder is 748 cm3.




Multiple Choice Questions (mcq)
Question 1.

A cylindrical pencil sharpened at one edge is the combination of
A. a cylinder and a cone

B. a cylinder and frustum of a cone

C. a cylinder and a hemisphere

D. two cylinders


Answer:


Pencil is a combination of Cylinder + Cone



Question 2.

A shuttlecock used for playing badminton is the combination of


A. Cylinder and a hemisphere

B. Frustum of a cone and a hemisphere

C. A cone and a hemisphere

D. A cylinder and a sphere


Answer:

Shuttle is a combination of Frustum of a cone + Hemisphere



Question 3.

A funnel is the combination of


A. a cylinder and a cone

B. a cylinder and a hemisphere

C. a cylinder and frustum of a cone

D. a cylinder and a hemisphere


Answer:

Funnel is a combination of Frustum of a cone + Cylinder



Question 4.

A Sarahi is a combination of


A. a sphere and a cylinder

B. a hemisphere and a cylinder

C. a cylinder and a cone

D. two hemispheres


Answer:

Sarahi is a combination of Sphere + Cylinder



Question 5.

The shape of a glass (tumbler) is usually in the form of


A. a cylinder

B. frustum of a cone

C. a cone

D. a sphere


Answer:

Glass is in the shape of a Frustum of a cone.



Question 6.

The shape of a Gilli in the Gilli-Danda game is a combination of


A. a cone and a cylinder

B. two cylinder

C. two cones and a cylinder

D. two cylinder and a cone


Answer:

Gilli in the gili-Danda is a combination of Frustum of a frustum of a cone + Cylinder + cone



Question 7.

A plumbline (sahul) is the combination of


A. A hemisphere and a cone

B. A cylinder and a cone

C. A cylinder and a cone

D. A cylinder and frustum of a cone


Answer:

Plumbline (sahul) is a combination of a Cone + hemisphere.


Question 8.

A cone is cut by a plane parallel to its base and the upper part is removed. The part that is left over is called


A. A cone

B. A sphere

C. A cylinder

D. frustum of a cone


Answer:

When a cone is cut by a plane parallel to its base and the upper part is removed. The part that is left over is called frustum of a cone.



Question 9.

During conversion of a solid from one shape to another, the volume of the new shape will
A. Decrease

B. Increase

C. Remain unaltered

D. Be doubled


Answer:

When a object of certain volume is melted and converted to some other shape, the volume of the new object formed will be the same as the volume of the old object.


Question 10.

In a right circular cone, the cross section made by a plane parallel to the base is a
A. Sphere

B. Hemisphere

C. Circle

D. A semicircle


Answer:

In a right circular cone, the cross section made by a plane parallel to the base is a cicle.



Question 11.

A solid piece of iron in the form of a cuboid of dimensions (49 cm × 33 cm × 24 cm) is moulded to form a solid sphere. The radius of the sphere is
A. 19 cm

B. 21 cm

C. 23 cm

D. 25 cm


Answer:

Given: Dimension of cuboid (49 cm × 33 cm × 24 cm)


Volume of cuboid is : length × breadth × height


Volume of Solid Sphere is : × π × r3 (here r is radius of the sphere)


Let v1 be the volume of given cuboid.


∴ v1 = 49 cm × 33 cm × 24 cm = 38808 cm3


Let v2 be the volume of Solid Sphere.


We know that when a object is moulded from one shape to other its volume does not change.


∴ v1=v2


That is,


38808 = × π × r3


r3 = × 38808


r3 = 9261 r = ∛9261 = 21


∴ r = 21cm


That is, radius of the Solid sphere = 21cm


Question 12.

The radius (in cm) of the largest right circular cone that can be cut out from a cube of edge 4.2 cm is
A. 2.1

B. 4.2

C. 8.4

D. 1.05


Answer:

Given: edge of the cube = 4.2 cm


A right circular cone is a Cone whose height is perpendicular to the diameter (radius) of the base circle.


In a cube, a largest right circular Cone is formed when its base lies on one of the faces of the Cube and its tip lies on the opposite face.



∴ Diameter of largest right circular Cone in Cube = edge length of cube.


∴ Diameter = 4.2 cm


⇒ Radius = = = 2.1 cm


∴ Radius of the largest right circular Cone in Cube is 2.1 cm


Question 13.

A metallic solid sphere of radius 9 cm is melted to form a solid cylinder of radius 9 cm. The height of the cylinder is
A. 12 cm

B. 18 cm

C. 36 cm

D. 96 cm


Answer:

Given: Radius of the solid sphere = 9 cm


Radius of the cylinder = 9 cm


Volume of Solid Sphere is: × π × r3 (here r is radius of the sphere)


Volume of Solid Cylinder is: π × r2 × h (here r is radius and h is height of the cylinder)


Let v1 be the volume of given Solid Sphere.


∴ v1 = × π × 93 cm3


Let v2 be the volume of Solid Cylinder.


∴ v2 = π × 92 × h


We know that when a object is moulded from one shape to other its volume does not change.


∴ v1=v2


That is,


× π × 93 = π × 92 × h


h = = = 12 cm


∴ h = 12 cm


That is Height of the Cylinder is 12 cm


Question 14.

A rectangular sheet of paper 40 cm × 22 cm, is rolled to form a hollow cylinder of height 40 cm. The radius of the cylinder (in cm) is
A. 3.5

B. 7

C.

D. 5


Answer:

Given: Dimensions of rectangular sheet: 40cm × 22cm


Height of the Hollow Cylinder : 40cm


Area of the Rectangle is = length × breadth


Curved surface Area of the Cylinder = 2πrh (where r and h are radius and height of cylinder respectively)


Let a1 be the area of Rectangle


∴ a1 = 40 × 22 cm2


Let a2 be the Curved surface area of Cylinder


∴ a2 = 2 × π × r × 40 cm2


We know that when area of a surface doesn’t change even if its shape is changed.


∴ a1 = a2


⇒ 40 × 22 = 2 × π × r × 40


⇒ r = = = = = 3.5 cm


∴ Radius of the Cylinder is 3.5cm


Question 15.

The number of solid sphere, each of diameter 6 cm, that can be made by melting a solid metal cylinder of height 45 cm and diameter 4 cm, is
A. 2

B. 4

C. 5

D. 6


Answer:

Given: Diameter of the Solid Sphere is: 6 cm


Height of the Cylinder is: 45cm


Diameter of the Cylinder is: 4cm


Volume of Solid Cylinder is: π × r2 × h (here r is radius and h is height)


Volume of the Solid Sphere is: × π × r3 (here r is the radius of the Sphere)


Let v1 be the volume of given Cylinder


∴ v1 = π × 22 × 45 cm3 (4cm is diameter, ∴ 2cm is the radius of cylinder)


Let v2 be the volume of Solid Sphere.


V2 = × π × 33 cm3 (6cm is the diameter, ∴ 3cm is the radius of the Sphere)


We know that when a object is moulded from one shape to other its volume does not change.


Let n be the number of Solid Sphere of diameter 6cm required.


∴ v1=n × v2 (volume of n Spheres = volume of Cylinder)


That is,


π × 22 × 45 =n × × π × 33


n = = = = 5


That is 5 Solid Spheres of diameter 6 cm can be formed by the Solid Cylinder of height 45 cm and diameter 4 cm.


Question 16.

The surface areas of two sphere are in the ratio 16:9. The ratio of their volume is
A. 64:27

B. 16:9

C. 4:3

D. 163:93


Answer:

Given: Surface area ratio of two Spheres is: 16:9


Volume of the Sphere is: × π × r3 (where r is radius of sphere)


Surface area of the sphere is: 4 × π × r2 (where r is radius of sphere)


Let S1 and S2 be two different spheres.


(Surface area of) S1: (Surface area of) S2 = 16:9


4 × π × (r1)2: 4 × π × (r2)2 = 16:9 (here r1 and r2 are the radii of S1 and S2 respectively)


(r1)2: (r2)2 = 16:9


r1: r2 = √16:√9


r1: r2 = 4:3


Now,


Let V1 and V2 be the volumes of the spheres S1 and S2 respectively.


∴ V1:V2 = × π × (r1)3: × π × (r2)3 (here r1 and r2 are the radii of S1 and S2 respectively)


⇒ V1:V2 = (r1)3: (r2)3


⇒ V1:V2 = (4)3: (3)3


⇒ V1:V2 = 64:27


∴ The ratios of the volumes is: 64:27


Question 17.

If the surface area of a sphere is 616 cm2, its diameter (in cm) is
A. 7

B. 14

C. 28

D. 56


Answer:

Given: surface area of a sphere is 616 cm2


Surface area of the sphere is: 4 × π × r2 (where r is radius of sphere)


∴ 4 × π × r2 = 616 cm2


r2 = = = 49


r = √49 = 7 cm


∴ Diameter = 2 × r = 2 × 7 = 14cm


Question 18.

If the radius of a sphere becomes 3 times then its volume will become
A. 3 times

B. 6 times

C. 9 times

D. 27 times


Answer:

Let r1 be the initial radius of the sphere.


∴ r1= r


Let r2 be the radius after increasing it 3 times the size of initial radius.


∴ r2 = 3r


Let V1 be the initial volume of the sphere


∴ V1 = × π × (r1)3 = × π × r3


Let V2 be the volume of the sphere after its radius is increased by 3 times.


∴ V2 = × π × (r2)3 = × π × (3r)3


⇒ V2 = 27 × × π × r3


∴ If radius is increased by 3 times, its volume will be increased by 27 times.


Question 19.

If the height of a bucket in the shape of frustum of a cone is 16 cm and the diameter of its two circular ends are 40 cm and 16 cm then its slant height is
A. 20 cm

B. 12 √5 cm

C. 8 √13

D. 16 cm


Answer:

Given: Height of the frustum of a cone: 16 cm


Diameters of the Circular ends: 40cm and 16 cm.


Radius of the Circular ends: = 20cm and = 8cm



Here slant height h can be found by using Pythagoras theorem.


∴ s2 = h2 + (R-r)2 (here R is 20cm and r is 8cm)


⇒ s2 = 162 + (20—8)2


⇒ s2 = 162 + (12)2


⇒ s2 = 256 + 144


⇒ s2 = 400


⇒ s = √400 = 20


∴ Slant height 0f the Frustum is 20cm


Question 20.

A sphere of diameter 18 cm is dropped into a cylinder vessel of diameter 36 cm, partly filled with water. If the sphere is completely submerged then the water level rises by
A. 3 cm

B. 4 cm

C. 5 cm

D. 6 cm


Answer:

Given: Diameter of a sphere: 18cm ⇒ radius = = 9cm


Diameter of Cylindrical vessel: 36cm ⇒ radius = = 18cm


It is given that Sphere is dropped into the cylindrical vessel containing some water.


∴ Volume of sphere = Volume of water in Cylinder displaced (raised)


Let V1 be the volume of the Sphere


∴ V1 = × π × (r1)3


V1 = × π × 93


Let V2 be the volume of the water displaced in the cylindrical vessel


∴ V2 = π × (r2)2 × h (here r2 is the radius of the Cylinder and h is the level of water raised in the vessel after dropping the sphere into the cylindrical vessel)


V2 = π × 182 × h


Since V1=V2


× π × 93 = π × 182 × h


h = = = 3cm


∴ The water level rises by 3cm when the dropped sphere is completely submerged in the cylindrical vessel.


Question 21.

A solid right circular cone is cut into two parts at the middle of its height by a plane parallel to its base. The ratio of the volume of the smaller cone to the whole cone is
A. 1:2

B. 1:4

C. 1:6

D. 1:8


Answer:


Given: A solid right circular cone is cut into two parts at the middle of its height by a plane parallel to its base


Let ‘H’ be the height of the cone.


Let ‘R’ be the Radius of the complete cone.


Volume of a cone is given by: πr2h


Here,


AB = BD =


Let r be the radius of the smaller cone.


∴ In ΔABC and ΔADE


∠ABC = ADE (90°)


∠CAB = ∠EAB (common)


∴ ΔABC ΔADE (AA similarity criterion)


= (Corresponding sides are proportional)


=


⇒ R = 2r


Volume of smaller cone = π(r)2 × h = π(BC)2 × AB = π(r)2 × = cm3


Volume of whole cone = π(r)2 × h = π(DE)2 × AD = π(2r)2 × H = πr2H cm3


= =


∴ The ratio of the volume of the smaller cone to the whole cone is 1:8


Question 22.

The radii of the circular ends of a bucket of height 40 cm are 24 cm and 15 cm. The slant height (in cm) of the bucket is
A. 41

B. 43

C. 49

D. 51


Answer:

Bucket is in the shape of a frustum of a cone


Therefore,


Given: Height of the frustum: 40 cm


Radius of the Circular ends: 24cm and 15cm



Here slant height h can be found by using Pythagoras theorem.


∴ s2 = h2 + (R-r)2 (here R is 20cm and r is 8cm)


⇒ s2 = 402 + (24—15)2


⇒ s2 = 402 + (9)2


⇒ s2 = 1600 + 81


⇒ s2 = 1681


⇒ s = √1681 = 41


∴ Slant height 0f the Frustum is 41cm


Question 23.

A solid is hemispherical at the bottom and conical (of same radius) above it. If the surface areas of the two parts are equal then the ratio of its radius and the slant the height of the conical part is
A. 1:2

B. 2:1

C. 1:4

D. 4:1


Answer:

Given: Bottom of a solid is hemispherical and conical above it, both have same radius


and same surface areas.


∴ CSA of hemisphere = CSA of Cone


⇒ 2 × π × r2 = πrl (where r is the radius and l is the slant height)


=


=


∴ r:l = 1:2


That is ratio of radius and the slant height of the given solid is 1:2.


Question 24.

If the radius of the base of a right circular cylinder is halved, keeping the height the same. Then the ratio of the volume of the cylinder thus obtained to the volume of original is
A. 1 : 2

B. 2 : 1

C. 1 : 4

D. 4 : 1


Answer:

Given: Radius of the base of a right circular cylinder is halved, keeping the height the same.


Let initial Radius of Right Circular Cylinder be ‘r’.


∴ Radius of the Cylinder after its radius is halved is ‘


Let ‘h’ be the height of the both the cylinders.


Let V1 be the volume of the initial Cylinder.


Let V2 be the volume of the Cylinder after the initial Cylinders base radius is halved.


Volume of the Cylinder is given by: πr2h


∴ V1:V2 = π(r1)2h : π(r2)2h


∴ V1:V2 = π(r)2h : π(r/2)2h


⇒ V1:V2 = 1:


⇒ V1:V2 = 4:1


Thus, ratio of the volume of the cylinder thus obtained to the volume of original is 4:1.


Question 25.

A cubical ice-cream brick of edge 22 cm is to be distributed among some children by filled ice-cream cones of radius 2 cm and height 7 cm up to its brim. How many children will get the ice-cream cones?
A. 163

B. 263

C. 363

D. 463


Answer:

Given: Cubical ice-cream brick of edge 22.


Ice-cream cone of radius 2 cm and height 7 cm.


Let ‘n’ be the number of students who get ice-cream cones.


Let V1 be the volume of the Cubical Ice-cream brick.


Volume of Cube is given by: a3 (where a is the edge length)


∴ V1 = a3 = 223


Let V2 be the Volume of the Ice-cream Cone.


Volume of Cone is given by: × π × r2 × h (where r is the radius of the base and h is the edge height of the cone)


∴ V2 = × π × r2 × h = × π × 22 × 7


Here,


V1 = n × V2


∴ 223 = n × × π × 22 × 7


n = = = 363


∴ 363 Children can get ice cream cones.


Question 26.

A mason constructs a wall of dimension (270 cm × 300cm × 350cm) with bricks, each of size (22.5cm × 11.25cm × 8.75cm) and it is assumed that 1/8 space is covered by the mortar. Number of bricks used to construct the wall is
A. 11000

B. 11100

C. 11200

D. 11300


Answer:

Given: A wall of dimension (270 cm × 300cm × 350cm).


A wall of dimension (22.5cm × 11.25cm × 8.75cm).


space is covered by the mortar.


Here volume of the wall is covered by mortar, ∴ 1 – = volume of the wall is covered by bricks.


Volume of the cuboid is given by: lbh (here l, b, h are length , breadth, height respectively).


Let V1 be the volume of the wall.


∴ V1 = l × b × h = 270 × 300 × 350


⇒ V1 = 270 × 300 × 350 = 28350000 cm2


Let V2 be the volume of the brick.


∴ V2 = l × b × h = 22.5 × 11.25 × 8.75


⇒ V2 = 22.5 × 11.25 × 8.75 = cm2


Let ‘n’ be the number of bricks required to occupy volume of the wall.


∴ n × = 28350000 ×


⇒ n = 28350000 × × = 11200


∴ n = 11200


That is 11200 bricks required to occupy volume of the wall.


Question 27.

Twelve solid sphere of the same size are made by melting a solid metallic cylinder of base diameter 2 cm and height 16 cm. The diameter of each sphere is
A. 2 cm

B. 3 cm

C. 4 cm

D. 6 cm


Answer:

Given: A solid metallic cylinder of base diameter 2 cm and height 16 cm.


Twelve solid sphere of the same size are made by melting a solid metallic cylinder .


Let V1 be the volume of the cylinder


Volume of the cylinder is given by: π × r2 × h


∴ V1 = π × 12 × 16 (diameter is 2cm, ∴ radius is 1cm)


Let V2 be the volume of the each Sphere.


Volume of the Sphere is given by: × π × r3 (where r is the radius of the sphere)


∴ V2 = × π × r3


Here,


V1 = 12 × V2


∴ π × 12 × 16 = 12 × × π × r3


⇒ r3 = = = 1cm


∴ r = ∛1 = 1 cm


∴ Diameter of the Sphere is 2 × r = 2 × 1 = 2cm


Question 28.

The diameter of two circular ends of a bucket are 44 cm and 24 cm, and the height of the bucket is 35 cm. The capacity of the bucket is
A. 31.7 litres

B. 32.7 litres

C. 33.7 litres

D. 34.7 litres


Answer:

Given: Diameter of two circular ends of a bucket are 44 cm and 24 cm, and the height of the bucket is 35 cm.


Bucket is in the shape of frustum.


Let V be the Volume of the Bucket(Frustum)


Volume of the frustum is given by: × h × (R2 + r2 + Rr) (here r and R are the radii of smaller and larger circular ends respectively)


∴ V = × h × (R2 + r2 + R × r)


⇒ V = × 35 × (222 + 122 + 22 × 12) (diameters are 44 and 24 cm, ∴ their radii are 22cm and 12cm respectively)


⇒ V = × 35 × (484 + 144 + 264) = × 35 × (892)


⇒ V = × 35 × (892) = 32706.6 cm3 = 32.7 litres (∵ 1000cm3 = 1 litre)


∴ The capacity of the bucket is: 32.7 litres


Question 29.

The slant height of a bucket is 45 cm and the radii of its top and bottom are 28 cm and 7 cm respectively. The curved surface area of the bucket is
A. 4953 cm2

B. 4952 cm2

C. 4951 cm2

D. 4950 cm2


Answer:

Given: slant height of a bucket is 45


Radii of its top and bottom are 28 cm and 7 cm respectively.


Bucket is in the shape of a frustum.


Let A be the CSA of the Frustum


CSA of the frustum is given by: πl(r1 + r2) (here l is the slant height and r1 and r2 are the radii of top and bottom of the bucket(frustum))


∴ A = πl(r1 + r2)


⇒ A = π × 45 × (28 + 7) = π × 45 × (35) = 4950 cm2


∴ A = 4950 cm2


That is , curved surface area of the bucket is 4950 cm2


Question 30.

The volumes of two Spheres are in the ratio 64: 27. The ratio of their surface areas is
A. 9:16

B. 16:9

C. 3:4

D. 4:3


Answer:

Given: Volume ratio of two Spheres is: 64:27


Volume of the Sphere is: × π × r3 (where r is radius of sphere)


Surface area of the sphere is: 4 × π × r2 (where r is radius of sphere)


Let S1 and S2 be two different spheres.


(Volume of) S1: (Volume of) S2 = 64:27


× π × (r1)3: × π × (r2)3 = 64:27 (here r1 and r2 are the radii of S1 and S2 respectively)


(r1)3: (r2)3 = 64:27


r1: r2 = ∛64:∛27


r1: r2 = 4:3


Now,


Let SA1 and SA2 be the surface areas of the spheres S1 and S2 respectively.


∴ SA1:SA2 = 4 × π × (r1)2:4 × π × (r2)2 (here r1 and r2 are the radii of S1 and S2 respectively)


⇒ SA1:SA2 = (r1)2: (r2)2


⇒ SA1:SA2 = (4)2: (3)2


⇒ SA1:SA2 = 16:9


∴ The ratio of the Surface area of spheres is: 16:9


Question 31.

A hollow cube of internal edge 22 cm is filled with spherical marbles of diameter 0.5 cm and 1/8 space of the cube remains unfilled. Number of marbles required is
A. 142296

B. 142396

C. 142496

D. 142596


Answer:

Given: hollow cube of internal edge 22 cm


Spherical marbles of diameter 0.5 cm ⇒ radius = = 0.25 cm


space of the cube remains unfilled.


Here, space of the cube is unfilled, that is of the cube is filled with marbles of radius 0.25cm.


Volume of the sphere is : × π × r3 (where ‘r’ is the radius of the cube)


Let V1 be the volume of the Sphere.


∴ V1 = × π × (0.25)3 = cm3


Volume of the Cube : a3 (where ‘a’ is the edge length of the cube)


Let V2 be the volume of the cube.


∴ V1 = (22)3 = 10648 cm3


Let ‘n’ be the number of sphere required to fill the space of the cube.


∴ n × V1= V2 ×


⇒ n × = 10648 ×


⇒ n × = 9317


⇒ n = 9317 × = 142296


∴ Number of marbles required to fill of the cube id 142296.


Question 32.

A metallic spherical shell of internal and external diameter 4 cm and 8 cm respectively, is melted and recast into the form of a cone of base diameter 8 cm. The height of the cone is
A. 12 cm

B. 14 cm

C. 15 cm

D. 8 cm


Answer:

Given: Metallic spherical shell of internal and external diameter 4 cm and 8 cm respectively


Recast into the form of a cone of base diameter 8 cm.


Volume of spherical shell is : × π × [(r1)3 — (r2)3] (here r1 an r2 are External and internal radii respectively)


Let v1 be the volume of given Spherical shell.


∴ v1 = × π × [()3 — ()3] = × π × [(4)3 — (2)3] = × π × [64 — 8] = × π × 56 cm3


Volume of the cone is given by : × π × r2 × h


Let v2 be the volume of cone.


∴ V2 = × π × ()2 × h = × π × 16 × h cm3


We know that when a object is moulded from one shape to other its volume does not change.


∴ v1=v2


That is,


× π × 56 = × π × 16 × h


h = = = 14


∴ h = 14cm


That is, height of the cone = 14cm


Question 33.

A medicine capsule is in the shape of a cylinder of diameter 0.5 cm with two hemisphere struck to each of its ends. The length of the entire capsule is 2 cm. The capacity of the capsule is
A. 0.33 cm3

B. 0.34 cm3

C. 0.35 cm3

D. 0.36 cm3


Answer:

Given: A cylinder of diameter 0.5 cm


The length of the entire capsule is 2 cm


Here, Radius of the Cylinder and Hemisphere is = 0.25cm


Height of the Cylinder = length of capsule – 2 × (radius of hemisphere) = 2 – 0.5 = 1.5cm


Volume of Cylinder is: πr2h (here r, h are radius and height of the cylinder respectively)


Volume of hemisphere is: πr3 (here r is the radius of the hemisphere)


Let V1 be the Cylindrical part of the capsule


∴ V1 = πr2h = π × (0.25)2 × (1.5) = 0.09375π cm3


Let V2 be the Cylindrical part of the capsule


∴ V1 = πr3 = π(0.25)3 = π cm3


∴ Volume of the capsule = V1 + 2V2 = 0.09375π + 2 × π = 0.3599 0.36cm3


∴ Volume of the capsule is 0.36cm3


Question 34.

The length of the longest pole that can be kept in a room (12 m × 9 m × 8 m) is
A. 29 m

B. 21 m

C. 19 m

D. 17 m


Answer:

Given: A room with dimensions (12 m × 9 m × 8 m)


Room is in the shape of a Cuboid.
Longest rod that can be placed in a room is nothing but its diagonal.
Let length of the rod be ‘L’.


Length of diagonal of a Cuboid = √(l2 + b2 + h2)


∴ L = √(l2 + b2 + h2)


L = √(122 + 92 + 82) m
⇒ L= √(144 + 81 + 64) m
⇒ L = √289 m
⇒ L = 17 m
∴ The length of the longest rod is 17 m


Question 35.

The length of the diagonal of a cube is 6 √ 3 cm. Its total surface area is
A. 144 cm2

B. 216 cm2

C. 180 cm2

D. 108 cm2


Answer:

Given: Length of the diagonal of a cube is 6 √ 3 cm.


We know that the Length of a diagonal of a cube is given by: a√ 3 (here ‘a’ is edge length of cube)


∴ a√ 3 = 6√ 3


⇒ a = 6cm


∴ Edge length of the cube is 6cm.


Surface area of a cube is given by: 6a2 (here ‘a’ is the edge length)


Let ‘S’ be the surface area of the given Cube.


∴ S = 6a2


⇒ S = 6 × (6)2


⇒ S = 216 cm2


∴ Surface area of the given cube is 216 cm2


Question 36.

The volume of a cube is 2744 cm3. Its surface area is
A. 196 cm2

B. 1176 cm2

C. 784 cm2

D. 588 cm2


Answer:

Given: volume of a cube is 2744 cm3.


We know that the Volume of a cube is given by: a3 (here ‘a’ is edge length of cube)


∴ a3 = 2744


⇒ a = ∛ 2744


⇒ a =14cm


∴ Edge length of the cube is 14cm.


Surface area of a cube is given by: 6a2 (here ‘a’ is the edge length)


Let ‘S’ be the surface area of the given Cube.


∴ S = 6a2


⇒ S = 6 × (14)2


⇒ S = 1176 cm2


∴ Surface area of the given cube is 1176 cm2


Question 37.

The total surface area of a cube is 864 cm2. Its volume is
A. 3456 cm3

B. 432 cm3

C. 1728 cm3

D. 3456 cm3


Answer:

Given: The total surface area of a cube is 864 cm2.


We know that the Volume of a cube is given by: 6a2 (here ‘a’ is edge length of cube)


∴ 6a2 = 864


⇒ a2 = = 144


⇒ a =√ 144


⇒ a =12cm


∴ Edge length of the cube is 12cm.


Volume of a cube is given by: a3 (here ‘a’ is the edge length)


Let ‘V’ be the Volume of the given Cube.


∴ V = a3


⇒ V = (12)3


⇒ V = 1728 cm3


∴ Volume of the given cube is 1728 cm3


Question 38.

How many bricks each measuring (25 cm × 11.25 cm × 6 cm) will be required to construct a wall (8 m × 6 m × 22.5 cm)?
A. 8000

B. 6400

C. 4800

D. 7200


Answer:

Given: Bricks each measuring (25 cm × 11.25 cm × 6 cm)


A wall with dimensions (8 m × 6 m × 22.5 cm)


We know that the Brick and wall are in the shape of a Cuboid.


∴ Volume of a Brick and wall is given by: l × b × h (here l,b,h are the length , breadth, height of the wall and brick)


Let V1 be the volume of the Wall.


∴ V1 = l × b × h = 800 × 600 × 22.5 (∵ 8 m = 800cm and 6 m = 600cm)


⇒ V1 = 10800000 cm3


Let V2 be the volume of a Brick.


∴ V2 = l × b × h = 25 × 11.25 × 6


⇒ V2 = 1687.5 cm3


Let ‘n’ be the number of Bricks required to build the Wall.


∴ V1 = n × V2


10800000 = n × 1687.5


⇒ n = = 6400


∴ n = 6400


That is, 6400 bricks are required to build the wall.


Question 39.

The area of the base of a rectangular tank is 6500 cm2 and the volume of water contained in it is 2.6 m3. The depth of water in the tank is
A. 3.5 m

B. 4 m

C. 5 m

D. 8 m


Answer:

Given: The area of the base of a rectangular tank is 6500 cm2.


Volume of water contained in it is 2.6 m3.


We know that Rectangular tank is in the shape of a cuboid.


And also area of the base of a cuboid is given by : l × b (here l and b are length and breadth respectively)


∴ l × b = 6500 cm2 ….1


Volume of a cuboid is given by: l × b × h (here l, b, h are length, breadth, height respectively)


∴ l × b × h = 2.6 m3 = 2.6 × 1000000 cm3 = 2600000 cm3 …2


From —1 and —2


6500 × h = 2600000


h = = 400 cm = m = 4m


∴ h = 4m


That is, depth of water in the tank is 4m


Question 40.

The volume of a wall, 5 times as high as it is broad and 8 times as long as it is high, is 12.8 m3. The breadth of the wall is
A. 30 cm

B. 40 cm

C. 22.5 cm

D. 25 cm


Answer:

Given: The volume of a wall is 12.8 m3.


Height is 5 times the breadth and Length is 8 times the height.


Let breadth = x


∴ Height = 5x


⇒ Length = 8 × (5x) = 40x


Volume of a Cuboid is given by: l × b × h (here l, b, h are, length, breadth, height respectively)


∴ l × b × h = 12.8


⇒ x × 5x × 40x = 12.8


⇒ 200x3 = 12.8


⇒ x3 = m3 =


⇒ x= ∛ m = m = 0.4 m


∴ x = 0.4m = 0.4 × 100 = 40cm


That is Breadth is: 40cm


Question 41.

If the areas of three adjacent faces of a cuboid are x, y, z respectively then the volume of the cuboid is
A. xyz

B. 2 xyz

C.

D.


Answer:

Given: areas of three adjacent faces of a cuboid are x, y, z respectively.


Let l, b, h be the length , breadth, height of the cuboid respectively.


∴ l × b = x – 1


b × h = y – 2


h × l = z – 3


Volume of the cuboid is: l × b × h


multiply eq’s –1, –2, –3


That is ,


l × b × b × h × l × h = x × y × z l2 × b2 × h2


⇒ l2 × b2 × h2 = xyz


⇒ (l × b × h)2 = xyz


⇒ (V)2 = xyz (∵Volume of the cuboid is: l × b × h)


⇒ V = √xyz


∴ V = √xys


That is volume of the given Cuboid is : √xyz


Question 42.

The sum of length, breadth and height of a cuboid is 19 cm and its diagonal is 5 √ 5 cm. Its surface area is
A. 361 cm2

B. 125 cm2

C. 236 cm2

D. 486 cm2


Answer:

Given: Sum of length, breadth and height of a cuboid is 19 cm.


Length of diagonal is 5 √ 5 cm.


Let l, b, h be the length , breadth, height of the cuboid respectively.


∴ l + b + h = 19cm – 1


Length of a diagonal in a cuboid is given by : √(l2 + b2 + h2)


∴ √(l2 + b2 + h2) = 5√5 cm ⇒ (l2 + b2 + h2) = (5√5)2 = 125 cm2 – 2


Surface area of Cuboid is: 2(lb + bh + hl)


On squaring eq – 1 on both sides


We get


(l + b + h)2 = 192


⇒ l2 + b2 + h2 + 2(lb + bh + hl) = 361 cm2


⇒ 125+ 2(lb + bh + hl) = 361 ( from eq – 2)


⇒ 2(lb + bh + hl) = 361 – 125 = 236 cm2


∴ 2(lb + bh + hl) = 236 cm2


That is, Surface area of the given Cuboid is 236cm2


Question 43.

If each edge of a cube is increased by 50%, the percentage increase in the surface area is
A. 50%

B. 75%

C. 100%

D. 125%


Answer:

Given: Edge of a cube is increased by 50%


Let ‘a’ be the Edge of the cube


Area of the cube is : 6l2 (where ‘l’ is the edge of a cube)


Let A1 be the initial surface area of the cube.


∴ A1 = 6a2


Now,


50% of the edge is: a × =


∴ Edge = a + after increasing edge by 50%


That is edge of the cube after increasing it by 50% is


Let A2 be the surface area of the cube after increasing the edge by 50%


∴ A2= 6 × ()2 = × 6a2


Here increase in area = A2 –A1


⇒ increase in area = × 6a2 – 6a2 = × 6a2


Now, increase in percentage is:


Increase in % = × 100 = × 100 = 125%


∴ If each edge of a cube is increased by 50%, then the percentage increase in the surface area is 125%


Question 44.

How many bags of grain can be stored in a cuboidal granary (8 m × 6 m × 3 m), if each bag occupies a space of 0.64 m3?
A. 8256

B. 90

C. 212

D. 225


Answer:

Given: cuboidal granary with dimensions (8 m × 6 m × 3 m)


Volume of the bag : 0.64 m3


Volume of a Cuboid is given by: l × b × h (here l, b, h are length, breadth, height of the Cuboid respectively)


Let ‘V’ be the Volume of the Cuboidal granary.


∴ V = l × b × h = 8 × 6 × 3 = 144


Let ‘n’ be the number of bags that can fit in cuboidal granary.


∴ n × 0.64 = 144


⇒ n = = 225


∴ n = 225


That is a total of 225 bags, each of volume 0.64 m3 can fit in Cuboidal granary.


Question 45.

A cube of side 6 cm is cut into a number of cubes each of side 2 cm. The number of cubes formed is
A. 6

B. 9

C. 12

D. 27


Answer:

Given: A cube of side 6 cm.


It is cut into a number of cubes each of side 2 cm.


Volume of a cube is given by: a3 (here ‘a’ is the side of a cube).


Let V1 be the volume of a cube with side 6cm


∴ V1 = a3


⇒ V1 = 63 = 216 cm3


Let V2 be the volume of a cube with side 2cm


∴ V1 = a3


⇒ V1 = 23 = 8 cm3


Let ‘n’ be the number of cubes of side 2cm which are cut from cube of side 6cm.


∴ n × V2 = v1


⇒ n × 8 = 216


⇒ n = = 27


∴ Total of 27 cubes each of side 2cm can be cut from a cube of side 6cm.


Question 46.

In a shower, 5 cm of rain falls. The volume of the water that falls on 2 hectares of ground, is
A. 100 m3

B. 10 m3

C. 1000 m3

D. 10000 m3


Answer:

Given: 5 cm of rain falls.


2 hectares of ground.


2 hectares = 20000 m3 (∵ 1 hectares = 10000 m3)


∴ Area of land which is filled with rain upto 5cm high is 20000 m3


Let V be the volume of the water on the land.


The volume of the water on the land is: area(of land) × height of the water on the land.


∴ V = Area(of land) × height


⇒ V = 20000 × (∵ 1cm = m )


⇒ V = 1000 m3


∴ The volume of the water that falls on 2 hectares of ground, is: 1000m3


Question 47.

Two cubes have their volumes in the ratio 1:27. The ratio of their surface areas is
A. 1:3

B. 1:8

C. 1:9

D. 1:18


Answer:

Given: Volumes of the cubes are in the ratio 1:27.


Volume of a cube is given by: a3 ( here ‘a’ is the side of the cube).


Surface are of a cube is given by: a2 ( here ‘a’ is the side of the cube).


Let a1, a2 be the side of first cube and second cube respectively.


∴ (a1)3: (a2)3 = 1:27


⇒ a1: a2 = ∛1 : ∛27


⇒ a1: a2 = 1 : 3


⇒ (a1)2: (a2)2 = 12: 32


⇒ (a1)2: (a2)2 = 1 : 9


∴ Ratio of the surface areas of the given cubes is 1:9.


Question 48.

The diameter of the base of a cylinder is 4 cm and its height is 14 cm. The volume of the cylinder is
A. 176cm3

B. 196 cm3

C. 276 cm3

D. 352 cm3


Answer:

Given: Diameter of the base of a cylinder is 4 cm.


Its height is 14 cm.


Volume of the Cylinder is given by: πr2h. (here r and h are radius and diameter respectively)


Let V be the volume of the cylinder.


∴ V = πr2h


⇒ V = π × ()2 × 14 (∵ 4 is the diameter and is the radius)


⇒ V = π × (2)2 × 14 = π × 4 × 14


⇒ V = 176 cm3


∴ The volume of the cylinder is 176 cm3


Question 49.

The diameter of a cylinder is 28 cm and its height is 20 cm. The total surface area of the cylinder is
A. 2993 cm2

B. 2992 cm2

C. 2292 cm2

D. 2229 cm2


Answer:

Given: Diameter of the base of a cylinder is 28 cm.


Its height is 20 cm.


Volume of the Cylinder is given by: 2πr(r + h). (here r and h are radius and diameter respectively)


Let S be the Surface area of the cylinder.


∴ V = 2πr(r + h)


⇒ S = 2 × π × () × [() + 20] (∵ 28 is the diameter and is the radius)


⇒ V = 2 × π × 14 × (14 + 20) = 2 × π × 14 × 24


⇒ V = 2992 cm2


∴ The Surface area of the cylinder is 2992 cm2


Question 50.

The height of a cylinder is 14 cm and its curved surface area is 264 cm2. The volume of the cylinder is
A. 308 cm3

B. 396 cm3

C. 1232 cm3

D. 1848 cm3


Answer:

Given: The height of a cylinder is 14 cm.


Its curved surface area is 264 cm2.


Curved surface area of a Cylinder is : 2πrh (here r and h are radius and height respectively)


∴ 2πrh = 264


⇒ 2 × π × r × 14 = 264


⇒r = = 3cm


∴ r = 3cm


Volume of a Cylinder is: πr2h


Let V be the volume of the Cylinder


∴ V = πr2h


⇒ V = π × 32 × 14 = 396cm3


∴ Volume of the Cylinder is 396cm3


Question 51.

The curved surface area of a cylinder is 1760 cm2 and its base radius is 14 cm. The height of the cylinder is
A. 10 cm

B. 15 cm

C. 20 cm

D. 40 cm


Answer:

Given: The radius of a cylinder is 14 cm.


Its curved surface area is 1760 cm2.


Curved surface area of a Cylinder is : 2πrh (here r and h are radius and height respectively)


∴ 2πrh = 1760


⇒ 2 × π × 14 × h = 1760


⇒h = = 20 cm


∴ h = 20cm


That is height of the given Cylinder is 20cm.


Question 52.

The ratio of the total surface area to the lateral surface area of a cylinder with base radius 80 cm and height 20 cm is
A. 2 : 1

B. 3 : 1

C. 4 : 1

D. 5 : 1


Answer:

Given: Radius and height of the Cylinder are 80cm and 20cm respectively.


Lateral Surface area of a Cylinder is : 2πrh


Let S1 be the Lateral surface area of the Cylinder


∴ S1 = 2πrh


⇒ S1 = 2 × π × 80 × 20 = 3200π


Total Surface area of a Cylinder is : 2πr(r + h)


Let S2 be the Total surface area of the Cylinder


∴ S2 = 2πr(r + h)


⇒ S2 = 2 × π × 80 × (80 + 20) = 16000π


Ratio of Total surface area to Lateral surface area id S2 : S1


∴ S2 : S1 = 16000π : 3200π


⇒ S2 : S1 = 5:1


∴ The ratio of the total surface area to the lateral surface area of a cylinder is 5:1


Question 53.

The curved surface area of a cylindrical pillar is 264 m2 and its volume is 924 m3 . The height of the pillar is
A. 4 m

B. 5 m

C. 6 m

D. 7 m


Answer:

Given: CSA of a cylindrical pillar is 264 m2


Volume of cylindrical pillar id 924 m3


CSA of a cylinder is : 2πrh (here r and h are radius and height respectively)


∴ 2πrh = 264 …. 1


Volume of the Cylinder is : πr2h (here r and h are radius and height respectively)


∴ πr2h = 924 …. 2


Divide eq –2 by eq –1


We get,


=


= ⇒ r = 7


∴ r = 7


Substitute ‘r’ in eq –1


∴ 2 × π × 7 × h = 264


⇒ 2 × × 7 × h = 264


⇒ 44 × h=264


⇒ h = = 6 cm


∴ height of the cylindrical pillar is 6 cm


Question 54.

The ratio between the radius of the base and the height of the cylinder is 2: 3. If its volume is 1617 cm3, the total surface area of the cylinder is
A. 308 cm2

B. 462 cm2

C. 540 cm2

D. 770 cm2


Answer:

Given: The ratio between the radius of the base and the height of the cylinder is 2: 3.


Volume of the Cylinder is 1617cm3


Let 2x and 3x be radius and height of the Cylinder respectively.


Volume of the Cylinder is given as: πr2h


∴ πr2h = 1617


⇒ π × (2x)2 × (3x) = 1617


⇒ 12π × x3 = 1617


⇒ x3 = = =


⇒ x = ∛ () =


∴ r = 2 × = 7cm


and, h = 3 × = 10.5 cm


Total surface area of a cylinder is: 2πrh(r + h)


Let S be the TSA of a cylinder


∴ S = 2πrh(r + h)


⇒ S = 2 × π × (7) × (7 + 10.5) = 2 × π × (7) × 17.5 = 770


∴ S = 770cm2


That is, Total surface area of the cylinder is 770 cm2


Question 55.

The radii of two cylinders are in the ratio 2:3 and their height in the ratio 5 : 3. The ratio of their volume is
A. 27 : 20

B. 20:27

C. 4 : 9

D. 9 : 4


Answer:

Given: The radii of two cylinders are in the ratio 2:3.


Heights of the cylinders are in the ratio 5:3.


Volume of cylinder is: πr2h (here r and h are radius and height of the cylinder respectively)


Let V1 be the volume of first cylinder


∴ V1 = π(r1)2h1


Let V2 be the volume of second cylinder


∴ V2 = π(r2)2h2


∴ V1 : V2 = π(r1)2h1 : π(r2)2h2


⇒ V1 : V2 = π × (2)2 × 5 : π × (3)2 × 3


⇒ V1 : V2 = 20π : 27π = 20:27


∴ V1 : V2 = 20:27


That is the ratio of their volume is 20:27.


Question 56.

Two circular cylinder of equal volume have their height in the ratio 1: 2. The ratio of their radii is
A. 1: √ 2

B. √ 2: 1

C. 1 : 2

D. 1 : 4


Answer:

Given: Two cylinders of equal volume.


Heights of the cylinders are in the ratio 1:2


Volume of cylinder is: πr2h (here r and h are radius and height of the cylinder respectively)


Let V1 be the volume of first cylinder


∴ V1 = π(r1)2h1


Let V2 be the volume of second cylinder


∴ V2 = π(r2)2h2


Here,


V1=V2


⇒ π(r1)2h1 = π(r2)2h2


⇒ (r1)2h1 = (r2)2h2


⇒ (r1)2 : (r2)2 = h2 : h1


⇒ (r1)2 : (r2)2 = 2 : 1


⇒ r1 : r2= √2 :1


∴ ratio of the radii of given cylinders is √2 :1


Question 57.

The radius of the base of a cone is 5 cm and its height is 12 cm. Its curved surface area is
A. 60π cm2

B. 65 π

C. 30 π cm2

D. none of these


Answer:

Given: The height of the cone is 12 cm.


The radius of the cone is 5 cm.


Curved surface area of a cone is : πrl (here r and l are radius and slant height respectively)


l = √(r2 + h2)


∴ πrl = πr√(r2 + h2)


Let S be the CSA of the cone.


∴ S = πr√(r2 + h2)


⇒ S = π × (5) × √((5)2 + (12)2)


⇒ S = π × (5) × √(25 + 144) = π × (5) × √(169)


⇒ S = π × (5) × 13 = 65π


∴ S = 65π cm2


That is CSA of the cone is 65π cm2


Question 58.

The diameter of the base of a cone is 42 cm and its volume is 12936 cm3. Its height is
A. 28 cm

B. 21 cm

C. 35 cm

D. 14 cm


Answer:

Given: Diameter of the base of a cone is 42 cm.


Volume of the cone is 12936 cm3.


Volume of the cone is given by: × π × r2 × h


× π × r2 × h = 12936


× π × ()2 × h = 12936 (here diameter = 42cm, ∴ r = = 21)


× π × (21)2 × h = 12936


⇒ h = = 28cm


∴ Height of the given cone is 28cm


Question 59.

The area of the base of a right circular cone is 154 cm2 and its height is 14cm. Its curved surface area is
A. 154 √ 5 cm2

B. 154 √ 7 cm 2

C. 77 √ 7 cm2

D. 77 √ 5cm2


Answer:

Given: Area of the base of a right circular cone is 154 cm2.


Height of the cone is 14 cm.


Curved surface area of a cone is : πrl (here r and l are radius and slant height respectively)


Area of the base is given by: πr2


∴ π × r2 = 154


⇒ r2 = = 49


⇒ r = √49 = 7cm


∴ radius of the base of the cone is 7cm


Now,


l = √(r2 + h2)


∴ πrl = πr√(r2 + h2)


Let S be the CSA of the cone.


∴ S = πr√(r2 + h2)


⇒ S = π × (7) × √((7)2 + (14)2)


⇒ S = π × (7) × √(49 + 196) = π × (7) × √(245)


⇒ S = π × (7) × 7√5 = × 7 × 7√5 = 154√5


∴ S = 154√5 cm2


That is CSA of the cone is 154√5 cm2


Question 60.

On increasing each of the radius of the base and the height of a cone by 20% its volume will be increased by
A. 20%

B. 40%

C. 60%

D. 72.8%


Answer:

Given: Radius of the base and the height of a cone is increased by 20%


The volume of the cone is : πr2h


New radius = r + r = r = r


Similarly, new radius = h


New Volume = × π × (r)2 × (h) = × πr2h


Increase in volume = × πr2h – πr2h = × πr2h


∴ Increase in % =( × 100 )% = 72.8%


∴ Volume will be increased by 72.8%


Question 61.

The radii of the base of a cylinder and a cone are in the ratio 3:4. If they have their height in the ratio 2:3, the ratio between their volumes is:
A. 12 cm

B. 14 cm

C. 15 cm

D. 18 cm


Answer:

Given: The radii of the base of a cylinder and a cone are in the ratio 3:4.


Heights of the base of a cylinder and a cone are in the ratio 2:3.


Volume of cylinder is: πr2h (here r and h are radius and height of the cylinder respectively)


Volume of cylinder is: πr2h


Let V1 be the volume of first cylinder


∴ V1 = π(r1)2h1


Let V2 be the volume of the cone.


∴ V2 = π(r2)2h2


∴ V1 : V2 = π(r1)2h1 : π(r2)2h2


⇒ V1 : V2 = π × (3)2 × 2 : × π × (4)2 × 3


⇒ V1 : V2 = 18π : × 48π = 18:16 = 9:8


∴ V1 : V2 = 9:8


That is the ratio of their volume is 9:8.


Question 62.

A metallic Cylinder of radius 8cm and height 2cm is melted and converted into a right circular cone of height 6cm. The radius of the base of the cone is
A. 4 cm

B. 5 cm

C. 6 cm

D. 8 cm


Answer:

Given: A metallic Cylinder of radius 8cm and height 2cm


Right circular cone of height 6cm


Volume of a cylinder is given by: π × r2 × h


Volume of a cone is given by: × π × r2 × h


Let V1 be the volume of the Cylinder


∴ V1 = π × r2 × h


⇒ V1 = π × (8)2 × (2) = 128π


Let V2 be the volume of the cone


∴ V2 = × π × r2 × h


⇒ V2 = × π × (r)2 × (6) = 2πr2


Here, Solid Cylinder is melted and made into a Solid cone.


∴ V1 = V2


⇒ 128π = 2πr2


⇒ 2πr2 = 128π


⇒ r2 = = 64


⇒ r2 = 64


⇒ r = √64 = 8cm


∴ Radius of the base of the cone is 8cm.


Question 63.

The height of the conical tent is 14m and its floor area is 346.5m2. How much canvas, 1.1m wide, will be required for it
A. 490 m

B. 525 m

C. 665 m

D. 860 m


Answer:

Given: The height of the conical tent is 14m and its floor area is 346.5m2.


Canvas of width 1.1m


Surface area of the tent is: πrl


Let be the Surface area of the cone.


Area of the floor is : πr2= 346.5 m2


⇒ πr2= 346.5


⇒ r2 = =


⇒ r = √( ) =


⇒ l = √(h2 + r2) (here l is the slant height and h , r are height and radius respectively of cone)


⇒ l = √(142 + ()2) = √(196 + ) = √ =


∴ S = πrl = π × =


∴ Area of the canvas required to cover the tent is: m2


∴ l × b = m2 (here l is length and b is breadth or width of the canvas)


⇒ l × 1.1 = ( width of the canvas is 1.1m)


⇒ l = = 525m


∴ Length of the canvas required to cover the tent is 525m


Question 64.

The diameter of a sphere is 14cm. Its Volume is
A. 1428 cm3

B. 1439 cm3

C.

D. 1440 cm3


Answer:

Given: Diameter of the sphere is 14cm


Radius of the sphere is = 7cm


Volume of the Sphere is given by: πr3 (here r is the radius of the sphere)


Let V be the volume of the sphere)


∴ V = πr3


⇒ V = π(7)3 = × 343


⇒ V = × 22 × 49 = = 1437


∴ Volume of the sphere is 1437


Question 65.

The ratio between the volumes of two spheres is 8:27. What is the ratio between the surface areas?
A. 2:3

B. 4:5

C. 5:6

D. 4:9


Answer:

Given: Volumes of the spheres are in the ratio 8:27.


Volume of a sphere is given by: πr3 (here ‘r’ is the radius of the sphere).


Surface are of a sphere is given by: 4πr2 (here ‘r’ is the radius of the sphere).


Let r1, r2 be the radii of first sphere and second sphere respectively.


π(r1)3: π(r2)3 = 2:27


⇒ (r1)3: (r2)3 = 8 : 27


⇒ r1: r2 = ∛8 : ∛27


⇒ (r1): (r2) = 2: 3


Now , here


4π(r1)2: 4π(r2)2 = (r1)2 : (r1)2


⇒ (r1)2 : (r1)2 = (2)2 : (3)2 = 4:9


∴ Ratio of the surface areas of the given spheres is 4:9.


Question 66.

A hollow metallic Sphere with external diameter8cm and internal diameter 4cm is melted and moulded into a cone having base diameter 8cm. The height of the cone is
A. 12 cm

B. 14 cm

C. 15 cm

D. 18 cm


Answer:

Given: A hollow metallic Sphere with external diameter8cm and internal diameter 4cm


A cone having base diameter 8cm


Radius of the cone is: = 4cm


External Radius of the sphere is = 4cm


Internal Radius of the sphere is = 2cm


Volume of a Hollow Sphere is given by: π((r1)3 – (r2)3) (here r1 and r2 are External and internal radii of the hollow sphere respectively)


Let V1 be the Volume of the Hollow sphere.


∴ V1 = π((r1)3 – (r2)3)


⇒ V1 = π((4)3 – (2)3) = π(64 – 8) = π(56)


Volume of the cone is given by × π × r2 × h


Let V2 be the volume of the cone


∴ V2 = × π × r2 × h


⇒ V2 = × π × (4)2 × h


Here, Hollow sphere is melted and moulded into a cone.


∴ V1 = V2


π(56) = × π × (8)2 × h


× π × 16 × h = × π × (56)


⇒ h = = 14cm


∴ Height of the cone is 14cm


Question 67.

A metallic cone having base radius 2.1 cm and height 8.4 cm is melted and moulded into a sphere. The radius of the sphere is
A. 2.1 cm

B. 1.05 cm

C. 1.5 cm

D. 2 cm


Answer:

Given: A metallic cone having base radius 2.1 cm and height 8.4 cm


Volume of a cone is given by: × π × r2 × h


Let V1 be the volume of the cone


∴ V1 = × π × r2 × h


⇒ V1 = × π × (2.1)2 × (8.4)


Volume of a sphere is given by: × π × r3


Let V2 be the volume of the cone


∴ V1 = × π × r3


Here,


V1 = V2


× π × (2.1)2 × (8.4) = × π × r3


⇒ r3 = × π × (2.1)2 × (8.4) × = (2.1)3


⇒ r3 = (2.1)3


⇒ r = ∛(2.1)3 = 2.1 cm


∴ Radius of the sphere is 2.1 cm


Question 68.

The volume of a hemisphere is 19404 cm3. The total surface area of the hemisphere is
A. 4158 cm2

B. 16632 cm2

C. 8316 cm2

D. none of these


Answer:

Given: The volume of a hemisphere is 19404 cm3.


Volume of the hemisphere is given by: × π × r3


× π × r3 = 19404


⇒ r3 = 19404 × = 9261


⇒ r3 = 9261 ⇒ r = ∛9261


⇒ r = 21


Now, Total surface area of hemisphere is given by: 3πr2


Let S be the TSA


∴ S = 3 × π × r2


⇒ S = 3 × π × (21)2 = 4158 cm2


∴ S = 4158 cm2


That is TSA of the given sphere is 4158 cm2


Question 69.

The surface area of a sphere is 154 cm2. The volume of the sphere is
A.

B.

C.

D. none of these


Answer:

Given: The surface area of a sphere is 154 cm2.


TSA of the sphere is given by: 4 × π × r2


∴ 4 × π × r2 = 154


⇒ r2 = 154 × =


⇒ r2 = ⇒ r = √


⇒ r =


Now, Volume of hemisphere is given by: πr3


Let V be the Volume of the hemisphere


∴ V = × π × r3


⇒ V = × π × ()3 = cm3 = 179 cm3


∴ V = 179 cm3


That is Volume of the given sphere is 179 cm3


Question 70.

The total surface area of a hemisphere of radius 7 cm is
A. (588π) cm2

B. (392π) cm2

C. (147π) cm2

D. (98π) cm2


Answer:

Given: Radius of the hemisphere: 7cm .


TSA of the hemisphere is given by: 3πr2


Let S be the TSA of the hemisphere.


∴ S = 3πr2


⇒ S = 3 × π × (7)2 = 147π cm2


∴ TSA of the hemisphere is 147π cm2


Question 71.

The circular ends of a bucket are of radii 35 cm and 14 cm and the height of the bucket is 40 cm. Its volume is
A. 60060 cm3

B. 80080 cm3

C. 70040 cm3

D. 80160 cm3


Answer:

Given: The circular ends of a bucket are of radii 35 cm and 14 cm and the height of the bucket is 40 cm.


Bucket is in the shape of frustum.


Let V be the Volume of the Bucket(Frustum)


Volume of the frustum is given by: × h × (R2 + r2 + Rr) (here r and R are the radii of smaller and larger circular ends respectively)


∴ V = × h × (R2 + r2 + R × r)


⇒ V = × 40 × (352 + 142 + 35 × 14)


⇒ V = × 40 × (1225 + 196 + 490) = × 40 × (1911)


⇒ V = × 40 × (1911) = 80080 cm3


∴ The volume of the bucket is: 80080 cm3


Question 72.

If the radii of the end of a bucket are 5 cm and 15 cm and it is 24 cm high then its surface area is (use π = 3.14)
A. 1815.3 cm3

B. 1711.3 cm2

C. 2025.3 cm2

D. 2360 cm2


Answer:

Given: The radii of the end of a bucket are 5 cm and 15 cm and it is 24 cm high


Bucket is in the shape of frustum.


TSA of a frustum of a cone = πl(r1 + r2) + πr12 + πr22 (here l , r1, r2 are the slant height, radii of the frustum)


Let S be the TSA of the bucket


∴ S = πl(r1 + r2) + π(r2)2 (here , top of the bucket is not closed but bottom is closed, ∴ π(r2)2 = 0 )


l = √(h2 + (R-r)2)


⇒ S = π × √(h2 + (R-r)2) × (r1 + r2) + π(r2)2


⇒ S = π × √(242 + (15-5)2) × (5 + 15) + π × (5)2


⇒ S = π × √(576 + 100) × (20) + π × 25


⇒ S = π × √(676) × (20) + π × 25


⇒ S = π × 26 × (20) + π × 25


⇒ S = π × 520 + π × 25


⇒ S = π × (520 + 25)


⇒ S= 3.14 × 545 = 1711.3 cm2


∴ The surface area of the bucket is 1711.3 cm2


Question 73.

A circus tent is cylindrical to a height of 4 m and conical above it. If its diameter is 105 m and its slant height is 40 m, the total area of canvas required is
A. 1760 m2

B. 2640 m2

C. 3960 m2

D. 7920 m2


Answer:

Given: A circus tent is cylindrical to a height of 4 m and conical above it.


Its diameter is 105 m and its slant height is 40 m.


CSA of the Cylinder is given by: 2πrh (here, r and h are radius and height respectively)


Let V1 be the CSA of the cylindrical part of tent


∴ V1 = 2πrh


⇒ V1 = 2 × π × × 4 = 1320cm2 (here diameter is 105 cm, ∴ radius = cm)


CSA of the cone is given by: πrl (here, r and l are radius and slant height respectively)


Let V2 be the CSA of the conical part of tent


∴ V2 = πrl


⇒ V2 = π × × 40 = 6600 cm2 (here diameter is 105 cm, ∴ radius = cm)


∴ Total area of the canvas is V1 + V2


∴ V1 + V2 = 1320 + 6600 = 7920cm2


∴ Total area of the canvas is: 7920 cm2


Question 74.

Match the following columns:


Answer:

a—(q), b—(s), c—(p), d—(r)


a) Given: A solid metallic sphere of radius 8 cm


Solid right cones with height 4 cm and radius of the base 8 cm.


Volume of a metallic sphere is given by: × π × r3


Volume of a Right cone is given by: × π × r2 × h


Let V1 be the Volume of the metallic sphere.


∴ V1 = × π × r3 = × π × (8)3


Let V2 be the Volume of the Solid right cone.


∴ V2 = × π × r2 × h = × π × (8)2 × 4


Let ‘n’ be the number of right circular cones that are made from melting the metallic sphere.


∴ V1 = n × V2


× π × (8)3 = n × × π × (8)2 × 4


n = = 8


∴ 8 cones are formed from melting the metallic sphere.


b) Given: A 20-m-deep well with diameter 14 m ⇒ radius = 7cm


A platform 44 m by 14 m


Volume of a cylinder is given by: π × r2 × h


Volume of a platform(cuboid) is given by: l × b × h (here l, b, h are length, breadth, height respectively)


Let V1 be the Volume of the Well.


∴ V1 = π × r2 × h = π × (7)2 × 20


Let V2 be the Volume of the platform


∴ V2 = l × b × h = 44 × 14 × h


Here V1 = V2


∴ 44 × 14 × h = π × (7)2 × 20


⇒ h = = = 5cm


∴ h = 5cm


That is height of the platform is 5cm.


c) Given: A sphere of radius 6 cm


A cylinder of radius 4 cm


Volume of a metallic sphere is given by: × π × r3


Volume of a Cylinder is given by: π × r2 × h


Let V1 be the Volume of the metallic sphere.


∴ V1 = × π × r3 = × π × (6)3


Let V2 be the Volume of the Solid Cylinder.


∴ V2 = π × r2 × h = π × (4)2 × h


Here V1 = V2


∴ π × (4)2 × h = × π × (6)3


⇒ h = = 18cm


∴ h = 18cm


That is height of the cylinder is 18 cm.


d) Given: Volume ratio of two Spheres is: 64:27


Volume of the Sphere is: × π × r3 (where r is radius of sphere)


Surface area of the sphere is: 4 × π × r2 (where r is radius of sphere)


Let V1 and V2 be the volumes of different spheres.


V1: V2 = 64:27


× π × (r1)3: × π × (r2)3 = 64:27 (here r1 and r2 are the radii of V1 and V2 respectively)


(r1)3: (r2)3 = 64:27


r1: r2 = ∛64:∛27


r1: r2 = 4:3


Now,


Let S1 and S2 be the Surface areas of the spheres.


∴ S1:S2 = 4 × π × (r1)2:4 × π × (r2)2 (here r1 and r2 are the radii of S1 and S2 respectively)


⇒ S1:S2 = (r1)2: (r2)2


⇒ S1:S2 = (4)2: (3)2


⇒ S1:S2 = 16:9


∴ The ratios of the Surface areas is: 16:9



Question 75.

Match the following columns:


Answer:

a—(q), b—(s), c—(p), d—(r)


a) Given: The radii of the circular ends of a bucket are 20 cm and 10 cm respectively.


Height of the bucket is 30cm


Bucket is in the shape of frustum.


Let V be the Volume of the Bucket(Frustum)


Volume of the frustum is given by: × h × (R2 + r2 + Rr) (here r and R are the radii of smaller and larger circular ends respectively)


∴ V = × h × (R2 + r2 + R × r)


⇒ V = × 30 × (202 + 102 + 20 × 10)


⇒ V = × 30 × (400 + 100 + 200) = × 30 × (700)


⇒ V = × 30 × (700) = 22000 cm3


∴ The capacity of the bucket is: 22000 cm3


b) Given: Height of the frustum of a cone: 15 cm


radii of the Circular ends: 28cm and 20 cm.



Here slant height h can be found by using Pythagoras theorem.


∴ s2 = h2 + (R-r)2 (here R is 28cm and r is 20cm)


⇒ s2 = 152 + (28—20)2


⇒ s2 = 152 + (8)2


⇒ s2 = 225 + 64


⇒ s2 = 289


⇒ s = √289 = 17


∴ Slant height 0f the Frustum is 17cm


c) Given: The radii of the end of a bucket are 33 cm and 27 cm and its slant height is 10 cm


TSA of a frustum of a cone = πl(r1 + r2) + πr12 + πr22 (here l , r1, r2 are the slant height, radii of the frustum)


Let S be the TSA of the Frustum.


∴ S = πl(r1 + r2) + π(r2)2 + π(r1)2


⇒ S = π × 10 × (33 + 27) + π(33)2 + π(27)2


⇒ S = π(600 + 1089 + 729) = 2418π


∴ TSA of frustum is 2418π.


d) Given: Three solid metallic sphere of radii 3 cm, 4 cm and 5 cm.


Volume of the Solid sphere is: πr3 (here r is the radius of the sphere).


Let V1 be the volume of the sphere with radius 3cm.


Let V2 be the volume of the sphere with radius 4cm.


Let V3 be the volume of the sphere with radius 5cm.


∴ V1 = πr3 = π(3)3


∴ V2 = πr3 = π(4)3


∴ V3 = πr3 = π(5)3


Here,


Let V be the Volume of the sphere that is formed by melting the spheres with volumes V1 , V2 , V3 .


∴ V = V1 + V2 + V3 = π(3)3 + π(3)3 + π(3)3


⇒ V = π(33 + 43 + 53) = π(27 + 64+ 125) = π(216)


⇒ V = π(216) = π(6)3


Here, we can see that radius of the Sphere is 6cm, ∴ diameter = 2 × 6 = 12cm


∴ Diameter of sphere formed by melting spheres with volumes V1, V2 , V3 is 12cm



Question 76.

Each question consists of two statements, namely, Assertion (A) and Reason (R). For selecting the correct answer, use the following code:

Assertion (A): If the radii of the circular ends of a bucket 24 cm high are 15 cm and 5 cm respectively, then the surface area of the bucket is 545 π cm2.

Reason (R): If the radii of the circular ends of the frustum of a cone are R and r respectively and its height is h, then its surface area is π {R2 + r2 + l(R - r)},where l2 = h2 + (R –r)2.
A. Both Assertion (A) and Reason (R) are true and Reason (R) is a correct explanation of Assertion (A).

B. Both Assertion (A) and Reason (R) are true but Reason (R) is not a correct explanation of Assertion (A).

C. Assertion (A) is true and Reason (R) is false.

D. Assertion (A) is false and Reason (R) is true.


Answer:

Assertion is wrong and Reason is Wrong.


Explanation:


Assertion (A):


Given: The radii of the end of a bucket are 5 cm and 15 cm and it is 24 cm high


Bucket is in the shape of frustum.


TSA of a frustum of a cone = πl(r1 + r2) + πr12 + πr22 (here l , r1, r2 are the slant height, radii of the frustum)


Let S be the TSA of the bucket


∴ S = πl(r1 + r2) + π(r2)2 (here , top of the bucket is not closed but bottom is closed, ∴ π(r2)2 = 0 )


l = √(h2 + (R-r)2)


⇒ S = π × √(h2 + (R-r)2) × (r1 + r2) + π(r2)2


⇒ S = π × √(242 + (15-5)2) × (5 + 15) + π × (5)2


⇒ S = π × √(576 + 100) × (20) + π × 25


⇒ S = π × √(676) × (20) + π × 25


⇒ S = π × 26 × (20) + π × 25


⇒ S = π × 520 + π × 25


⇒ S = π × (520 + 25)


⇒ S= 3.14 × 545 = 1711.3 cm2


∴ The surface area of the bucket is 1711.3 cm2


Reason(R):


Here,


Surface area is π {R2 + r2 + l(R + r)},where l2 = h2 + (R –r)2.


Assertion is wrong and Reason is Wrong.


Question 77.

Each question consists of two statements, namely, Assertion (A) and Reason (R). For selecting the correct answer, use the following code:

Assertion (A): A hemisphere of radius 7 cm is to be painted outside on the surface. The total cost of painting at Rs 5 per cm2 is Rs 2300.

Reason (R): The total surface area of a hemisphere is 3πr2.
A. Both Assertion (A) and Reason (R) are true and Reason (R) is a correct explanation of Assertion (A).

B. Both Assertion (A) and Reason (R) are true but Reason (R) is not a correct explanation of Assertion (A).

C. Assertion (A) is true and Reason (R) is false.

D. Assertion (A) is false and Reason (R) is true.


Answer:

Assertion is wrong and Reason is correct.


Explanation:


Assertion (A):


Given: A hemisphere of radius 7 cm.


Total surface area of the hemisphere is: 3πr2


Let S be the TSA of the hemisphere.


∴ S = 3πr2 = 3π(7)2


∴ S = 462cm2


The total cost to pain it: 462 × 5 = Rs 2310


Reason(R):


The total surface area of a hemisphere is 3πr2.


∴ Assertion is wrong and Reason is correct.


Question 78.

Each question consists of two statements, namely, Assertion (A) and Reason (R). For selecting the correct answer, use the following code:

Assertion (A): The number of coins 1.75 cm in diameter and 2 mm thick from a melted cuboid (10 cm × 5.5 cm × 3.5 cm) is 400.

Reason (R): Volume of a cylinder of base radius r and height h is given by V= (πr2h) cubic units. And, area of a cuboid = (l × b × h) cubic units.
A. Both Assertion (A) and Reason (R) are true and Reason (R) is a correct explanation of Assertion (A).

B. Both Assertion (A) and Reason (R) are true but Reason (R) is not a correct explanation of Assertion (A).

C. Assertion (A) is true and Reason (R) is false.

D. Assertion (A) is false and Reason (R) is true.


Answer:

Assertion is correct and Reason is correct explanation of the given assertion.


Explanation:


Assertion (A):


Given: A coin which is 1.75 cm in diameter and 2 mm thick.


A cuboid with dimensions (10cm × 5.5cm × 3.5cm)


A coin is in the form of a cylinder.


Volume of the Cylinder is given by: πr2h


Let V1 be the volume of the coin.


∴ V1 = πr2h


⇒ V1 = π × ()2 × (here, 1.75 is the diameter,∴ radius = , and 1mm = cm)


⇒ V1 = π × (0.875)2 × 0.2 = cm2


Volume of a cuboid is given by: l × b × h


Let V2 be the volume of the of the cuboid.


∴ V2 = l × b × h


⇒ V2 = 10 × 5.5 × 3.5 = 192.5 cm2


Let ‘n’ be the number of coin melted .


∴ n × V1 = V2


⇒ n × = 192.5


⇒ n = 192.5 × = 400


∴ n = 400


That is 400 coin when melted can be moulded into a cuboid of given dimensions.


Reason(R):


Volume of a cylinder of base radius r and height h is given by V= (πr2h) cubic units. And, area of a cuboid = (l × b × h) cubic units.


∴ Assertion is correct and Reason is correct explanation of the given assertion.


Question 79.

Each question consists of two statements, namely, Assertion (A) and Reason (R). For selecting the correct answer, use the following code:

Assertion (A): If the volumes of two sphere are in the ratio 27:8 then their surface areas are in the ratio 3:2

Reason (R): Volume of a sphere .

Surface area of a sphere = 4πR2.
A. Both Assertion (A) and Reason (R) are true and Reason (R) is a correct explanation of Assertion (A).

B. Both Assertion (A) and Reason (R) are true but Reason (R) is not a correct explanation of Assertion (A).

C. Assertion (A) is true and Reason (R) is false.

D. Assertion (A) is false and Reason (R) is true.


Answer:

Assertion is wrong and Reason is Wrong.


Explanation:


Assertion (A):


Given: volumes of two sphere are in the ratio 27:8.


Volume of the sphere is given by: πr3


Let V1 be the volume of the first sphere.


Let V2 be the volume of the first sphere.


∴ V1:V2 = π(r1)3 : π(r2)3


⇒ 27:8 = (r1)3 : (r2)3


⇒ r1 : r2 = 3:2


Surface area of the sphere is given by: 4πr2


Let S1 be the Surface area of the sphere.


Let S2 be the Surface area of the sphere.


∴ S1 : S2 = 4π(r1)2:4π(r2)2


⇒ S1 : S2 = (r1)2: (r2)2


⇒S1 : S2 = (3)2: (2)2


⇒S1 : S2 = 9:4


Reason(R):


Volume of a sphere = πr3.


Surface area of a sphere = 4πR2.


Assertion is wrong and Reason is Wrong.


Question 80.

Each question consists of two statements, namely, Assertion (A) and Reason (R). For selecting the correct answer, use the following code:

Assertion (A): The curved surface area of a cone of base radius 3 cm and height 4 cm is (15π) cm2.

Reason (R): Volume of a cone = πr2h.
A. Both Assertion (A) and Reason (R) are true and Reason (R) is a correct explanation of Assertion (A).

B. Both Assertion (A) and Reason (R) are true but Reason (R) is not a correct explanation of Assertion (A).

C. Assertion (A) is true and Reason (R) is false.

D. Assertion (A) is false and Reason (R) is true.


Answer:

Assertion is correct and Reason is Wrong.


Explanation:


Assertion (A):


Given: A cone of radius 3cm and height 4cm.


CSA of the cone is given by: πrl (here r is radius and l is slant height)


l = √(h2 + r2)


Let S be the CSA of the cone


∴ S = πrl = π × r × √(h2 + r2)


⇒ S = π × 4 × √(42 + 32) = π × 3 × √(16 + 9) = π × 3 × √(25) = π × 3 × 5 = 15π


∴ S =15π


Reason (R):


Volume of a cone is : πr2h


∴ Assertion is correct and Reason is Wrong.



Formative Assessment (unit Test)
Question 1.

Find the number of solid sphere, each of diameter 6 cm, that could be moulded to

Form a solid metallic cylinder of height 45 cm and diameter 4 cm.


Answer:

Given: Diameter of each solid sphere = 6 cm


∴ Radius of each sphere = 6/2 = rs = 3 cm


Diameter of cylinder = 4 cm


∴ Radius of cylinder = 4/2 = rc = 2 cm


Height of cylinder = h = 45 cm


Formula: volume of sphere = (4/3) × π × rs3


Volume of cylinder = π × rc2 × h


Spheres are moulded to form cylinder which means the volume remains the same


Let ‘n’ be the number of spheres required


i.e. volume of n spheres = volume of cylinder


∴ n × (4/3) × π × 33 = π × 22 × 45


n × 4 × 27 = 4 × 3 × 45


n = 45/9 = 5


Number of solid spheres made = 5



Question 2.

Two right circular cylinder of equal volume have their height in the ratio 1:2. What is the ratio of their radii?


Answer:

Let the two cylinders be with volume V1 and V2 with their respective radii and height as r1, r2 and h1, h2


Now given ratio of their heights i.e. h1:h2 = 1:2


∴ h1/h2 = 1/2


Volume of cylinder = πr2h


Given that volumes of both cylinder are equal i.e. V1 = V2


∴ π × r12 × h1 = π × r22 × h2


h1/h2 = r22/ r12


r22/ r12 = 1/2


r2/r1 = 1/√2


r1/r2 = √2/1


Therefore the ratio of their radii is r1:r2 = √2:1



Question 3.

A circus tent is cylindrical to a height of 4 m and conical above it. If its diameter is 105 m and its slant height is 40 m, find the total area of the canvas required.


Answer:

Given: diameter of base of cone and the cylinder = 105 m


∴ Radius of cylinder = rcl = 105/2 = 51 m


Radius of cone = rco = 105/2 = 51 m


Height of cylinder = h = 4 m


Slant height of cone = l = 40 m


Formula: Surface area of cylinder = 2πrclh + 2πrcl2


Surface area of cone = πrco2 + πrcol


Since we don’t require canvas for the top surface and bottom surface of cylinder and also for the base of cone we should subtract those areas from the surface area


Area of upper and lower surfaces of cylinder = 2πrcl2


∴ Area of canvas required for cylinder = 2πrclh + 2πrcl2 - 2πrcl2


= 2πrclh


= 2 × 3.14 × 51 × 4


= 1281.12 m2


Area of base of cone = πrco2


∴ area of canvas required for cone = πrco2 + πrcol - πrco2


= πrcol


= 3.14 × 51 × 40


= 6405.6 m2


Total area of canvas required = Area of canvas required for cylinder + area of canvas


required for cone


= 1281.12 + 6405.6


= 7686.72 m2


∴ Total area of canvas required = 7686.72 m2



Question 4.

The radii of the top and bottom of a bucket of slant height 45 cm are 28 cm and 7 cm respectively. Find the curved surface area of the bucket.


Answer:

Given: slant height of bucket = l = 45 cm


Radius of bottom circle = r = 7 cm


Radius of top circle = R = 28 cm


As the bucket is in the form of frustum


Formula: Total surface area of frustum = πr2 + πR2 + π(R + r)l cm2


Now we have asked curved surface area, so we should subtract the top and bottom surface


areas which are flat circles.


Surface area of top = πr2


Surface area of bottom = πR2


∴ Curved surface area = total surface area – πr2 - πR2 cm2


= πr2 + πR2 + π(R + r)l – πr2 - πR2 cm2


= π(R + r)l cm2


= 3.14 × (28 + 7) × 45 cm2


= 3.14 × 35 × 45 cm2


= 4945.5 cm2


Therefore, curved surface area of bucket = 4945.5 cm2



Question 5.

A solid metal cone with radius of base 12 cm and height 24 cm is melted to form solid spherical balls of diameter 6 cm each. Find the number of balls formed.


Answer:

Given: base radius of cone = rc = 12 cm


Height of cone = h = 24 cm


Diameter of spherical ball = 6 cm


Radius of spherical ball = rs = 6/2 = 3 cm


Formula: volume of cone = (1/3)πrc2h


Volume of sphere = (4/3)πrs3


Let n be the number of spherical balls made


As the cone is melted and then the spherical balls are made therefore the volume remains same


i.e. volume of n spherical balls made = volume of cone


∴ n × (4/3) × π × rs3 = (1/3) × π × rc2 × h


n × 4 × 33 = 122 × 24


n × 9 = 12 × 24


n = 32


∴ Number of balls formed = 32



Question 6.

A hemisphere bowl of internal diameter 30 cm is full of a liquid. This liquid is filled into cylindrical - shapes bottles each of diameter 5 cm and height 6 cm. How many bottles are required?


Answer:

Given: diameter of hemisphere = 30 cm


∴ Radius of hemisphere = rh = 30/2 = 15cm


Diameter of cylindrical shaped bottles = 5 cm


∴ radius of cylindrical shaped bottles = 5/2 = rc = 2.5 cm


Height of cylindrical shaped bottle = h = 6 cm


Formula: volume of hemisphere = (volume of sphere/2) = (2/3)πrh3


Volume of cylinder = πrc2h


Let ‘n’ bottles are required


As we are filling the cylindrical bottles with liquid in hemispherical bowl hence we can say that


volume of liquid in cylindrical bottles = volume of liquid in hemisphere


∴ n × π × rc2 × h = (2/3) × π × rh3


n × 2.52 × 6 × 3 = 2 × 153


n × 6.25 × 9 = 3375


n = 3375/56.25


n = 60


Therefore 60 cylindrical shaped bottles are required to fill the liquid from hemispherical bowl.



Question 7.

A solid metallic sphere of diameter 21 cm is melted and recast into small cones, each of diameter 3.5 cm and height 3 cm. Find the number of cones so formed.


Answer:

Given: diameter of sphere = 21 cm


∴ radius of sphere = rs = (21/2) cm


Diameter of cone = 3.5 cm


∴ radius of cone = rc = 3.5/2 = 1.75 = (7/4) cm


Height of cone = h = 3 cm


Formula: volume of sphere = (4/3)πrs3


Volume of cone = (1/3)πrc2h


Sphere is melted and then cones are made from molten metal therefore the volume remains same


Let ‘n’ be the number of cones made


i.e. volume of n cones = volume of sphere


∴ n × (1/3) × π × rc2 × h = (4/3) × π × rs3




Therefore number of cones formed = 504



Question 8.

The diameter of a sphere is 42 cm. It is melted and drawn into a cylindrical wire of diameter 2.8 cm. Find the length of the wire.


Answer:

Given: diameter of sphere = 42 cm


∴ radius of sphere = 42/2 = rs = 21 cm


Diameter of cylindrical wire = 2.8 cm


∴ Radius of cylindrical wire = rc = 1.4 = (7/5)cm


Let l be the length of wire


Formula: volume of sphere = (4/3)πrs3


Volume of wire = πrc2l


Sphere is melted and wire is made from it


∴ volume of sphere = volume of wire


(4/3)πrs3 = πrc2l


4 × 213 = 3 × (7/5)2 × l


4 × 21 × 9 × 25 = 3 × l


2100 × 3 = l


l = 6300 cm


Therefore length of wire formed = 6300 cm = 63 meters



Question 9.

A drinking glass is in the shape of frustum of a cone of height 21 cm with 6 cm and 4 cm as the diameters of its two circular ends. Find the capacity of the glass.


Answer:

Given: Height of glass = h = 21 cm


Diameter of lower circular end of glass = 4 cm


Diameter of upper circular end of glass = 6 cm


∴ Radius of lower circular end = r = 4/2 = 2 cm


∴ Radius of upper circular end = R = 6/2 = 3 cm



Capacity of glass = volume of frustum



= 22 × (9 + 4 + 6)


= 22 × 19


= 418 cm3


∴ Capacity of glass = 418 cm3



Question 10.

Two cubes, each of volume 64 cm3, are joined end to end. Find the total surface area of the resulting cuboid.


Answer:


volume of each cube = 64 cm3


Let a be the side length of each cube


∴ a3 = 64


a = 4 cm


The figure shows both the cubes joined together after joining we get a cuboid of length 2a and breadth a and height a


Length of cuboid formed = l = 2a


l = 8 cm


Breadth of cuboid formed = b = 4 cm


Height of cuboid formed = h = 4 cm


In the cuboid so formed there are 4 rectangular surfaces of length l = 8 and breadth b = 4 and 2 square surfaces of length 4


Total surface area of cuboid = 4 × l × b + 2a2


= (4 × 8 × 4) + (2 × 42)


= 128 + 32


= 160 cm2


Total surface area of cuboid = 160 cm2



Question 11.

The radius of the base and the height of a solid right circular cylinder are in the ratio 2:3 and its volume is 1617 cm3. Find the total surface area of the cylinder. [Take π = 22/7.]


Answer:

Given: volume of cylinder = 1617 cm3


Let r be the radius of base and h be the height of cone


r:h = 2:3


∴ r/h = 2/3


3r = 2h


h = 3r/2 …(i)


Formula: volume of cylinder = πr2h


∴ 1617 = (22/7) r2h


r2h = 514.5


Using (i) we have


∴ r2 × (3r/2) = 514.5


3r3 = 1029


r3 = 343


r = 7 cm


h = 21/2 cm


Total surface area of cylinder = 2πr2 + 2πrh


= 2 × (22/7) × 72 + 2 × (22/7) × 7 × (21/2)


= 308 + 462


= 770 cm2


Therefore total surface area of cylinder = 770 cm2



Question 12.

A toy is in the form of a cone mounted on a hemisphere on a hemisphere of common base radius 7 cm. The total height of the toy is 31 cm. find the total surface area of the toy.


Answer:


Given: Height of toy = h = 31 cm


Radius of hemisphere = radius of base of cone = r = 7 cm


From the figure we can calculate height of cone as


Height of cone = hc = h - r


= 31 - 7 = 24 cm


∴ hc = 24 cm


Formula: surface area of hemisphere = surface area of sphere/2 = 2πr2


Curved surface area of cone = πrl


Where l is slant height



l = √(49 + 576) = 25 cm


∴ l = 25 cm


Total surface area of toy = curved surface area of cone + surface area of hemisphere


Surface area of hemisphere = 2 × π × r2


= 2 × (22/7) × 72


= 308 cm2


Curved surface area of cone = π × r × l


= (22/7) × 7 × 25


= 550 cm2


∴ Total surface area of toy = 308 + 550


= 858 cm2


∴ Total surface area of toy = 858 cm2



Question 13.

A hemispherical bowl of internal radius 9 cm is full of water. This water is to be filled in cylindrical bottles of diameter 3 cm and height 4 cm. find the number of bottles needed to fill the whole water of the bowl.


Answer:

Given: Radius of hemisphere = rh = 9 cm


Diameter of cylindrical shaped bottles = 3 cm


∴ radius of cylindrical shaped bottles = rc = 3/2 = 1.5 cm


Height of cylindrical shaped bottle = h = 4 cm


Formula: volume of hemisphere = (volume of sphere/2) = (2/3)πrh3


Volume of cylinder = πrc2h


Let ‘n’ bottles are required


As we are filling the cylindrical bottles with liquid in hemispherical bowl hence we can say that


volume of liquid in cylindrical bottles = volume of liquid in hemisphere


∴ n × π × rc2 × h = (2/3) × π × rh3


n × (3/2)2 × 4 × 3 = 2 × 93


n = 33 × 2


n = 27 × 2


n = 54


Therefore 54 cylindrical shaped bottles are required to fill the liquid from hemispherical bowl.



Question 14.

The surface areas of a sphere and a cube are equal. Find the ratio of their volumes. [Takes π = 22/7.]


Answer:

Let r be the radius of sphere and a be the side length of cube.


Let Ss be the surface area of sphere and Sc be the surface area of cube and Vs be volume of sphere and Vc be volume of cube


∴ Ss = 4πr2 and Sc = 4a2


Given that surface area of sphere and cube are equal


∴ Ss = Sc


4πr2 = 6a2


r2/ a2 = 3/2π



Vs = (4/3) πr3


Vc = a3


∴ Vs/Vc = 4πr3/3a3


Using (i)



∴ Vs/Vc = √21/√11


Therefore ratio of their volumes is Vs:Vc = √21:√11



Question 15.

The slant height of the frustum of a cone is 4 cm and the perimeters (i.e. , circumferences) of its circular ends are 18 cm and 6 cm. Find the curved surface area of the frustum.


Answer:

Given: perimeter of upper circle = 18 cm


Perimeter of lower circle = 6 cm


Slant height of frustum = l = 4 cm


Formula: Total surface area of frustum = πr2 + πR2 + π(R + r)l cm2


Let r be the radius of lower circle and R be the radius of upper circle


Now perimeter of circle = circumference of circle = 2π × radius


∴ Perimeter of upper circle = 2πR


18 = 2 × π × R


R = 9/π cm


Perimeter of lower circle = 2πr


6 = 2 × π × r


r = 3/π cm


Now we have asked curved surface area, so we should subtract the top and bottom surface areas which are flat circles.


Surface area of top = πR2


Surface area of bottom = πr2


∴ Curved surface area = total surface area - πr2 - πR2 cm2


= πr2 + πR2 + π(R + r)l - πr2 - πR2 cm2


= π(R + r)l cm2


= π × [(9/π) + (3/π)] × 4 cm2


= (9 + 3) × 4 cm2


= 48 cm2


∴ curved surface area = 48 cm2



Question 16.

A solid is composed of a cylinder with hemisphere ends. If the whole length of the solid is 104 cm and the radius of each hemispherical end is 7 cm, find the surface area of the solid.


Answer:


Total length of solid = l = 104 cm as shown in figure


The solid consist of a cylinder and two hemispheres


Let the height of cylinder be h


We get the height h by subtracting the radii of left and right hemisphere from the total length l as seen in figure


∴ h = 104 - (7 + 7) cm


∴ h = 90 cm


Let r be the radius of hemisphere and the radius of cylinder


∴ r = 7 cm


There are two hemisphere one at left and one at right both of same radius r and two hemispheres make one sphere


Surface area of sphere = 4πr2


The flat circles i.e. the upper and lower circles of cylinder are not to be considered in the surface area of whole solid as they are covered by the hemispheres therefore for cylinder we will take its curved surface area


Curved surface area of cylinder = 2πrh


Surface area of solid = surface area of sphere + curved surface area of cylinder


= 4πr2 + 2πrh


= 2 × (22/7) × 7 × (2 × 7 + 90)


= 44 × 104


= 4576 cm2


Therefore total surface area of solid = 4576 cm2



Question 17.

From a solid cylinder whose height is 15 cm and diameter 16 cm, a conical cavity of the same height and same diameter is hollowed out. Find the total surface area of the remaining solid. [Use π = 3.14.]


Answer:

after removing the conical solid the cylinder would look like this



Given: height of cylinder = height of cone = h = 15 cm


Diameter of cylinder = diameter of base cone = 16 cm


∴ radius of cylinder = radius of base of cone = 16/2 = r = 8 cm


Formula: total surface area of cylinder = 2πr2 + πrh


Total surface area of cone = πrl + πr2


Where l is the slant height


l = √(r2 + h2)


l = √(82 + 152)


l = √289


l = 17 cm


In the solid as seen in figure we have the curved surface of cylinder and the base of cylinder as there is no top circular face of the cylinder we should subtract its area from total surface area of cylinder


Area of top circular surface of cylinder = πr2


∴ surface area of cylinder in solid = 2πr2 + πrh - πr2


= πr2 + πrh


= 3.14 × 8 × (8 + 15)


= 3.14 × 8 × 23


= 577.76 cm2


Now there is a hollow conical part with no base of the cone as seen in the figure therefore we should subtract the surface area of base of cone from the total surface area of cone


Surface area of base of cone = πr2


∴ Surface area of conical part in solid = πrl + πr2 - πr2


= πrl


= 3.14 × 8 × 17


= 427.04 cm2


Therefore total surface area of solid = surface area of cylinder in solid + surface area of


conical part in solid


= 577.76 + 427.04


= 1004.8 cm2


Surface area of solid = 1004.8 cm2



Question 18.

A solid rectangular block of dimension 4.4 m, 2.6 m, 1 m is cast into a hollow cylindrical pipe of internal radius 30 cm and thickness 5 cm. Find the length of the pipe.


Answer:


Given: length of the block = l = 4.4 m


Width of the block = w = 2.6 m


Height of the block = h = 1 m


Inner radius of pipe = r = 30 cm = 0.3 m


Thickness of pipe = t = 5 cm = 0.05 m


∴ outer radius of pipe as seen in the cross section of pipe = R = r + t = 30 + 5 = 35 cm = 0.35 m


Let l be the length of the pipe


Formula: volume of block = l × w × h


= 4.4 × 2.6 × 1


= 11.44 m3


Volume of block = 11.44 m3


Volume of pipe = π × (radius)2 × (length)


Volume of pipe material = volume of full pipe(R = 0.35) – volume of hollow cylinder(r = 0.3)


= π × 0.352 × l - π × 0.32 × l


= π × l × [(35/100)2 - (3/10)2]


= π × l × [(35/100) + (3/10)] × [(35/100) - (3/10)]



= (22/7) × l × (13/400) m3


∴ volume of pipe material = (22/7) × l × (13/400) m3


The pipe is made from the block


∴ volume of block = volume of pipe material


∴ 11.44 = (22/7) × l × (13/400)



∴ l = 28 × 4


∴ l = 112 m


Length of the pipe = 112 m



Question 19.

An open metal bucket is in the shape of a frustum of a cone, mounted on a hollow cylindrical base made of the same metallic sheet. The diameter of the two circular ends of the bucket are 45 cm and 25 cm, the total vertical height of the bucket is 40 cm and that of the cylindrical base is 6 cm. Find the area of the metallic sheet used to make the bucket. Also, find the volume of water the bucket can hold, in litres.


Answer:


Given: radius of upper circular end of frustum = R = 45 cm


Radius of lower circular end of frustum = radius of cylindrical base = r = 25 cm


Height of bucket = hb = 40 cm


Height of cylindrical base = hc = 6 cm


From the figure height of frustum = hf = hb - hc


= 40 - 6


∴ hf = 34 cm



Volume of cylinder = πr2hc


Curved surface area of cylinder = 2πrhc


curved surface area of frustum = π(R + r)l cm2


Where l = slant height




= √(400 + 1156)


= 39.44 cm


Area of metallic sheet used = curved surface area of frustum + curved surface area of base


cylinder + area of base circle of cylinder


now, curved surface area of frustum = π × (R + r) × l cm2


= (22/7) × (45 + 25) × 39.44 cm2


= 22 × 10 × 39.44 cm2


= 8676.8 cm2


Curved surface area of base cylinder = 2πrhc


= 2 × (22/7) × 25 × 6


= 942.85 cm2


Surface area of base circle of cylinder = πr2


= (22/7) × 252


= 1964.28 cm2


∴ Area of metallic sheet used = 8676.8 + 942.85 + 1964.28


= 11583.93 cm2


Therefore, area of metallic sheet used to make the bucket is 11583.93 cm2 i.e. 1.158393 m2


Volume of water bucket can hold = volume of bas cylinder + volume of frustum


Volume of base cylinder = πr2hc


= (22/7) × 252 × 6


= 11785.71 cm3




= 35.62 × 3775


= 134465.5 cm3


∴ volume of water bucket can hold = 11785.71 + 134465.5


= 146251.21 cm3


Now 1 litre is 1000 cm3


∴ 146251.21 cm3 = 146251.21/1000 = 146.25121 litres


Volume of water bucket can hold = 146.25121 litres



Question 20.

A farmer connect a pipe of internal diameter 20 cm from a canal into a cylindrical tank which is 10 m diameter and 2 m deep. If the water flows through the pipe at the of 4 km/hr, in how much time will the tank be filled completely?


Answer:

Given: diameter of pipe = 20 cm


∴ radius of pipe = rp = 20/2 = 10 cm = 0.1 m


Diameter of tank = 10 m


∴ radius of cylindrical tank = rc = 10/2 = 5 m


Depth of cylindrical tank = height of cylindrical tank = h = 2 m


Rate of flow of water through pipe = 4 km/hr


1 km = 1000 m


4 km/hr = 4000 m/hr


Volume of water required to completely fill the tank is equal to the volume of cylinder


Time require to fill the tank = volume of cylindrical tank/volume of water flown through pipe per hr


volume of cylindrical tank = π × rc2 × h


= 3.14 × 25 × 2


= 157 m3


volume of water flown through pipe per hr = π × rp2 × 4000


= 3.14 × 0.12 × 4000


= 3.14 × 40


= 125.6 m3/hr


Time require to fill the tank = 157/125.6


= 1.25 hrs


Therefore, it will take 1.25 hours to fill the tank completely.